You are on page 1of 253

Question 1 of 74

gathered by dr. elbarky, for free, not intended for profit by anyone elsewhere.

 

A 62 year old man is identified as having a rectal cancer. Following diagnostic


work up no metastatic disease is identified. The tumour is 2cm from the anal
verge. On MRI, lesion is T2, N0. Which of the following represents the correct
course of action?

Proceed to abdomino-perineal excision of the colon and rectum (ELAPE)

Undertake a low anterior resection and loop ileostomy

Undertake a Hartmanns procedure

Offer radical external beam radiotherapy followed by abdomino-perineal


excision of the colon and rectum (ELAPE)

Offer radical radiotherapy followed by low anterior resection and loop


ileostomy

The tumour is too low for restorative surgery to be considered with an acceptable
functional outcome. The tumour will therefore require an ELAPE style abdomino
perineal resection. Since the lesion is T2 there is no prognostic benefit from
adding radiotherapy which will confer additional morbidity.

Please rate this question:

 Discuss and give feedback

Next question 

Colorectal cancer treatment

Patients diagnosed as having colorectal cancer should be completely staged using


CT of the chest/ abdomen and pelvis. Their entire colon should have been
evaluated with colonoscopy or CT colonography. Patients whose tumours lie below
the peritoneal reflection should have their mesorectum evaluated with MRI.

Once their staging is complete patients should be discussed within a dedicated


colorectal MDT meeting and a treatment plan formulated.
Treatment of colonic cancer
Cancer of the colon is nearly always treated with surgery. Stents, surgical bypass
and diversion stomas may all be used as palliative adjuncts. Resectional surgery is
gathered by dr. elbarky, for free, not intended for profit by anyone elsewhere.

the only option for cure in patients with colon cancer. The procedure is tailored to
the patient and the tumour location. The lymphatic drainage of the colon follows
the arterial supply and therefore most resections are tailored around the resection
of particular lymphatic chains (e.g. ileo-colic pedicle for right sided tumours).
Some patients may have confounding factors that will govern the choice of
procedure, for example a tumour in a patient from a HNPCC family may be better
served with a panproctocolectomy rather than segmental resection. Following
resection the decision has to be made regarding restoration of continuity. For an
anastomosis to heal the key technical factors include; adequate blood supply,
mucosal apposition and no tissue tension. Surrounding sepsis, unstable patients
and inexperienced surgeons may compromise these key principles and in such
circumstances it may be safer to construct an end stoma rather than attempting
an anastomosis.
When a colonic cancer presents with an obstructing lesion; the options are to
either stent it or resect. In modern practice it is unusual to simply defunction a
colonic tumour with a proximal loop stoma. This differs from the situation in the
rectum (see below).
Following resection patients with risk factors for disease recurrence are usually
offered chemotherapy, a combination of 5FU and oxaliplatin is common.

Rectal cancer
The management of rectal cancer is slightly different to that of colonic cancer.
This reflects the rectum's anatomical location and the challenges posed as a
result. Tumours located in the rectum can be surgically resected with either an
anterior resection or an abdomino - perineal resection. The technical aspects
governing the choice between these two procedures can be complex to appreciate
and the main point to appreciate for the MRCS is that involvement of the sphincter
complex or very low tumours require APER. In the rectum a 2cm distal clearance
margin is required and this may also impact on the procedure chosen. In addition
to excision of the rectal tube an integral part of the procedure is a meticulous
dissection of the mesorectal fat and lymph nodes (total mesorectal excision/
TME). In rectal cancer surgery invovlement of the cirumferential resection margin
carries a high risk of disease recurrence. Because the rectum is an extraperitoneal
structure (until you remove it that is!) it is possible to irradiate it, something which
cannot be offered for colonic tumours. This has a major impact in rectal cancer
treatment and many patients will be offered neoadjuvent radiotherapy (both long
and short course) prior to resectional surgery. Patients with T1, 2 and 3 /N0
disease on imaging do not require irradiation and should proceed straight to
surgery. Patients with T4 disease will typically have long course chemo
radiotherapy. Patients presenting with large bowel obstruction from rectal cancer
should not undergo resectional surgery without staging as primary treatment (very
different from colonic cancer). This is because rectal surgery is more technically
demanding, the anastomotic leak rate is higher and the danger of a positive
resection margin in an unstaged patient is high. Therefore patients with
obstructing rectal cancer should have a defunctioning loop colostomy.

Summary of procedures
The operations for cancer are segmental resections based on blood supply and
gathered by dr. elbarky, for free, not intended for profit by anyone elsewhere.

lymphatic drainage. These commonly performed procedures are core knowledge


for the MRCS and should be understood.

Site of Risk of
cancer Type of resection Anastomosis leak

Right colon Right hemicolectomy Ileo-colic Low


<5%

Transverse Extended right hemicolectomy Ileo-colic Low


<5%

Splenic Extended right hemicolectomy Ileo-colic Low


flexure <5%

Splenic Left hemicolectomy Colo-colon 2-5%


flexure

Left colon Left hemicolectomy Colo-colon 2-5%

Sigmoid High anterior resection Colo-rectal 5%


colon

Upper Anterior resection (TME) Colo-rectal 5%


rectum

Low rectum Anterior resection (Low TME) Colo-rectal 10%


(+/- Defunctioning
stoma)

Anal verge Abdomino-perineal excision of None n/a


colon and rectum

In the emergency setting, where the bowel has perforated, the risk of an
anastomotic breakdown is much greater, particularly when the anastomosis is
colon-colon. In this situation, an end colostomy is often safer and can be reversed
later. When resection of the sigmoid colon is performed and an end colostomy is
fashioned the operation is referred to as a Hartmans procedure. Whilst left sided
resections are more risky, ileo-colic anastomoses are relatively safe even in the
emergency setting and do not need to be defunctioned.

References
A review of the diagnosis and management of colorectal cancer and a summary of
the UK National Institute of Clinical Excellence guidelines is provided in:
Poston G, et al . Diagnosis and management of colorectal cancer: summary of
NICE guidance. BMJ 2011: 343: d 6751.
gathered by dr. elbarky, for free, not intended for profit by anyone elsewhere.

Next question 

Display my notes on this topic

          

Save my notes

Question stats

A 38.1%
B 15.3%
C 6.5%
D 26.1%
E 14%

38.1% of users answered this question correctly

Search eMRCS

Search term Go

 Open MRCS Part A textbook (../review/textbook.php)

External links

+ Suggest a link

Dashboard

Question 2 of 74
gathered by dr. elbarky, for free, not intended for profit by anyone elsewhere.

 

A 53 year old man has a 1cm polyp identified and completely removed during a
colonoscopy. Histology confirms a low grade adenoma. What is the correct follow
up?

Suggest participation in bowel cancer screening but no further routine


endoscopy

Repeat endoscopy in 5 years.

Repeat endoscopy in 3 years.

Segmental resection of the affected area.

Barium enema at 5 years.

In the UK, the guidance has now changed and patients like this are managed
expectantly with suggestion that they participate in bowel cancer screening
programmes.

Please rate this question:

 Discuss and give feedback

Next question 

Colonic polyps

Colonic Polyps
May occur in isolation, or greater numbers as part of the polyposis syndromes. In
FAP greater than 100 polyps are typically present. The risk of malignancy in
association with adenomas is related to size, and is the order of 10% in a 1cm
adenoma. Isolated adenomas seldom give risk of symptoms (unless large and
distal). Distally sited villous lesions may produce mucous and if very large,
electrolyte disturbances may occur.

Follow up of colonic polyps


Group Action

Colorectal cancer Colonoscopy 1 year post resection


gathered by dr. elbarky, for free, not intended for profit by anyone elsewhere.

Large non pedunculated colorectal One off scope at 3 years


polyps (LNPCP), R0 resection

Large non pedunculated colorectal Site check at 2-6 months and then a
polyps (LNPCP) R1 or non en bloc further scope at 12 months
resection

High risk findings at baseline One off surveillance at 3 years


colonoscopy

No high risk findings at baseline No colonoscopic surveillance and invite


colonoscopy participation in NHSBCSP programme
when due

High risk findings


More than 2 premalignant polyps including 1 or more advanced
colorectal polyps
OR
More than 5 pre malignant polyps

Exceptions to guidelines
If patient more than 10 years younger than lower screening age and has polyps but
no high risk findings, consider colonoscopy at 5 or 10 years.

Segmental resection or complete colectomy should be considered when:

1. Incomplete excision of malignant polyp


2. Malignant sessile polyp
3. Malignant pedunculated polyp with submucosal invasion
4. Polyps with poorly differentiated carcinoma
5. Familial polyposis coli
-Screening from teenager up to 40 years by 2 yearly sigmoidoscopy/colonoscopy
-Panproctocolectomy and Ileostomy or Restorative Panproctocolectomy.

Rectal polypoidal lesions may be amenable to trans anal endoscopic microsurgery.

References
Rutter MD et al. British Society of Gastroenterology/Association of Coloproctology
of Great Britain and Ireland/Public Health England post- polypectomy and post-
colorectal cancer resection surveillance guidelines. Gut 2019;0:123.
Next question 
gathered by dr. elbarky, for free, not intended for profit by anyone elsewhere.

Display my notes on this topic

          

Save my notes

Question stats

A 9.7%
B 18.2%
C 56.6%
D 9.6%
E 5.9%

9.7% of users answered this question correctly

Search eMRCS

Search term Go

 Open MRCS Part A textbook (../review/textbook.php)

External links

+ Suggest a link

Dashboard

3

Question 3 of 74
gathered by dr. elbarky, for free, not intended for profit by anyone elsewhere.

 

A 23 year old man is admitted to hospital with diarrhoea and severe abdominal
pain. He was previously well and his illness has lasted 18 hours. What is the likely
cause?

Laxative abuse

Clostridium difficile infection

Salmonella gastroenteritis infection

Campylobacter jejuni infection

Ulcerative colitis

Severe abdominal pain tends to favour Campylobacter infection.

Please rate this question:

 Discuss and give feedback

Next question 

Diarrhoea

World Health Organisation definitions


Diarrhoea: > 3 loose or watery stool per day
Acute diarrhoea < 14 days
Chronic diarrhoea > 14 days

Acute Diarrhoea

Gastroenteritis May be accompanied by abdominal pain or


nausea/vomiting

Diverticulitis Classically causes left lower quadrant pain,


diarrhoea and fever
Acute Diarrhoea

Antibiotic therapy More common with broad spectrum antibiotics


Clostridium difficile is also seen with antibiotic use
gathered by dr. elbarky, for free, not intended for profit by anyone elsewhere.

Constipation causing A history of alternating diarrhoea and constipation


overflow may be given
May lead to faecal incontinence in the elderly

Chronic
Diarrhoea

Irritable Extremely common. The most consistent features are


bowel abdominal pain, bloating and change in bowel habit. Patients
syndrome may be divided into those with diarrhoea predominant IBS and
those with constipation predominant IBS.
Features such as lethargy, nausea, backache and bladder
symptoms may also be present

Ulcerative Bloody diarrhoea may be seen. Crampy abdominal pain and


colitis weight loss are also common. Faecal urgency and tenesmus
may occur

Crohn's Crampy abdominal pains and diarrhoea. Bloody diarrhoea less


disease common than in ulcerative colitis. Other features include
malabsorption, mouth ulcers perianal disease and intestinal
obstruction

Colorectal Symptoms depend on the site of the lesion but include


cancer diarrhoea, rectal bleeding, anaemia and constitutional
symptoms e.g. Weight loss and anorexia

Coeliac In children may present with failure to thrive, diarrhoea


disease and abdominal distension
In adults lethargy, anaemia, diarrhoea and weight loss
are seen. Other autoimmune conditions may coexist

Other conditions associated with diarrhoea include:


Thyrotoxicosis
Laxative abuse
Appendicitis with pelvic abscess or pelvic appendix
Radiation enteritis

Diagnosis
Stool culture
Abdominal and digital rectal examination
Consider colonoscopy (radiological studies unhelpful)
Thyroid function tests, serum calcium, anti endomysial antibodies, glucose
gathered by dr. elbarky, for free, not intended for profit by anyone elsewhere.

Next question 

Display my notes on this topic

          

Save my notes

Question stats

A 9.7%
B 13.5%
C 23.6%
D 45.9%
E 7.3%

45.9% of users answered this question correctly

Search eMRCS

Search term Go

 Open MRCS Part A textbook (../review/textbook.php)

External links

+ Suggest a link

Question 4 of 74
gathered by dr. elbarky, for free, not intended for profit by anyone elsewhere.

 

A 73 year old lady presents with constipation and no organic disease is identified
on investigation. Which of the following types of laxatives works by direct bowel
stimulation?

Magnesium sulphate

Lactulose

Potassium sodium tatrate

Methylcellulose

Senna

Senna contains glycosides. It passes unchanged into the colon where bacteria
hydrolyse the glycosidic bond, releasing the anthracene derivatives. These
stimulate the myenteric plexus.

Please rate this question:

 Discuss and give feedback

Next question 

Laxatives

Bulk forming laxatives

Bran
Psyllium
Methylcellulose

Osmotic laxatives
Magnesium sulphate
Magnesium citrate
Sodium phosphate
Sodium sulphate
gathered by dr. elbarky, for free, not intended for profit by anyone elsewhere.

Potassium sodium tatrate


Polyethylene glycol
Docusate

Stimulant laxatives

Bisacodyl
Sodium picosulphate
Senna
Ricinoleic acid

Next question 

Display my notes on this topic

          

Save my notes

Question stats

A 13.2%
B 18.8%
C 10.9%
D 8.7%
E 48.4%

48.4% of users answered this question correctly

Search eMRCS

Search term Go

Question 5 of 74
gathered by dr. elbarky, for free, not intended for profit by anyone elsewhere.

 

A 43 year old male has been troubled with symptoms of post defecation bleeding
for many years. On examination, he has large prolapsed haemorroids, colonoscopy
shows no other disease. What is the best course of action?

Injection with 20% phenol

Injection with 80% phenol

Rubber band ligation

Excisional haemorrhoidectomy

Haemorrhoidal artery ligation

Prolapsed haemorroids are best managed surgically if symptomatic. Note that


phenol injections are usually only used for minor internal haemorroids. Where
phenol is used, low concentration phenol in oil is the correct agent, the 80%
phenolic solution above is used to ablate the nail bed in toe nail surgery! Either
way, phenol does not work for haemorrhoidal disease in general.

Please rate this question:

 Discuss and give feedback

Next question 

Benign proctology

Condition Features Treatment

Fissure in Painful, bright red rectal Stool softeners, topical


ano bleeding diltiazem or GTN, botulinum
toxin, Sphincterotomy
Haemorroids Painless, bright red rectal Stool softeners, avoid straining,
bleeding occurs following surgery (see below)
defecation and bleeds onto
the toilet paper and into the
gathered by dr. elbarky, for free, not intended for profit by anyone elsewhere.

toilet pan

Fistula in ano May initially present with an Lay open if low, no sphincter
abscess and then involvement or IBD, if complex,
persisting discharge onto high or IBD insert seton and
the perineum, separate consider other options (see
from the anus below)

Peri anal Peri anal swelling and Incision and drainage, leave the
abscess surrounding erythema cavity open to heal by
secondary intention

Pruritus ani Peri anal itching, occasional Avoid scented products, use
mild bleeding (if severe skin wet wipes rather than tissue,
damage) avoidance of scratching, ensure
no underlying faecal
incontinence

Overview of surgical therapies


Haemorroidal disease
The treatment of haemorroids is usually conservative. Acutely thrombosed
haemorroids may be extremely painful. Treatment of this acute condition is usually
conservative and consists of stool softeners, ice compressions and topical GTN or
diltiazem to reduce sphincter spasm. Most cases managed with this approach will
settle over the next 5-7 days. After this period there may be residual skin tags that
merit surgical excision or indeed residual haemorroidal disease that may
necessitate haemorroidectomy.
Patients with more chronic symptoms are managed according to the stage of their
disease, small mild internal haemorroids causing little symptoms are best
managed conservatively. More marked symptoms of bleeding and occasional
prolapse, where the haemorroidal complex is largely internal may benefit from
stapled haemorroidopexy. This procedure excises rectal tissue above the dentate
line and disrupts the haemorroidal blood supply. At the same time the excisional
component of the procedure means that the haemorroids are less prone to
prolapse. Adverse effects of this procedure include urgency, which can affect up to
40% of patients (but settles over 6-12 months) and recurrence. The procedure
does not address skin tags and therefore this procedure is unsuitable if this is the
dominant symptom.
Large haemorroids with a substantial external component may be best managed
with a Milligan Morgan style conventional haemorroidectomy. In this procedure
three haemorroidal cushions are excised, together with their vascular pedicle.
Excision of excessive volumes of tissue may result in anal stenosis. The procedure
is quite painful and most surgeons prescribe metronidazole post operatively as it
decreases post operative pain.

Fissure in ano
Probably the most efficient and definitive treatment for fissure in ano is lateral
gathered by dr. elbarky, for free, not intended for profit by anyone elsewhere.

internal sphincterotomy. The treatment is permanent and nearly all patients will
recover. Up to 30% will develop incontinence to flatus. There are justifiable
concerns about using this procedure in females as pregnancy and pelvic floor
damage together with a sphincterotomy may result in faecal incontinence. The
usual first line therapy is relaxation of the internal sphincter with either GTN or
diltiazem (the latter being better tolerated) applied topically for 6 weeks. Treatment
failures with topical therapy will usually go on to have treatment with botulinum
toxin. This leads to more permanent changes in the sphincter and this may
facilitate healing.
Typical fissures usually present in the posterior midline, multiple or unusually
located fissures should prompt a search for an underlying cause such as
inflammatory bowel disease or internal prolapse.
Refractory cases where the above treatments have failed may be considered for
advancement flaps.

Fistula in ano
The most effective treatment for fistula is laying it open (fistulotomy). When the
fistula is below the sphincter and uncomplicated, this is a reasonable option.
Sphincter involvement and complex underlying disease should be assessed both
surgically and ideally with imaging (either MRI or endoanal USS). Surgery is then
usually staged, in the first instance a draining seton suture may be inserted. This
avoids the development of recurrent sepsis and may allow resolution. In patients
with Crohns disease the seton should be left in situ long term and the patient
managed medically, as in these cases attempts at complex surgical repair nearly
always fail. Fistulas not associated with IBD may be managed by advancement
flaps, instillation of plugs and glue is generally unsuccessful. A newer technique of
ligation of intersphincteric tract (LIFT procedure) is reported to have good results
in selected centres.

Next question 

Display my notes on this topic

          

Save my notes

Question stats
A 7.1%
B 6%
gathered by dr. elbarky, for free, not intended for profit by anyone elsewhere.

C 34%
D 46.6%
E 6.3%

46.6% of users answered this question correctly

Search eMRCS

Search term Go

 Open MRCS Part A textbook (../review/textbook.php)

External links

+ Suggest a link

Dashboard

10

11

12

Question 6 of 74
gathered by dr. elbarky, for free, not intended for profit by anyone elsewhere.

 

A 28 year old man is reviewed in the clinic. He has suffered from Crohns disease
for many years, he has recently undergone a sub total colectomy. However, he has
residual Crohns in his rectum and this is the cause of ongoing symptoms. Medical
therapy is proving ineffective. What is the best course of action?

Abdomino perineal excision of the colon and rectum

Proctectomy

Hartmanns procedure

Ileo-rectal anastomosis

Formation of ileo-anal pouch

An abdomino-perineal excision of the colon and rectum is a cancer procedure and


not appropriate in the context of inflammatory bowel disease. The only appropriate
surgical option here is a proctectomy to remove the rectal stump and anal canal.

Please rate this question:

 Discuss and give feedback

Next question 

IBD

Ulcerative colitis Vs Crohns

Crohn's disease Ulcerative colitis

Distribution Mouth to anus Rectum and colon

Macroscopic Cobblestone appearance, Contact bleeding


changes apthoid ulceration
Crohn's disease Ulcerative colitis

Depth of Transmural inflammation Superficial inflammation


disease
gathered by dr. elbarky, for free, not intended for profit by anyone elsewhere.

Distribution Patchy Continuous


pattern

Histological Granulomas (non caseating Crypt abscesses,


features epithelioid cell aggregates with Inflammatory cells in the
Langerhans' giant cells) lamina propria

Surgical treatment

Ulcerative colitis
In UC the main place for surgery is when medical treatment has failed, in the
emergency setting this will be a sub total colectomy, end ileostomy and a mucous
fistula. Electively it will be a pan proctocolectomy, an ileoanal pouch may be a
selected option for some. Remember that longstanding UC increases colorectal
cancer risk.

(https://d2zgo9qer4wjf4.cloudfront.net/images_eMRCS/swb042b.jpg)
Image sourced from Wikipedia
(https://d2zgo9qer4wjf4.cloudfront.net
(http://en.wikipedia.org/wiki/Ulcerative
/images_eMRCS/swb042b.jpg)
colitis)

Crohn's disease
Unlike UC Crohn's patients need to avoid surgeons, minimal resections are the rule.
They should not have ileoanal pouches as they will do poorly with them.
Management of Crohn's ano rectal sepsis is with a minimal approach, simply drain
sepsis and use setons to facilitate drainage. Definitive fistula surgery should be
avoided.
gathered by dr. elbarky, for free, not intended for profit by anyone elsewhere.

(https://d2zgo9qer4wjf4.cloudfront.net/images_eMRCS/swb043b.jpg)
Image sourced from Wikipedia (https://d2zgo9qer4wjf4.cloudfront.net
(http://en.wikipedia.org/wiki/Crohn) /images_eMRCS/swb043b.jpg)

Next question 

Display my notes on this topic

          

Save my notes

Question stats

A 20.4%
B 44.5%
C 11.5%

Question 7 of 74
gathered by dr. elbarky, for free, not intended for profit by anyone elsewhere.

 

A 33 year old lady is admitted with recurrent discharging fistula in ano. She is also
known to have ano rectal Crohns disease. On examination, she is found to have a
low anal fistula with involvement of a very small amount of the external anal
sphincter muscle. What is the most appropriate course of action?

Insertion of a loose seton

Fistulotomy

Core fistulectomy

Core fistulectomy and advancement flap

Insertion of a cutting seton

Fistula in ano in patients with Crohns disease should be managed with


insertion of seton.

In patients with IBD, management of fistula should be minimalistic and complex


procedures best avoided. Laying open fistulas in this situation is likely to result in a
chronic and non healing wound.

Please rate this question:

 Discuss and give feedback

Next question 

Anal fistula

Fistula in ano is the most common form of ano rectal sepsis. Fistulae will have
both an internal opening and external opening, these will be connected by tract(s).
Complexity arises because of the potential for multiple entry and exit sites,
together with multiple tracts. Fistulae are classified into four main groups
according to anatomical location and the degree of sphincter involvement. Simple
uncomplicated fistulae are low and do not involve more than 30% of the external
sphincter. Complex fistulae involve the sphincter, have multiple branches or are
non cryptoglandular in origin[1]
gathered by dr. elbarky, for free, not intended for profit by anyone elsewhere.

Assessment
Examination of the perineum for signs of trauma, external openings or the
stigmata of IBD is important. Digital rectal examination may reveal the cord linking
the internal and external openings. At the same time the integrity of the sphincter
mechanism can be assessed. Low, uncomplicated fistulas may not require any
further assessment, other groups will usually require more detailed investigation.
For the fistula, the use of endo-anal USS with instillation of hydrogen peroxide into
the fistula tract may be helpful. Ano-rectal MRI scanning is also a useful tool, it is
sensitive and specific for the identification of fistula anatomy, branching tracts and
identifying occult sphincter involvement[2].

Identification of the internal opening


Fistulas with an external opening less than 3cm from the anal verge will typically
obey Goodsalls rule (see below).

(https://d2zgo9qer4wjf4.cloudfront.net/images_eMRCS/swb142b.png)
Image sourced from Wikipedia (https://d2zgo9qer4wjf4.cloudfront.net
(http://en.wikipedia.org/wiki/Goodsall) /images_eMRCS/swb142b.png)
Therapies
Seton suture
A seton is a piece of material that is passed through the fistula between the
internal and external openings that allows the drainage of sepsis. This is important
as undrained septic foci may drain along the path of least resistance, which may
result in the development of accessory tracts and openings. Their main use is in
treating complex fistula. Two types of seton are recognised, simple and cutting.
Simple setons lie within the fistula tract and encourage both drainage and fibrosis.
A cutting seton is inserted and the skin incised. The suture is tightened and re-
tightened at regular intervals. This may convert a high fistula to a low fistula. Since
the tissue will scar surrounding the fistula it is hoped that this technique will
minimise incontinence[3]. Unfortunately, a large retrospective review of the
literature related to the use of cutting setons has found that they are associated
with a 12% long term incontinence rate [4]

Fistulotomy
gathered by dr. elbarky, for free, not intended for profit by anyone elsewhere.

Low fistulas, that are simple should be treated by fistulotomy once the acute
sepsis has been controlled. Fistulotomy (where safe) provides the highest healing
rates [5]. Because fistulotomy is regarded as having a high cure rate, there are
some who prefer to use this technique with more extensive sphincter involvement.
In these patients the fistulotomy is performed as for a low fistula. However, the
muscle that is encountered is then divided and reconstructed with an overlapping
sphincter repair. A price is paid in terms of incontinence with this technique and up
to 12.5% of patients who were continent pre-operatively will have issues relating to
continence post procedure[6]. The same group also randomised between
fistulotomy and sphincter reconstruction and ano-rectal advancement flaps for the
treatment of complex cryptoglandular fistulas and reported similar outcomes in
terms of recurrence (>90%) and disturbances to continence (20%)[7].
Other authors have found adverse outcomes following fistulotomy in patients who
have undergone previous surgery, are of female gender or who have high internal
openings [8], in these patients careful assessment of pre-operative sphincter
function should be considered mandatory prior to fistulotomy.

Anal fistula plugs and fibrin glue


The desire to avoid injury to the sphincter complex has led to surgeons using both
fibrin glue and plugs to try and improve fistula healing. Meticulous preparation of
the tract and prior use of a draining seton is likely to improve chances of success.
The use of anal fistula plugs in high transphincteric fistula of cryptoglandular
origin is to be discouraged because of the high incidence of non response in
patients treated with such devices [9]In most patients septic complications are the
reasons for failure [10]. Fibrin glue is a popular option for the treatment of fistula.
There is variability of reported healing rates In some cases initial success rates of
up to 50% healing at six months are reported (in patients with complex cryptogenic
fistula). Of these successes 25% suffer a long term recurrence of fistula [11].
There are, however, no obvious cases of damage to the sphincter complex and the
use of the devices does not appear to adversely impact on subsequent surgical
options.

Ano-rectal advancement flaps


This procedure is primarily directed at high fistulae, and is considered attractive as
a sphincter saving operation. The procedure is performed either with the patient in
the prone jack knife position or in lithotomy (depending upon the site of the
fistula). The dissection is commenced in the sub mucosal plane (which may be
infiltrated with dilute adrenaline solution to ease dissection). The dissection is
continued into healthy proximal tissue. This is brought down and sutured over the
defect.
Follow up of patients with cryptoglandular fistulas treated with advancement flaps
shows a success in up to 80% patients[12-14]. With most recurrences occurring in
the first 6 months following surgery[12]. Continence was affected in some
patients, with up to 10% describing major continence issues post operatively.
Ligation of the intersphincteric tract procedure
In this procedure an incision is made in the intersphincteric groove and the fistula
tract dissected out in this plane and divided. A greater than 90% cure rate within 4
gathered by dr. elbarky, for free, not intended for profit by anyone elsewhere.

weeks was initially reported[15]. Others have subsequently performed similar


studies on larger numbers of patients with similar success rates.

Fistulotomy at the time of abscess drainage?


A Cochrane review conducted in 2010 suggests that primary fistulotomy for low,
uncomplicated fistula in ano may be safe and associated with better outcomes in
relation to long term chronic sepsis[16]. However, there is a danger that such
surgery performed by non specialists may result in a higher complication rate and
therefore the traditional teaching is that primary treatment of acute sepsis is
incision and drainage only. All agree that high/ complex fistulae should never be
subject to primary fistulotomy in the acute setting.

References
1. Parks, A.G., P.H. Gordon, and J.D. Hardcastle, A classification of fistula-in-ano. Br
J Surg, 1976. 63(1): p. 1-12.
2. Lunniss, P.J., et al., Magnetic resonance imaging of fistula-in-ano. Dis Colon
Rectum, 1994. 37(7): p. 708-18.
3. Misra, M.C. and B.M. Kapur, A new non-operative approach to fistula in ano. Br J
Surg, 1988. 75(11): p. 1093-4.
4. Ritchie, R.D., J.M. Sackier, and J.P. Hodde, Incontinence rates after cutting seton
treatment for anal fistula. Colorectal Dis, 2009. 11(6): p. 564-71.
5. Tyler, K.M., C.B. Aarons, and S.M. Sentovich, Successful sphincter-sparing
surgery for all anal fistulas. Dis Colon Rectum, 2007. 50(10): p. 1535-9.
6. Perez, F., et al., Prospective clinical and manometric study of fistulotomy with
primary sphincter reconstruction in the management of recurrent complex fistula-
in-ano. Int J Colorectal Dis, 2006. 21(6): p. 522-6.
7. Perez, F., et al., Randomized clinical and manometric study of advancement flap
versus fistulotomy with sphincter reconstruction in the management of complex
fistula-in-ano. Am J Surg, 2006. 192(1): p. 34-40.
8. Garcia-Aguilar, J., et al., Anal fistula surgery. Factors associated with recurrence
and incontinence. Dis Colon Rectum, 1996. 39(7): p. 723-9.
9. Ortiz, H., et al., Randomized clinical trial of anal fistula plug versus endorectal
advancement flap for the treatment of high cryptoglandular fistula in ano. Br J
Surg, 2009. 96(6): p. 608-12.
10. El-Gazzaz, G., M. Zutshi, and T. Hull, A retrospective review of chronic anal
fistulae treated by anal fistulae plug. Colorectal Dis, 2010. 12(5): p. 442-7.
11. Haim, N., et al., Long-term results of fibrin glue treatment for cryptogenic
perianal fistulas: a multicenter study. Dis Colon Rectum, 2011. 54(10): p. 1279-83.
12. Ortiz, H., et al., Length of follow-up after fistulotomy and fistulectomy
associated with endorectal advancement flap repair for fistula in ano. Br J Surg,
2008. 95(4): p. 484-7.
13. Kodner, I.J., et al., Endorectal advancement flap repair of rectovaginal and other
complicated anorectal fistulas. Surgery, 1993. 114(4): p. 682-9; discussion 689-90.
14. Abbas, M.A., R. Lemus-Rangel, and A. Hamadani, Long-term outcome of
endorectal advancement flap for complex anorectal fistulae. Am Surg, 2008.
74(10): p. 921-4.
15. Rojanasakul, A., et al., Total anal sphincter saving technique for fistula-in-ano;
the ligation of intersphincteric fistula tract. J Med Assoc Thai, 2007. 90(3): p.
gathered by dr. elbarky, for free, not intended for profit by anyone elsewhere.

581-6.
16. Malik, A.I., R.L. Nelson, and S. Tou, Incision and drainage of perianal abscess
with or without treatment of anal fistula. Cochrane Database Syst Rev, 2010(7): p.
CD006827.

Next question 

Display my notes on this topic

          

Save my notes

Question stats

A 44.8%
B 22.1%
C 8.4%
D 9%
E 15.7%

44.8% of users answered this question correctly

Search eMRCS

Search term Go

 Open MRCS Part A textbook (../review/textbook.php)


Question 8 of 74
gathered by dr. elbarky, for free, not intended for profit by anyone elsewhere.

 

What is the most likely explanation for a 63 year old male to complain of a painless
blood stained mucous rectal discharge 6 months following a Hartmann's
procedure?

Pelvic abscess

Crohns disease

Dysplasia of the rectal stump

Diversion proctitis

Fissure in ano

Once the bowel has been disconnected, a degree of inflammation is commonly


seen in the quiescent bowel. This is typically referred to as diversion colitis.
Dysplasia is not usually seen in this context as a Hartmanns procedure is not
usually a treatment modality used for IBD (which is the main risk factor for
dysplasia).

Please rate this question:

 Discuss and give feedback

Next question 

Rectal bleeding

Rectal bleeding is a common cause for patients to be referred to the surgical clinic.
In the clinical history it is useful to try and localise the anatomical source of the
blood. Bright red blood is usually of rectal anal canal origin, whilst dark red blood is
more suggestive of a proximally sited bleeding source. Blood which has entered
the GI tract from a gastro-duodenal source will typically resemble malaena due to
the effects of the digestive enzymes on the blood itself.

In the table below we give some typical bleeding scenarios together with physical
examination findings and causation.
Cause Type of Features in history Examination findings
bleeding
gathered by dr. elbarky, for free, not intended for profit by anyone elsewhere.

Fissure in Bright red Painful bleeding that Muco-epithelial defect


ano rectal occurs post defecation usually in the midline
bleeding in small volumes. posteriorly (anterior
Usually antecedent fissures more likely to
features of be due to underlying
constipation disease)

Haemorroids Bright red Post defecation Normal colon and


rectal bleeding noted both on rectum. Proctoscopy
bleeding toilet paper and drips may show internal
into pan. May be haemorrhoids. Internal
alteration of bowel haemorrhoids are
habit and history of usually impalpable.
straining. No blood
mixed with stool. No
local pain.

Crohns Bright red Bleeding that is Perineal inspection may


disease or mixed accompanied by other show fissures or
blood symptoms such as fistulae. Proctoscopy
altered bowel habit, may demonstrate
malaise, history of indurated mucosa and
fissures (especially possibly strictures. Skip
anterior) and lesions may be noted at
abscesses. colonoscopy.

Ulcerative Bright red Diarrhoea, weight loss, Proctitis is the most


colitis bleeding nocturnal marked finding. Peri
often incontinence, passage anal disease is usually
mixed of mucous PR. absent. Colonoscopy
with stool will show continuous
mucosal lesion.

Rectal Bright red Alteration of bowel Usually obvious


cancer blood habit. Tenesmus may mucosal abnormality.
mixed be present. Symptoms Lesion may be fixed or
volumes of metastatic disease. mobile depending upon
disease extent.
Surrounding mucosa
often normal, although
polyps may be present.
Image showing a fissure in ano. Typically these are located posteriorly and in the
midline. Fissures at other sites may be associated with underlying disease.
gathered by dr. elbarky, for free, not intended for profit by anyone elsewhere.

(https://d2zgo9qer4wjf4.cloudfront.net/images_eMRCS/swb054b.jpg)
Image sourced from Wikipedia
(https://d2zgo9qer4wjf4.cloudfront.net
(http://en.wikipedia.org/wiki/Anal
/images_eMRCS/swb054b.jpg)
fissure)

Colonoscopic image of internal haemorroids. Note these may often be impalpable.

(https://d2zgo9qer4wjf4.cloudfront.net/images_eMRCS/swb055b.jpg)
Image sourced from Wikipedia
(https://d2zgo9qer4wjf4.cloudfront.net
(http://en.wikipedia.org
/images_eMRCS/swb055b.jpg)
/wiki/Haemorrhoids)
Investigation
All patients presenting with rectal bleeding require digital rectal examination
and procto-sigmoidoscopy as a minimal baseline.
Remember that haemorrhoids are typically impalpable and to attribute
gathered by dr. elbarky, for free, not intended for profit by anyone elsewhere.

bleeding to these in the absence of accurate internal inspection is


unsatisfactory.
In young patients with no other concerning features in the history a carefully
performed sigmoidoscopy that demonstrates clear haemorrhoidal disease
may be sufficient. If clear views cannot be obtained then patients require
bowel preparation with an enema and a flexible sigmoidscopy performed.
In those presenting with features of altered bowel habit or suspicion of
inflammatory bowel disease a colonoscopy is the best test.
Patients with excessive pain who are suspected of having a fissure may
require an examination under general or local anaesthesia.
In young patients with external stigmata of fissure and a compatible history
it is acceptable to treat medically and defer internal examination until the
fissure is healed. If the fissure fails to heal then internal examination
becomes necessary along the lines suggested above to exclude internal
disease.

Special tests
In patients with a malignancy of the rectum the staging investigations
comprise an MRI of the rectum to identify circumferential resection margin
compromise and to identify mesorectal nodal disease. In addition to this CT
scanning of the chest abdomen and pelvis is necessary to stage for more
distant disease. Some centres will still stage the mesorectum with endo
rectal ultrasound but this is becoming far less common.

Patients with fissure in ano who are being considered for surgical
sphincterotomy and are females who have an obstetric history should
probably have ano rectal manometry testing performed together with endo
anal ultrasound. As this service is not universally available it is not
mandatory but in the absence of such information there are continence
issues that may arise following sphincterotomy.

Management

Disease Management

Fissure in ano GTN ointment 0.2% or diltiazem cream applied topically is


the usual first line treatment. Botulinum toxin for those who
fail to respond. Internal sphincterotomy for those who fail
with botox, can be considered earlier in males.
Haemorroids Lifestyle advice, for small internal haemorrhoids can
consider injection sclerotherapy or rubber band ligation. For
external haemorrhoids consider haemorrhoidectomy.
Modern options include HALO procedure and stapled
gathered by dr. elbarky, for free, not intended for profit by anyone elsewhere.

haemorrhoidectomy.

Inflammatory Medical management- although surgery may be needed for


bowel disease fistulating Crohns (setons).

Rectal cancer Anterior resection or abdomino-perineal excision of the


colon and rectum. Total mesorectal excision is now standard
of care. Most resections below the peritoneal reflection will
require defunctioning ileostomy. Most patients will require
preoperative radiotherapy.

Next question 

Display my notes on this topic

          

Save my notes

Question stats

A 6.9%
B 9.1%
C 30.7%
D 47.4%
E 5.9%

47.4% of users answered this question correctly

Search eMRCS

Search term Go

Question 9 of 74
gathered by dr. elbarky, for free, not intended for profit by anyone elsewhere.

 

A 78 year old lady is admitted with a 3 hour history of passage of dark red blood
per rectum. Prior to this event, she was otherwise well with no major medical co-
morbidities. On examination, she has a mild tachycardia but other vital signs are
normal, abdomen is soft and non tender. Digital rectal exam reveals dark blood but
no other findings. What is the most likely underlying cause?

Diverticular disease

Meckels diverticulum

Jejunal diverticulosis

Angiodysplasia of the colon

Colonic cancer

Diverticular disease is the commonest cause of lower GI bleeding in adults.

Please rate this question:

 Discuss and give feedback

Next question 

Lower Gastrointestinal bleeding

Colonic bleeding
This typically presents as bright red or dark red blood per rectum. Colonic bleeding
rarely presents as malaena type stool, this is because blood in the colon has a
powerful laxative effect and is rarely retained long enough for transformation to
occur and because the digestive enzymes present in the small bowel are not
present in the colon. Up to 15% of patients presenting with haemochezia will have
an upper gastrointestinal source of haemorrhage.
As a general rule right sided bleeds tend to present with darker coloured blood
than left sided bleeds. Haemorrhoidal bleeding typically presents as bright red
rectal bleeding that occurs post defecation either onto toilet paper or into the toilet
pan. It is very unusual for haemorrhoids alone to cause any degree of
gathered by dr. elbarky, for free, not intended for profit by anyone elsewhere.

haemodynamic compromise.

Causes

Cause Presenting features

Colitis Bleeding may be brisk in advanced cases, diarrhoea is


commonly present. Abdominal x-ray may show featureless
colon.

Diverticular Acute diverticulitis often is not complicated by major


disease bleeding and diverticular bleeds often occur sporadically.
75% all will cease spontaneously within 24-48 hours.
Bleeding is often dark and of large volume.

Cancer Colonic cancers often bleed and for many patients this may
be the first sign of the disease. Major bleeding from early
lesions is uncommon

Haemorrhoidal Typically bright red bleeding occurring post defecation.


bleeding Although patients may give graphic descriptions bleeding
of sufficient volume to cause haemodynamic compromise
is rare.

Angiodysplasia Apart from bleeding, which may be massive, these


arteriovenous lesions cause little in the way of symptoms.
The right side of the colon is more commonly affected.

Management
Prompt correction of any haemodynamic compromise is required. Unlike
upper gastrointestinal bleeding the first line management is usually
supportive. This is because in the acute setting endoscopy is rarely helpful.
When haemorrhoidal bleeding is suspected a proctosigmoidoscopy is
reasonable as attempts at full colonoscopy are usually time consuming and
often futile.
In the unstable patient the usual procedure would be an angiogram (either
CT or percutaneous), when these are performed during a period of
haemodynamic instability they may show a bleeding point and may be the
only way of identifying a patch of angiodysplasia.
In others who are more stable the standard procedure would be a
colonoscopy in the elective setting. In patients undergoing angiography
attempts can be made to address the lesion in question such as coiling.
Otherwise surgery will be necessary.
In patients with ulcerative colitis who have significant haemorrhage the
standard approach would be a sub total colectomy, particularly if medical
management has already been tried and is not effective.
gathered by dr. elbarky, for free, not intended for profit by anyone elsewhere.

Indications for surgery


Patients > 60 years
Continued bleeding despite endoscopic intervention
Recurrent bleeding
Known cardiovascular disease with poor response to hypotension

Surgery
Selective mesenteric embolisation if life threatening bleeding. This is most helpful
if conducted during a period of relative haemodynamic instability. If all
haemodynamic parameters are normal then the bleeding is most likely to have
stopped and any angiography normal in appearance. In many units a CT angiogram
will replace selective angiography but the same caveats will apply.

If the source of colonic bleeding is unclear; perform a laparotomy, on table colonic


lavage and following this attempt a resection. A blind sub total colectomy is most
unwise, for example bleeding from an small bowel arterio-venous malformation
will not be treated by this manoeuvre.

Summary of Acute Lower GI bleeding recommendations


Consider admission if:
* Over 60 years
* Haemodynamically unstable/profuse PR bleeding
* On aspirin or NSAID
* Significant co morbidity

Management
All patients should have a history and examination, PR and proctoscopy
Colonoscopic haemostasis aimed for in post polypectomy or diverticular
bleeding

References
http://www.sign.ac.uk/guidelines/fulltext/105/index.html

Next question 

Display my notes on this topic

          
Save my notes
gathered by dr. elbarky, for free, not intended for profit by anyone elsewhere.

Question stats

A 42.4%
B 9.2%
C 9.3%
D 27.9%
E 11.2%

42.4% of users answered this question correctly

Search eMRCS

Search term Go

 Open MRCS Part A textbook (../review/textbook.php)

External links

+ Suggest a link

Dashboard

8

Question 10 of 74
gathered by dr. elbarky, for free, not intended for profit by anyone elsewhere.

 

A 55 year old man presents with tenesmus and rectal bleeding. On examination he
has a large bulky rectal cancer at 5cm from the anal verge with tethering to the
prostate gland. Imaging shows no distant disease. What is the most appropriate
initial treatment modality?

Abdomino-perineral resection of the colon and rectum

Pelvic exenteration

Abdomino-perineal excision of the colon and rectum with prostatectomy

Long course chemoradiotherapy

Short course radiotherapy

Rectal cancers with threatened resection margins are managed with


radiotherapy and chemotherapy initially. This is not the case with colonic
cancers which are usually primarily resected.

T4 rectal cancers are managed with long course chemoradiotherapy. A dramatic


response is not uncommon. To embark on attempted resection at this stage is to
court failure.

Please rate this question:

 Discuss and give feedback

Next question 

Colorectal cancer treatment

Patients diagnosed as having colorectal cancer should be completely staged using


CT of the chest/ abdomen and pelvis. Their entire colon should have been
evaluated with colonoscopy or CT colonography. Patients whose tumours lie below
the peritoneal reflection should have their mesorectum evaluated with MRI.
Once their staging is complete patients should be discussed within a dedicated
colorectal MDT meeting and a treatment plan formulated.
gathered by dr. elbarky, for free, not intended for profit by anyone elsewhere.

Treatment of colonic cancer


Cancer of the colon is nearly always treated with surgery. Stents, surgical bypass
and diversion stomas may all be used as palliative adjuncts. Resectional surgery is
the only option for cure in patients with colon cancer. The procedure is tailored to
the patient and the tumour location. The lymphatic drainage of the colon follows
the arterial supply and therefore most resections are tailored around the resection
of particular lymphatic chains (e.g. ileo-colic pedicle for right sided tumours).
Some patients may have confounding factors that will govern the choice of
procedure, for example a tumour in a patient from a HNPCC family may be better
served with a panproctocolectomy rather than segmental resection. Following
resection the decision has to be made regarding restoration of continuity. For an
anastomosis to heal the key technical factors include; adequate blood supply,
mucosal apposition and no tissue tension. Surrounding sepsis, unstable patients
and inexperienced surgeons may compromise these key principles and in such
circumstances it may be safer to construct an end stoma rather than attempting
an anastomosis.
When a colonic cancer presents with an obstructing lesion; the options are to
either stent it or resect. In modern practice it is unusual to simply defunction a
colonic tumour with a proximal loop stoma. This differs from the situation in the
rectum (see below).
Following resection patients with risk factors for disease recurrence are usually
offered chemotherapy, a combination of 5FU and oxaliplatin is common.

Rectal cancer
The management of rectal cancer is slightly different to that of colonic cancer.
This reflects the rectum's anatomical location and the challenges posed as a
result. Tumours located in the rectum can be surgically resected with either an
anterior resection or an abdomino - perineal resection. The technical aspects
governing the choice between these two procedures can be complex to appreciate
and the main point to appreciate for the MRCS is that involvement of the sphincter
complex or very low tumours require APER. In the rectum a 2cm distal clearance
margin is required and this may also impact on the procedure chosen. In addition
to excision of the rectal tube an integral part of the procedure is a meticulous
dissection of the mesorectal fat and lymph nodes (total mesorectal excision/
TME). In rectal cancer surgery invovlement of the cirumferential resection margin
carries a high risk of disease recurrence. Because the rectum is an extraperitoneal
structure (until you remove it that is!) it is possible to irradiate it, something which
cannot be offered for colonic tumours. This has a major impact in rectal cancer
treatment and many patients will be offered neoadjuvent radiotherapy (both long
and short course) prior to resectional surgery. Patients with T1, 2 and 3 /N0
disease on imaging do not require irradiation and should proceed straight to
surgery. Patients with T4 disease will typically have long course chemo
radiotherapy. Patients presenting with large bowel obstruction from rectal cancer
should not undergo resectional surgery without staging as primary treatment (very
different from colonic cancer). This is because rectal surgery is more technically
demanding, the anastomotic leak rate is higher and the danger of a positive
resection margin in an unstaged patient is high. Therefore patients with
obstructing rectal cancer should have a defunctioning loop colostomy.
gathered by dr. elbarky, for free, not intended for profit by anyone elsewhere.

Summary of procedures
The operations for cancer are segmental resections based on blood supply and
lymphatic drainage. These commonly performed procedures are core knowledge
for the MRCS and should be understood.

Site of Risk of
cancer Type of resection Anastomosis leak

Right colon Right hemicolectomy Ileo-colic Low


<5%

Transverse Extended right hemicolectomy Ileo-colic Low


<5%

Splenic Extended right hemicolectomy Ileo-colic Low


flexure <5%

Splenic Left hemicolectomy Colo-colon 2-5%


flexure

Left colon Left hemicolectomy Colo-colon 2-5%

Sigmoid High anterior resection Colo-rectal 5%


colon

Upper Anterior resection (TME) Colo-rectal 5%


rectum

Low rectum Anterior resection (Low TME) Colo-rectal 10%


(+/- Defunctioning
stoma)

Anal verge Abdomino-perineal excision of None n/a


colon and rectum

In the emergency setting, where the bowel has perforated, the risk of an
anastomotic breakdown is much greater, particularly when the anastomosis is
colon-colon. In this situation, an end colostomy is often safer and can be reversed
later. When resection of the sigmoid colon is performed and an end colostomy is
fashioned the operation is referred to as a Hartmans procedure. Whilst left sided
resections are more risky, ileo-colic anastomoses are relatively safe even in the
emergency setting and do not need to be defunctioned.
References
A review of the diagnosis and management of colorectal cancer and a summary of
the UK National Institute of Clinical Excellence guidelines is provided in:
gathered by dr. elbarky, for free, not intended for profit by anyone elsewhere.

Poston G, et al . Diagnosis and management of colorectal cancer: summary of


NICE guidance. BMJ 2011: 343: d 6751.

Next question 

Display my notes on this topic

          

Save my notes

Question stats

A 12%
B 7.9%
C 30.6%
D 36.1%
E 13.5%

36.1% of users answered this question correctly

Search eMRCS

Search term Go

 Open MRCS Part A textbook (../review/textbook.php)

External links

+ Suggest a link

Question 11 of 74
gathered by dr. elbarky, for free, not intended for profit by anyone elsewhere.

 

A 55 year old man is found to have an anal cancer. His staging investigations show
no metastatic disease. What is the most appropriate treatment?

Radical abdominoperineal excision of the anus and rectum

Radical chemoradiotherapy

Excision proctectomy

External beam irradiation alone

Chemotherapy alone

Combined chemoradiotherapy is the standard treatment for anal cancer

Please rate this question:

 Discuss and give feedback

Next question 

Anal cancer

Cancers arising from the squamous epithelium of the anal canal


Arise inferior to the dentate line
Strongly linked to HPV type 16 infection
Other risk factors include ano-receptive intercourse, smoking and
immunosuppression
Presenting symptoms include anal discomfort, discharge or pruritus
Lymphatic spread typically occurs to the inguinal nodes
Diagnosis is made by EUA and biopsies
Staging is with CT scanning of the chest, abdomen and pelvis
First line treatment is typically with chemoradiotherapy
Second line treatment for non metastatic disease is with salvage radical
abdominoperineal excision of the anus and rectum

Next question 
gathered by dr. elbarky, for free, not intended for profit by anyone elsewhere.

Display my notes on this topic

          

Save my notes

Question stats

A 35.5%
B 35.7%
C 11.5%
D 11%
E 6.3%

35.7% of users answered this question correctly

Search eMRCS

Search term Go

 Open MRCS Part A textbook (../review/textbook.php)

External links

+ Suggest a link

Dashboard

1

Question 12 of 74
gathered by dr. elbarky, for free, not intended for profit by anyone elsewhere.

 

A 66 year old man is admitted as an emergency with torrential rectal bleeding.


Following resuscitation, an upper GI endoscopy is undertaken and it is normal.
However, he continues to bleed. What is the most appropriate course of action?

Arrange a CT angiogram

Arrange a laparotomy and sub total colectomy

Undertake a colonoscopy

Perform a laparoscopy and on table colonoscopy

Arrange a capsule endoscopy

Heavy lower GI bleeding should be investigated with an angiogram. It is best to


avoid emergency surgery as the bleeding site is very difficult to find. Unlike upper
GI endoscopy, colonoscopy in patients who are acutely bleeding is often difficult
and seldom helpful. If it is to be done, a therapeutic scope with twin working
channels is useful.

Please rate this question:

 Discuss and give feedback

Next question 

Lower Gastrointestinal bleeding

Colonic bleeding
This typically presents as bright red or dark red blood per rectum. Colonic bleeding
rarely presents as malaena type stool, this is because blood in the colon has a
powerful laxative effect and is rarely retained long enough for transformation to
occur and because the digestive enzymes present in the small bowel are not
present in the colon. Up to 15% of patients presenting with haemochezia will have
an upper gastrointestinal source of haemorrhage.

As a general rule right sided bleeds tend to present with darker coloured blood
than left sided bleeds. Haemorrhoidal bleeding typically presents as bright red
rectal bleeding that occurs post defecation either onto toilet paper or into the toilet
pan. It is very unusual for haemorrhoids alone to cause any degree of
haemodynamic compromise.
gathered by dr. elbarky, for free, not intended for profit by anyone elsewhere.

Causes

Cause Presenting features

Colitis Bleeding may be brisk in advanced cases, diarrhoea is


commonly present. Abdominal x-ray may show featureless
colon.

Diverticular Acute diverticulitis often is not complicated by major


disease bleeding and diverticular bleeds often occur sporadically.
75% all will cease spontaneously within 24-48 hours.
Bleeding is often dark and of large volume.

Cancer Colonic cancers often bleed and for many patients this may
be the first sign of the disease. Major bleeding from early
lesions is uncommon

Haemorrhoidal Typically bright red bleeding occurring post defecation.


bleeding Although patients may give graphic descriptions bleeding
of sufficient volume to cause haemodynamic compromise
is rare.

Angiodysplasia Apart from bleeding, which may be massive, these


arteriovenous lesions cause little in the way of symptoms.
The right side of the colon is more commonly affected.

Management
Prompt correction of any haemodynamic compromise is required. Unlike
upper gastrointestinal bleeding the first line management is usually
supportive. This is because in the acute setting endoscopy is rarely helpful.
When haemorrhoidal bleeding is suspected a proctosigmoidoscopy is
reasonable as attempts at full colonoscopy are usually time consuming and
often futile.
In the unstable patient the usual procedure would be an angiogram (either
CT or percutaneous), when these are performed during a period of
haemodynamic instability they may show a bleeding point and may be the
only way of identifying a patch of angiodysplasia.
In others who are more stable the standard procedure would be a
colonoscopy in the elective setting. In patients undergoing angiography
attempts can be made to address the lesion in question such as coiling.
Otherwise surgery will be necessary.
In patients with ulcerative colitis who have significant haemorrhage the
standard approach would be a sub total colectomy, particularly if medical
management has already been tried and is not effective.

Indications for surgery


gathered by dr. elbarky, for free, not intended for profit by anyone elsewhere.

Patients > 60 years


Continued bleeding despite endoscopic intervention
Recurrent bleeding
Known cardiovascular disease with poor response to hypotension

Surgery
Selective mesenteric embolisation if life threatening bleeding. This is most helpful
if conducted during a period of relative haemodynamic instability. If all
haemodynamic parameters are normal then the bleeding is most likely to have
stopped and any angiography normal in appearance. In many units a CT angiogram
will replace selective angiography but the same caveats will apply.

If the source of colonic bleeding is unclear; perform a laparotomy, on table colonic


lavage and following this attempt a resection. A blind sub total colectomy is most
unwise, for example bleeding from an small bowel arterio-venous malformation
will not be treated by this manoeuvre.

Summary of Acute Lower GI bleeding recommendations


Consider admission if:
* Over 60 years
* Haemodynamically unstable/profuse PR bleeding
* On aspirin or NSAID
* Significant co morbidity

Management
All patients should have a history and examination, PR and proctoscopy
Colonoscopic haemostasis aimed for in post polypectomy or diverticular
bleeding

References
http://www.sign.ac.uk/guidelines/fulltext/105/index.html

Next question 

Display my notes on this topic

          
Save my notes

Question stats
gathered by dr. elbarky, for free, not intended for profit by anyone elsewhere.

A 48.9%
B 12.7%
C 20%
D 12.2%
E 6.1%

48.9% of users answered this question correctly

Search eMRCS

Search term Go

 Open MRCS Part A textbook (../review/textbook.php)

External links

+ Suggest a link

Dashboard

9

Question 13 of 74
gathered by dr. elbarky, for free, not intended for profit by anyone elsewhere.

 

A 73 year old lady presents with large bowel obstruction. On examination, she has
a rectal cancer 6cm from the anal verge which has occluded the colonic lumen. An
abdominal x-ray shows a caecal diameter of 7cm. Which of the management
strategies outlined below is the most appropriate?

Construction of a loop ileostomy

Construction of a loop colostomy

Construction of a venting caecostomy

Abdomino-perineal resection of the colon and rectum

Low anterior resection and loop ileostomy

Bowel obstruction due to RECTAL cancer should be treated by loop


colostomy.
Bowel obstruction due to obstructing left sided COLON cancer is usually
treated by resection of the primary lesion and formation of colostomy.

This patient should be defunctioned, definitive surgery should wait until staging is
completed. A loop ileostomy will not satisfactorily decompress an acutely
obstructed colon. Low rectal cancers that are obstructed should not usually be
primarily resected. The obstructed colon that would be used for anastomosis
would carry a high risk of anastomotic dehiscence. In addition, as this is an
emergency presentation, staging may not be completed, an attempted resection
may therefore compromise the circumferential resection margin, with an
associated risk of local recurrence.

Please rate this question:

 Discuss and give feedback

Next question 
Colorectal cancer treatment

Patients diagnosed as having colorectal cancer should be completely staged using


CT of the chest/ abdomen and pelvis. Their entire colon should have been
gathered by dr. elbarky, for free, not intended for profit by anyone elsewhere.

evaluated with colonoscopy or CT colonography. Patients whose tumours lie below


the peritoneal reflection should have their mesorectum evaluated with MRI.

Once their staging is complete patients should be discussed within a dedicated


colorectal MDT meeting and a treatment plan formulated.

Treatment of colonic cancer


Cancer of the colon is nearly always treated with surgery. Stents, surgical bypass
and diversion stomas may all be used as palliative adjuncts. Resectional surgery is
the only option for cure in patients with colon cancer. The procedure is tailored to
the patient and the tumour location. The lymphatic drainage of the colon follows
the arterial supply and therefore most resections are tailored around the resection
of particular lymphatic chains (e.g. ileo-colic pedicle for right sided tumours).
Some patients may have confounding factors that will govern the choice of
procedure, for example a tumour in a patient from a HNPCC family may be better
served with a panproctocolectomy rather than segmental resection. Following
resection the decision has to be made regarding restoration of continuity. For an
anastomosis to heal the key technical factors include; adequate blood supply,
mucosal apposition and no tissue tension. Surrounding sepsis, unstable patients
and inexperienced surgeons may compromise these key principles and in such
circumstances it may be safer to construct an end stoma rather than attempting
an anastomosis.
When a colonic cancer presents with an obstructing lesion; the options are to
either stent it or resect. In modern practice it is unusual to simply defunction a
colonic tumour with a proximal loop stoma. This differs from the situation in the
rectum (see below).
Following resection patients with risk factors for disease recurrence are usually
offered chemotherapy, a combination of 5FU and oxaliplatin is common.

Rectal cancer
The management of rectal cancer is slightly different to that of colonic cancer.
This reflects the rectum's anatomical location and the challenges posed as a
result. Tumours located in the rectum can be surgically resected with either an
anterior resection or an abdomino - perineal resection. The technical aspects
governing the choice between these two procedures can be complex to appreciate
and the main point to appreciate for the MRCS is that involvement of the sphincter
complex or very low tumours require APER. In the rectum a 2cm distal clearance
margin is required and this may also impact on the procedure chosen. In addition
to excision of the rectal tube an integral part of the procedure is a meticulous
dissection of the mesorectal fat and lymph nodes (total mesorectal excision/
TME). In rectal cancer surgery invovlement of the cirumferential resection margin
carries a high risk of disease recurrence. Because the rectum is an extraperitoneal
structure (until you remove it that is!) it is possible to irradiate it, something which
cannot be offered for colonic tumours. This has a major impact in rectal cancer
treatment and many patients will be offered neoadjuvent radiotherapy (both long
and short course) prior to resectional surgery. Patients with T1, 2 and 3 /N0
disease on imaging do not require irradiation and should proceed straight to
surgery. Patients with T4 disease will typically have long course chemo
gathered by dr. elbarky, for free, not intended for profit by anyone elsewhere.

radiotherapy. Patients presenting with large bowel obstruction from rectal cancer
should not undergo resectional surgery without staging as primary treatment (very
different from colonic cancer). This is because rectal surgery is more technically
demanding, the anastomotic leak rate is higher and the danger of a positive
resection margin in an unstaged patient is high. Therefore patients with
obstructing rectal cancer should have a defunctioning loop colostomy.

Summary of procedures
The operations for cancer are segmental resections based on blood supply and
lymphatic drainage. These commonly performed procedures are core knowledge
for the MRCS and should be understood.

Site of Risk of
cancer Type of resection Anastomosis leak

Right colon Right hemicolectomy Ileo-colic Low


<5%

Transverse Extended right hemicolectomy Ileo-colic Low


<5%

Splenic Extended right hemicolectomy Ileo-colic Low


flexure <5%

Splenic Left hemicolectomy Colo-colon 2-5%


flexure

Left colon Left hemicolectomy Colo-colon 2-5%

Sigmoid High anterior resection Colo-rectal 5%


colon

Upper Anterior resection (TME) Colo-rectal 5%


rectum

Low rectum Anterior resection (Low TME) Colo-rectal 10%


(+/- Defunctioning
stoma)

Anal verge Abdomino-perineal excision of None n/a


colon and rectum

In the emergency setting, where the bowel has perforated, the risk of an
anastomotic breakdown is much greater, particularly when the anastomosis is
colon-colon. In this situation, an end colostomy is often safer and can be reversed
later. When resection of the sigmoid colon is performed and an end colostomy is
fashioned the operation is referred to as a Hartmans procedure. Whilst left sided
gathered by dr. elbarky, for free, not intended for profit by anyone elsewhere.

resections are more risky, ileo-colic anastomoses are relatively safe even in the
emergency setting and do not need to be defunctioned.

References
A review of the diagnosis and management of colorectal cancer and a summary of
the UK National Institute of Clinical Excellence guidelines is provided in:
Poston G, et al . Diagnosis and management of colorectal cancer: summary of
NICE guidance. BMJ 2011: 343: d 6751.

Next question 

Display my notes on this topic

          

Save my notes

Question stats

A 20.2%
B 45.9%
C 7.9%
D 11.3%
E 14.8%

45.9% of users answered this question correctly

Search eMRCS

Search term Go

 Open MRCS Part A textbook (../review/textbook.php)


Question 14 of 74
gathered by dr. elbarky, for free, not intended for profit by anyone elsewhere.

 

A 56 year old lady is investigated with a colonoscopy for a change in bowel habit.
However, due to adhesions from a previous hysterectomy, she experiences pain
and requests the procedure be terminated. The endoscopist feels that he reached
the splenic flexure. What is the best course of action?

Discharge the patient

Arrange an abdominal CT scan

Arrange a CT colonoscopy

Arrange a barium enema

Arrange a gastrograffin enema

Failed colonoscopy should be managed with a CT colonoscopy in the first instance


(more accurate than barium studies). If the procedure is needed for therapeutic
intervention then GA colonoscopy may be needed.

Please rate this question:

 Discuss and give feedback

Next question 

Colonic polyps

Colonic Polyps
May occur in isolation, or greater numbers as part of the polyposis syndromes. In
FAP greater than 100 polyps are typically present. The risk of malignancy in
association with adenomas is related to size, and is the order of 10% in a 1cm
adenoma. Isolated adenomas seldom give risk of symptoms (unless large and
distal). Distally sited villous lesions may produce mucous and if very large,
electrolyte disturbances may occur.

Follow up of colonic polyps


Group Action

Colorectal cancer Colonoscopy 1 year post resection


gathered by dr. elbarky, for free, not intended for profit by anyone elsewhere.

Large non pedunculated colorectal One off scope at 3 years


polyps (LNPCP), R0 resection

Large non pedunculated colorectal Site check at 2-6 months and then a
polyps (LNPCP) R1 or non en bloc further scope at 12 months
resection

High risk findings at baseline One off surveillance at 3 years


colonoscopy

No high risk findings at baseline No colonoscopic surveillance and invite


colonoscopy participation in NHSBCSP programme
when due

High risk findings


More than 2 premalignant polyps including 1 or more advanced
colorectal polyps
OR
More than 5 pre malignant polyps

Exceptions to guidelines
If patient more than 10 years younger than lower screening age and has polyps but
no high risk findings, consider colonoscopy at 5 or 10 years.

Segmental resection or complete colectomy should be considered when:

1. Incomplete excision of malignant polyp


2. Malignant sessile polyp
3. Malignant pedunculated polyp with submucosal invasion
4. Polyps with poorly differentiated carcinoma
5. Familial polyposis coli
-Screening from teenager up to 40 years by 2 yearly sigmoidoscopy/colonoscopy
-Panproctocolectomy and Ileostomy or Restorative Panproctocolectomy.

Rectal polypoidal lesions may be amenable to trans anal endoscopic microsurgery.

References
Rutter MD et al. British Society of Gastroenterology/Association of Coloproctology
of Great Britain and Ireland/Public Health England post- polypectomy and post-
colorectal cancer resection surveillance guidelines. Gut 2019;0:123.
Next question 
gathered by dr. elbarky, for free, not intended for profit by anyone elsewhere.

Display my notes on this topic

          

Save my notes

Question stats

A 7.7%
B 20.6%
C 48.5%
D 11.1%
E 12%

48.5% of users answered this question correctly

Search eMRCS

Search term Go

 Open MRCS Part A textbook (../review/textbook.php)

External links

+ Suggest a link

Dashboard

3

Question 15 of 74
gathered by dr. elbarky, for free, not intended for profit by anyone elsewhere.

 

A 65 year old lady is admitted with large bowel obstruction. On investigation with
CT, she is found to have a tumour of the mid rectum with no evidence of
metastatic disease. What is the most appropriate course of action?

Formation of a loop colostomy

Laparotomy and Hartmanns procedure

Pan proctocolectomy and end ileostomy

Low anterior resection and covering loop ileostomy

Low anterior resection and end colostomy

Avoid emergency resections in large bowel obstruction due to rectal cancer

This patient has presented with large bowel obstruction. However, in the case of
rectal cancer, she is incompletely staged as ability to completely resect the lesion
can only be determined with MRI scanning and this information is not provided.
Even if the lesion were resectable, in the emergency setting, it is often safer to
undertake a simple procedure such as a loop colostomy and then complete
surgery at a later date. A low anterior resection and loop ileostomy in this situation
would almost certainly leak (and for the reasons outlined above, may be
incomplete).

Please rate this question:

 Discuss and give feedback

Next question 

Large bowel obstruction

Colonic obstruction remains a common surgical problem. It is most commonly due


to malignancy (60%) and diverticular disease (20%). Volvulus affecting the colon
accounts for 5% of cases. Acute colonic pseudo-obstruction remains a potential
differential diagnosis in all cases. Intussusception affecting the colon (most often
due to tumours in the adult population) remains a rare but recognised cause.
The typical patient will present with gradual onset of progressive abdominal
gathered by dr. elbarky, for free, not intended for profit by anyone elsewhere.

distension, colicky abdominal pain and either obstipation or absolute constipation.


On examination abdominal distension is present, the presence of caecal
tenderness (assuming no overt evidence of peritonitis) is a useful sign to elicit. A
digital rectal examination and rigid sigmoidoscopy should be performed.
A plain abdominal x-ray is the usual first line test and; the caecal diameter and
ileocaecal valve competency should be assessed on this film.

Imaging modalities
Debate long surrounds the use of CT versus gastrograffin enemas. The latter
investigation has always been the traditional method of determining whether a
structural lesion is indeed present. However, in the UK the use of this technique
has declined and in most units a CT scan will be offered as the first line
investigation by the majority of radiologists (and is advocated by the ACPGBI). In
most cases this will provide sufficient detail to allow operative planning, and since
malignancy accounts for most presentations may also stage the disease. In the
event that the radiologist cannot provide a clear statement of lesion site, the
surgeon should have no hesitation in requesting a contrast enema.

Surgical options
The decision as to when to operate or not is determined firstly by the patients
physiological status. Unstable patients require resuscitation prior to surgery and
admission to a critical care unit for invasive monitoring and potential inotropic
support may be needed. In patients who are otherwise stable the decision then
rests on the radiological and clinical findings. As a general rule the old adage that
the sun should not rise and set on unrelieved large bowel obstruction still holds
true. A caecal diameter of 12cm or more in the presence of complete obstruction
with a competent ileocaecal valve and caecal tenderness is a sign of impending
perforation and a relative indication for prompt surgery.

Right sided and transverse lesions


Right sided lesions producing large bowel obstruction should generally be treated
by right hemicolectomy or its extended variant if the lesion lies in the distal
transverse colon or splenic flexure. In these cases an ileocolic anastomosis may
be easily constructed and even in the emergency setting has a low risk of
anastomotic leak.

Left sided lesions


The options here lie between sub total colectomy and anastomosis, left
hemicolectomy with on table lavage and primary anastomosis, left hemicolectomy
and end colostomy formation and finally colonic stent insertion.
The usefulness of colonic stents was the subject of a Cochrane review in 2011.
The authors concluded that on the basis of the data that they reviewed, there was
no benefit from the use of colonic stents over conventional surgical resection with
a tendency to better outcomes seen in the surgical group (1). A more recently
conducted meta analysis met with the same conclusion (2). However, the recently
concluded CREST trial has suggested that self expanding metallic stents can
improve outcomes and avoids a stoma.
gathered by dr. elbarky, for free, not intended for profit by anyone elsewhere.

Rectosigmoid lesions
Lesions below the peritoneal reflection that are causing obstruction should
generally be treated with a loop colostomy. Primary resection of unstaged rectal
cancer would most likely carry a high CRM positivity rate and cannot be condoned.
Where the lesion occupies the distal sigmoid colon the usual practice would be to
perform a high anterior resection. The decision surrounding restoration of
intestinal continuity would lie with the operating surgeon.

References
1. Sagar J. Colorectal stents for the management of malignant colonic
obstructions. Cochrane Database of Systematic Reviews 2011, Issue 11. Art. No.:
CD007378. DOI: 10.1002/14651858.CD007378.pub2.
2. Cirrochi et al Safety and efficacy of endoscopic colonic stenting as a bridge to
surgery in the management of intestinal obstruction due to left colon and rectal
cancer: A systematic review and meta-analysis. Surg Oncol. 2013 Mar;22(1):14-21.

Next question 

Display my notes on this topic

          

Save my notes

Question stats

A 30.4%
B 17.3%
C 8.1%
D 25.4%
E 18.9%

30.4% of users answered this question correctly

Search eMRCS

Question 16 of 74
gathered by dr. elbarky, for free, not intended for profit by anyone elsewhere.

 

A 74 year old gentleman presents with an obstructing carcinoma of the splenic


flexure. Attempts at placement of a colonic stent have failed. Which of the
operative options listed below offers the best solution to this problem?

Sub total colectomy

Extended right hemicolectomy

Standard right hemicolectomy

Standard left hemicolectomy

Transverse colectomy

Standard right hemicolectomy involves colonic division to the right of the


middle colic vessels
Extended right hemicolectomy involves division of the middle colic vessels
and usually resection of the splenic flexure as well.

The question always causes confusion and to understand it the information needs
to be carefully read. Firstly, the tumour is definitely at the splenic flexure and the
second point is that the operation is definitely an extended right hemicolectomy. A
left hemicolectomy or even the older operation of a transverse colectomy could be
considered if the patient was not obstructed. However, when obstruction is
present, an extended right hemicolectomy (which involves an ileocolic
anastomosis) is relatively safe even in the obstructed setting.

Please rate this question:

 Discuss and give feedback

Next question 

Colorectal cancer treatment


Patients diagnosed as having colorectal cancer should be completely staged using
CT of the chest/ abdomen and pelvis. Their entire colon should have been
evaluated with colonoscopy or CT colonography. Patients whose tumours lie below
the peritoneal reflection should have their mesorectum evaluated with MRI.
gathered by dr. elbarky, for free, not intended for profit by anyone elsewhere.

Once their staging is complete patients should be discussed within a dedicated


colorectal MDT meeting and a treatment plan formulated.

Treatment of colonic cancer


Cancer of the colon is nearly always treated with surgery. Stents, surgical bypass
and diversion stomas may all be used as palliative adjuncts. Resectional surgery is
the only option for cure in patients with colon cancer. The procedure is tailored to
the patient and the tumour location. The lymphatic drainage of the colon follows
the arterial supply and therefore most resections are tailored around the resection
of particular lymphatic chains (e.g. ileo-colic pedicle for right sided tumours).
Some patients may have confounding factors that will govern the choice of
procedure, for example a tumour in a patient from a HNPCC family may be better
served with a panproctocolectomy rather than segmental resection. Following
resection the decision has to be made regarding restoration of continuity. For an
anastomosis to heal the key technical factors include; adequate blood supply,
mucosal apposition and no tissue tension. Surrounding sepsis, unstable patients
and inexperienced surgeons may compromise these key principles and in such
circumstances it may be safer to construct an end stoma rather than attempting
an anastomosis.
When a colonic cancer presents with an obstructing lesion; the options are to
either stent it or resect. In modern practice it is unusual to simply defunction a
colonic tumour with a proximal loop stoma. This differs from the situation in the
rectum (see below).
Following resection patients with risk factors for disease recurrence are usually
offered chemotherapy, a combination of 5FU and oxaliplatin is common.

Rectal cancer
The management of rectal cancer is slightly different to that of colonic cancer.
This reflects the rectum's anatomical location and the challenges posed as a
result. Tumours located in the rectum can be surgically resected with either an
anterior resection or an abdomino - perineal resection. The technical aspects
governing the choice between these two procedures can be complex to appreciate
and the main point to appreciate for the MRCS is that involvement of the sphincter
complex or very low tumours require APER. In the rectum a 2cm distal clearance
margin is required and this may also impact on the procedure chosen. In addition
to excision of the rectal tube an integral part of the procedure is a meticulous
dissection of the mesorectal fat and lymph nodes (total mesorectal excision/
TME). In rectal cancer surgery invovlement of the cirumferential resection margin
carries a high risk of disease recurrence. Because the rectum is an extraperitoneal
structure (until you remove it that is!) it is possible to irradiate it, something which
cannot be offered for colonic tumours. This has a major impact in rectal cancer
treatment and many patients will be offered neoadjuvent radiotherapy (both long
and short course) prior to resectional surgery. Patients with T1, 2 and 3 /N0
disease on imaging do not require irradiation and should proceed straight to
surgery. Patients with T4 disease will typically have long course chemo
radiotherapy. Patients presenting with large bowel obstruction from rectal cancer
should not undergo resectional surgery without staging as primary treatment (very
gathered by dr. elbarky, for free, not intended for profit by anyone elsewhere.

different from colonic cancer). This is because rectal surgery is more technically
demanding, the anastomotic leak rate is higher and the danger of a positive
resection margin in an unstaged patient is high. Therefore patients with
obstructing rectal cancer should have a defunctioning loop colostomy.

Summary of procedures
The operations for cancer are segmental resections based on blood supply and
lymphatic drainage. These commonly performed procedures are core knowledge
for the MRCS and should be understood.

Site of Risk of
cancer Type of resection Anastomosis leak

Right colon Right hemicolectomy Ileo-colic Low


<5%

Transverse Extended right hemicolectomy Ileo-colic Low


<5%

Splenic Extended right hemicolectomy Ileo-colic Low


flexure <5%

Splenic Left hemicolectomy Colo-colon 2-5%


flexure

Left colon Left hemicolectomy Colo-colon 2-5%

Sigmoid High anterior resection Colo-rectal 5%


colon

Upper Anterior resection (TME) Colo-rectal 5%


rectum

Low rectum Anterior resection (Low TME) Colo-rectal 10%


(+/- Defunctioning
stoma)

Anal verge Abdomino-perineal excision of None n/a


colon and rectum

In the emergency setting, where the bowel has perforated, the risk of an
anastomotic breakdown is much greater, particularly when the anastomosis is
colon-colon. In this situation, an end colostomy is often safer and can be reversed
later. When resection of the sigmoid colon is performed and an end colostomy is
fashioned the operation is referred to as a Hartmans procedure. Whilst left sided
resections are more risky, ileo-colic anastomoses are relatively safe even in the
emergency setting and do not need to be defunctioned.
gathered by dr. elbarky, for free, not intended for profit by anyone elsewhere.

References
A review of the diagnosis and management of colorectal cancer and a summary of
the UK National Institute of Clinical Excellence guidelines is provided in:
Poston G, et al . Diagnosis and management of colorectal cancer: summary of
NICE guidance. BMJ 2011: 343: d 6751.

Next question 

Display my notes on this topic

          

Save my notes

Question stats

A 15.6%
B 43.2%
C 7.9%
D 25.5%
E 7.7%

43.2% of users answered this question correctly

Search eMRCS

Search term Go

 Open MRCS Part A textbook (../review/textbook.php)


Question 17 of 74
gathered by dr. elbarky, for free, not intended for profit by anyone elsewhere.

 

What is the earliest complication that can occur following construction of an


ileostomy?

Prolapse

Retraction

Necrosis

Parastomal hernia

Dermatitis

Read the question very carefully here. We did not ask for the most common
complication, rather, we asked for the earliest one. Dermatitis is the most
common, but its not the earliest. The earliest complications are vascular ones
and these usually occurs as a result of either inadvertent mesenteric division
or as a result of a stoma thats too tight.

Construction of a stoma may be complicated by several factors. Necrosis may


occur because of technical errors in mesenteric division, excessive tension or
failure to construct a fascial defect of adequate size to permit safe passage of the
mesentery and the bowel.

Please rate this question:

 Discuss and give feedback

Next question 

Ileostomy

Ileostomies are generally fashioned in the right iliac fossa in a triangle between the
anterior superior iliac spine, symphysis pubis and umbilicus. They should lie one-
third of the distance between the umbilicus and anterior superior iliac spine. A 2cm
skin incision is made and dissection continued through the rectus muscle. A
cruciate incision should be made, and generally dilated to admit two fingers. The
ileum is brought through the incisions and should generally be spouted to a final
length of 2.5cm. Ileostomies that are too short may cause problems with
gathered by dr. elbarky, for free, not intended for profit by anyone elsewhere.

appliance fixation and those which are too long may cause problems with tension
and subsequent ulceration or prolapse.

Complications following ileostomy construction include dermatitis (most


common), bowel obstruction (usually adhesional) and prolapse.

Ileostomy output is roughly in the range of 5-10ml/Kg/ 24 hours. Output in excess


of 20ml/Kg/24 hours usually requires supplementary intravenous fluids. Excessive
fluid losses are generally managed by administration of oral loperamide (up to
4mg QDS) to try and slow the output. Foods containing gelatine may also thicken
output. Early high output is not uncommon and most patients (50%) will respond to
conservative management.

Next question 

Display my notes on this topic

          

Save my notes

Question stats

A 9.6%
B 17.9%
C 45.6%
D 10%
E 16.9%

45.6% of users answered this question correctly

Search eMRCS

Search term Go

Question 18 of 74
gathered by dr. elbarky, for free, not intended for profit by anyone elsewhere.

 

A 23 year old lady presents with a posteriorly sited fissure in ano. Treatment with
stool softeners and topical GTN has failed to improve matters. Which of the
following would be the most appropriate next management step?

Lords procedure

Injection of botulinum toxin

Lateral internal sphincterotomy

Endoanal advancement flap

Surgical division of the external anal sphincter

The next most appropriate management option when GTN or other topical nitrates
has failed is to consider botulinum toxin injection. In males a lateral internal
sphincterotomy would be an acceptable alternative. In a female who has yet to
conceive this may predispose to delayed increased risk of sphincter dysfunction.
Division of the external sphincter will result in faecal incontinence and is not a
justified treatment for fissure.

Please rate this question:

 Discuss and give feedback

Next question 

Rectal bleeding

Rectal bleeding is a common cause for patients to be referred to the surgical clinic.
In the clinical history it is useful to try and localise the anatomical source of the
blood. Bright red blood is usually of rectal anal canal origin, whilst dark red blood is
more suggestive of a proximally sited bleeding source. Blood which has entered
the GI tract from a gastro-duodenal source will typically resemble malaena due to
the effects of the digestive enzymes on the blood itself.

In the table below we give some typical bleeding scenarios together with physical
examination findings and causation.

Cause Type of Features in history Examination findings


bleeding
gathered by dr. elbarky, for free, not intended for profit by anyone elsewhere.

Fissure in Bright red Painful bleeding that Muco-epithelial defect


ano rectal occurs post defecation usually in the midline
bleeding in small volumes. posteriorly (anterior
Usually antecedent fissures more likely to
features of be due to underlying
constipation disease)

Haemorroids Bright red Post defecation Normal colon and


rectal bleeding noted both on rectum. Proctoscopy
bleeding toilet paper and drips may show internal
into pan. May be haemorrhoids. Internal
alteration of bowel haemorrhoids are
habit and history of usually impalpable.
straining. No blood
mixed with stool. No
local pain.

Crohns Bright red Bleeding that is Perineal inspection may


disease or mixed accompanied by other show fissures or
blood symptoms such as fistulae. Proctoscopy
altered bowel habit, may demonstrate
malaise, history of indurated mucosa and
fissures (especially possibly strictures. Skip
anterior) and lesions may be noted at
abscesses. colonoscopy.

Ulcerative Bright red Diarrhoea, weight loss, Proctitis is the most


colitis bleeding nocturnal marked finding. Peri
often incontinence, passage anal disease is usually
mixed of mucous PR. absent. Colonoscopy
with stool will show continuous
mucosal lesion.

Rectal Bright red Alteration of bowel Usually obvious


cancer blood habit. Tenesmus may mucosal abnormality.
mixed be present. Symptoms Lesion may be fixed or
volumes of metastatic disease. mobile depending upon
disease extent.
Surrounding mucosa
often normal, although
polyps may be present.
Image showing a fissure in ano. Typically these are located posteriorly and in the
midline. Fissures at other sites may be associated with underlying disease.
gathered by dr. elbarky, for free, not intended for profit by anyone elsewhere.

(https://d2zgo9qer4wjf4.cloudfront.net/images_eMRCS/swb054b.jpg)
Image sourced from Wikipedia
(https://d2zgo9qer4wjf4.cloudfront.net
(http://en.wikipedia.org/wiki/Anal
/images_eMRCS/swb054b.jpg)
fissure)

Colonoscopic image of internal haemorroids. Note these may often be impalpable.

(https://d2zgo9qer4wjf4.cloudfront.net/images_eMRCS/swb055b.jpg)
Image sourced from Wikipedia
(https://d2zgo9qer4wjf4.cloudfront.net
(http://en.wikipedia.org
/images_eMRCS/swb055b.jpg)
/wiki/Haemorrhoids)
Investigation
All patients presenting with rectal bleeding require digital rectal examination
and procto-sigmoidoscopy as a minimal baseline.
gathered by dr. elbarky, for free, not intended for profit by anyone elsewhere.

Remember that haemorrhoids are typically impalpable and to attribute


bleeding to these in the absence of accurate internal inspection is
unsatisfactory.
In young patients with no other concerning features in the history a carefully
performed sigmoidoscopy that demonstrates clear haemorrhoidal disease
may be sufficient. If clear views cannot be obtained then patients require
bowel preparation with an enema and a flexible sigmoidscopy performed.
In those presenting with features of altered bowel habit or suspicion of
inflammatory bowel disease a colonoscopy is the best test.
Patients with excessive pain who are suspected of having a fissure may
require an examination under general or local anaesthesia.
In young patients with external stigmata of fissure and a compatible history
it is acceptable to treat medically and defer internal examination until the
fissure is healed. If the fissure fails to heal then internal examination
becomes necessary along the lines suggested above to exclude internal
disease.

Special tests
In patients with a malignancy of the rectum the staging investigations
comprise an MRI of the rectum to identify circumferential resection margin
compromise and to identify mesorectal nodal disease. In addition to this CT
scanning of the chest abdomen and pelvis is necessary to stage for more
distant disease. Some centres will still stage the mesorectum with endo
rectal ultrasound but this is becoming far less common.

Patients with fissure in ano who are being considered for surgical
sphincterotomy and are females who have an obstetric history should
probably have ano rectal manometry testing performed together with endo
anal ultrasound. As this service is not universally available it is not
mandatory but in the absence of such information there are continence
issues that may arise following sphincterotomy.

Management

Disease Management

Fissure in ano GTN ointment 0.2% or diltiazem cream applied topically is


the usual first line treatment. Botulinum toxin for those who
fail to respond. Internal sphincterotomy for those who fail
with botox, can be considered earlier in males.
Haemorroids Lifestyle advice, for small internal haemorrhoids can
consider injection sclerotherapy or rubber band ligation. For
external haemorrhoids consider haemorrhoidectomy.
Modern options include HALO procedure and stapled
gathered by dr. elbarky, for free, not intended for profit by anyone elsewhere.

haemorrhoidectomy.

Inflammatory Medical management- although surgery may be needed for


bowel disease fistulating Crohns (setons).

Rectal cancer Anterior resection or abdomino-perineal excision of the


colon and rectum. Total mesorectal excision is now standard
of care. Most resections below the peritoneal reflection will
require defunctioning ileostomy. Most patients will require
preoperative radiotherapy.

Next question 

Display my notes on this topic

          

Save my notes

Question stats

A 10.6%
B 54.5%
C 21.1%
D 7.5%
E 6.3%

54.5% of users answered this question correctly

Search eMRCS

Search term Go

Question 19 of 74
gathered by dr. elbarky, for free, not intended for profit by anyone elsewhere.

 

A 28 year old male presents with a discharging sinus in his natal cleft. He is found
to have a pilonidal sinus. Which statement is false?

Can occur in webs of fingers and the axilla

After drainage pilonidal abscesses should not be primarily closed

A rare complication is squamous cell carcinoma

In a patient with an acute abscess the Bascoms procedure is the


treatment of choice.

Treatment involves excising or laying open the sinus tract.

When performing incision and drainage for pilonidal abscess try to avoid
making the incision in the midline of the natal cleft.

Acute pilonidal abscesses should receive simple incision and drainage. Definitive
treatments such as a Bascoms procedure should not be undertaken when acute
sepsis is present.

Please rate this question:

 Discuss and give feedback

Next question 

Pilonidal sinus

Occur as a result of hair debris creating sinuses in the skin (Bascom theory).
Usually in the natal cleft of male patients after puberty.
It is more common in Caucasians related to their hair type and growth
patterns.
The opening of the sinus is lined by squamous epithelium, but most of its
wall consists of granulation tissue. Up to 50 cases of squamous cell
carcinoma have been described in patients with chronic pilonidal sinus
disease.
Hairs become trapped within the sinus.
Clinically the sinus presents when acute inflammation occurs, leading to an
gathered by dr. elbarky, for free, not intended for profit by anyone elsewhere.

abscess. Patients may describe cycles of being asymptomatic and periods


of pain and discharge from the sinus.
Treatment is difficult and opinions differ. Definitive treatment should never
be undertaken when acute infection or abscess is present as this will result
in failure.
Definitive treatments include the Bascom procedure with excision of the pits
and obliteration of the underlying cavity. The Karydakis procedure involves
wide excision of the natal cleft such that the surface is recontoured once the
wound is closed. This avoids the shearing forces that break off the hairs and
has reasonable results.

Pilonidal sinuses are most commonly located in the midline of the natal cleft, as
illustrated below

(https://d2zgo9qer4wjf4.cloudfront.net/images_eMRCS/swb073b.jpg)
Image sourced from Wikipedia
(https://d2zgo9qer4wjf4.cloudfront.net
(http://en.wikipedia.org/wiki/Pilonidal
/images_eMRCS/swb073b.jpg)
cyst)

Next question 
Display my notes on this topic

          
gathered by dr. elbarky, for free, not intended for profit by anyone elsewhere.

Save my notes

Question stats

A 24.3%
B 8.2%
C 13%
D 47%
E 7.6%

47% of users answered this question correctly

Search eMRCS

Search term Go

 Open MRCS Part A textbook (../review/textbook.php)

External links

+ Suggest a link

Dashboard

5

Question 20 of 74
gathered by dr. elbarky, for free, not intended for profit by anyone elsewhere.

 

A previously well 21 year old man is admitted with 2 week history of diarrhoea and
passage of blood and mucous rectally. He has previously undergone an ileocaecal
resection in the past for an inflammatory bowel disorder and takes mesalazine.
What is the most likely underlying cause?

Ulcerative colitis

Diversion proctitis

Crohns proctitis

Diverticular associated bleed

Irritable bowel syndrome

The history of a right sided resection is the patients young age are all strongly
suggestive of an existing diagnosis of Crohns disease (segmental resections are
not undertaken for UC). Since the bowel has remained in continuity, a diversion
colitis is not possible.

Please rate this question:

 Discuss and give feedback

Next question 

Rectal bleeding

Rectal bleeding is a common cause for patients to be referred to the surgical clinic.
In the clinical history it is useful to try and localise the anatomical source of the
blood. Bright red blood is usually of rectal anal canal origin, whilst dark red blood is
more suggestive of a proximally sited bleeding source. Blood which has entered
the GI tract from a gastro-duodenal source will typically resemble malaena due to
the effects of the digestive enzymes on the blood itself.

In the table below we give some typical bleeding scenarios together with physical
examination findings and causation.
Cause Type of Features in history Examination findings
bleeding
gathered by dr. elbarky, for free, not intended for profit by anyone elsewhere.

Fissure in Bright red Painful bleeding that Muco-epithelial defect


ano rectal occurs post defecation usually in the midline
bleeding in small volumes. posteriorly (anterior
Usually antecedent fissures more likely to
features of be due to underlying
constipation disease)

Haemorroids Bright red Post defecation Normal colon and


rectal bleeding noted both on rectum. Proctoscopy
bleeding toilet paper and drips may show internal
into pan. May be haemorrhoids. Internal
alteration of bowel haemorrhoids are
habit and history of usually impalpable.
straining. No blood
mixed with stool. No
local pain.

Crohns Bright red Bleeding that is Perineal inspection may


disease or mixed accompanied by other show fissures or
blood symptoms such as fistulae. Proctoscopy
altered bowel habit, may demonstrate
malaise, history of indurated mucosa and
fissures (especially possibly strictures. Skip
anterior) and lesions may be noted at
abscesses. colonoscopy.

Ulcerative Bright red Diarrhoea, weight loss, Proctitis is the most


colitis bleeding nocturnal marked finding. Peri
often incontinence, passage anal disease is usually
mixed of mucous PR. absent. Colonoscopy
with stool will show continuous
mucosal lesion.

Rectal Bright red Alteration of bowel Usually obvious


cancer blood habit. Tenesmus may mucosal abnormality.
mixed be present. Symptoms Lesion may be fixed or
volumes of metastatic disease. mobile depending upon
disease extent.
Surrounding mucosa
often normal, although
polyps may be present.
Image showing a fissure in ano. Typically these are located posteriorly and in the
midline. Fissures at other sites may be associated with underlying disease.
gathered by dr. elbarky, for free, not intended for profit by anyone elsewhere.

(https://d2zgo9qer4wjf4.cloudfront.net/images_eMRCS/swb054b.jpg)
Image sourced from Wikipedia
(https://d2zgo9qer4wjf4.cloudfront.net
(http://en.wikipedia.org/wiki/Anal
/images_eMRCS/swb054b.jpg)
fissure)

Colonoscopic image of internal haemorroids. Note these may often be impalpable.

(https://d2zgo9qer4wjf4.cloudfront.net/images_eMRCS/swb055b.jpg)
Image sourced from Wikipedia
(https://d2zgo9qer4wjf4.cloudfront.net
(http://en.wikipedia.org
/images_eMRCS/swb055b.jpg)
/wiki/Haemorrhoids)
Investigation
All patients presenting with rectal bleeding require digital rectal examination
and procto-sigmoidoscopy as a minimal baseline.
Remember that haemorrhoids are typically impalpable and to attribute
gathered by dr. elbarky, for free, not intended for profit by anyone elsewhere.

bleeding to these in the absence of accurate internal inspection is


unsatisfactory.
In young patients with no other concerning features in the history a carefully
performed sigmoidoscopy that demonstrates clear haemorrhoidal disease
may be sufficient. If clear views cannot be obtained then patients require
bowel preparation with an enema and a flexible sigmoidscopy performed.
In those presenting with features of altered bowel habit or suspicion of
inflammatory bowel disease a colonoscopy is the best test.
Patients with excessive pain who are suspected of having a fissure may
require an examination under general or local anaesthesia.
In young patients with external stigmata of fissure and a compatible history
it is acceptable to treat medically and defer internal examination until the
fissure is healed. If the fissure fails to heal then internal examination
becomes necessary along the lines suggested above to exclude internal
disease.

Special tests
In patients with a malignancy of the rectum the staging investigations
comprise an MRI of the rectum to identify circumferential resection margin
compromise and to identify mesorectal nodal disease. In addition to this CT
scanning of the chest abdomen and pelvis is necessary to stage for more
distant disease. Some centres will still stage the mesorectum with endo
rectal ultrasound but this is becoming far less common.

Patients with fissure in ano who are being considered for surgical
sphincterotomy and are females who have an obstetric history should
probably have ano rectal manometry testing performed together with endo
anal ultrasound. As this service is not universally available it is not
mandatory but in the absence of such information there are continence
issues that may arise following sphincterotomy.

Management

Disease Management

Fissure in ano GTN ointment 0.2% or diltiazem cream applied topically is


the usual first line treatment. Botulinum toxin for those who
fail to respond. Internal sphincterotomy for those who fail
with botox, can be considered earlier in males.
Haemorroids Lifestyle advice, for small internal haemorrhoids can
consider injection sclerotherapy or rubber band ligation. For
external haemorrhoids consider haemorrhoidectomy.
Modern options include HALO procedure and stapled
gathered by dr. elbarky, for free, not intended for profit by anyone elsewhere.

haemorrhoidectomy.

Inflammatory Medical management- although surgery may be needed for


bowel disease fistulating Crohns (setons).

Rectal cancer Anterior resection or abdomino-perineal excision of the


colon and rectum. Total mesorectal excision is now standard
of care. Most resections below the peritoneal reflection will
require defunctioning ileostomy. Most patients will require
preoperative radiotherapy.

Next question 

Display my notes on this topic

          

Save my notes

Question stats

A 23.6%
B 13.8%
C 53.4%
D 4.8%
E 4.5%

53.4% of users answered this question correctly

Search eMRCS

Search term Go

Question 21 of 74
gathered by dr. elbarky, for free, not intended for profit by anyone elsewhere.

 

A 34 year old lady presents to her general practitioner with peri anal discomfort.
The general practitioner diagnoses pruritus ani, which of the following is least
associated with the condition?

Hyperbilirubinaemia

Anal fissure

Leukaemia

Syphilis

Tuberculosis

Causes:

Systemic (DM, Hyperbilirubinaemia, aplastic anaemia)


Mechanical (diarrhoea, constipation, anal fissure)
Infections (STDs)
Dermatological
Drugs (quinidine, colchicine)
Topical agents

Please rate this question:

 Discuss and give feedback

Next question 

Ano rectal disease

Location: 3, 7, 11 o'clock position


Haemorrhoids Internal or external
Treatment: Conservative, Rubber band ligation,
Haemorrhoidectomy
Fissure in ano Location: midline 6 (posterior midline 90%) and 12 o'clock
position. Distal to the dentate line
Chronic fissure > 6/52: triad: Ulcer, sentinel pile, enlarged
anal papillae
gathered by dr. elbarky, for free, not intended for profit by anyone elsewhere.

Proctitis Causes: Crohn's, ulcerative colitis, Clostridium difficile

Ano rectal E.coli, staph aureus


abscess Positions: Perianal, Ischiorectal, Pelvirectal, Intersphincteric

Anal fistula Usually due to previous ano-rectal abscess


Intersphincteric, transsphincteric, suprasphincteric, and
extrasphincteric. Goodsalls rule determines location

Rectal prolapse Associated with childbirth and rectal intussceception. May


be internal or external

Pruritus ani Systemic and local causes

Anal neoplasm Squamous cell carcinoma commonest unlike


adenocarcinoma in rectum

Solitary rectal Associated with chronic straining and constipation.


ulcer Histology shows mucosal thickening, lamina propria
replaced with collagen and smooth muscle (fibromuscular
obliteration)

Rectal prolapse
Common especially in multiparous women.
May be internal or external.
Internal rectal prolapse can present insidiously.
External prolapse can ulcerate and in long term impair continence.
Diagnostic work up includes colonoscopy, defecating proctogram, ano rectal
manometry studies and if doubt exists an examination under anaesthesia.

Treatments for prolapse


In the acute setting reduce it (covering it with sugar may reduce swelling.
Delormes procedure which excises mucosa and plicates the rectum (high
recurrence rates) may be used for external prolapse.
Altmeirs procedure which resects the colon via the perineal route has lower
recurrence rates but carries the risk of anastamotic leak.
Rectopexy is an abdominal procedure in which the rectum is elevated and
usually supported at the level of the sacral promontory. Post operative
constipation may be reduced by limiting the dissection to the anterior plane
(laparoscopic ventral mesh rectopexy).
Pruritus ani
Extremely common.
Check not secondary to altered bowel habits (e.g. Diarrhoea)
Associated with underlying diseases such as haemorrhoids.
gathered by dr. elbarky, for free, not intended for profit by anyone elsewhere.

Examine to look for causes such as worms.


Proctosigmoidoscopy to identify associated haemorrhoids and exclude
cancer.
Treatment is largely supportive and patients should avoid using perfumed
products around the area.

Fissure in ano
Typically painful PR bleeding (bright red).
Nearly always in the posterior midline.
Usually solitary.

Treatment
Stool softeners.
Topical diltiazem (or GTN).
If topical treatments fail then botulinum toxin should be injected.
If botulinum toxin fails then males should probably undergo lateral internal
sphincterotomy.
Females who do not respond to botulinum toxin should undergo ano rectal
manometry studies and endo anal USS prior to being offered surgery such
as sphincterotomy.

Next question 

Display my notes on this topic

          

Save my notes

Question stats

A 23.8%
B 11.5%
C 22.3%
D 9.5%
E 32.9%

Question 22 of 74
gathered by dr. elbarky, for free, not intended for profit by anyone elsewhere.

 

A 60 year old lady is investigated for abdominal pain. A polyp is identified at the
proximal descending colon, three small polyps are also noted in the sigmoid colon.
The largest lesion is removed by snare polypectomy and the pathology report
states that this polyp is a low grade dysplastic adenoma measuring 3cm in
diameter. The remaining lesions are ablated using diathermy. What is the correct
management?

Repeat endoscopy in 1 year

No further endoscopic surveillance

Repeat endoscopy in 5 years

Segmental colonic resection

Repeat endoscopy in 3 years

She is in the high risk group and according to the 2020 guidelines should undergo
endoscopy at 3 years.

Please rate this question:

 Discuss and give feedback

Next question 

Colonic polyps

Colonic Polyps
May occur in isolation, or greater numbers as part of the polyposis syndromes. In
FAP greater than 100 polyps are typically present. The risk of malignancy in
association with adenomas is related to size, and is the order of 10% in a 1cm
adenoma. Isolated adenomas seldom give risk of symptoms (unless large and
distal). Distally sited villous lesions may produce mucous and if very large,
electrolyte disturbances may occur.

Follow up of colonic polyps


Group Action

Colorectal cancer Colonoscopy 1 year post resection


gathered by dr. elbarky, for free, not intended for profit by anyone elsewhere.

Large non pedunculated colorectal One off scope at 3 years


polyps (LNPCP), R0 resection

Large non pedunculated colorectal Site check at 2-6 months and then a
polyps (LNPCP) R1 or non en bloc further scope at 12 months
resection

High risk findings at baseline One off surveillance at 3 years


colonoscopy

No high risk findings at baseline No colonoscopic surveillance and invite


colonoscopy participation in NHSBCSP programme
when due

High risk findings


More than 2 premalignant polyps including 1 or more advanced
colorectal polyps
OR
More than 5 pre malignant polyps

Exceptions to guidelines
If patient more than 10 years younger than lower screening age and has polyps but
no high risk findings, consider colonoscopy at 5 or 10 years.

Segmental resection or complete colectomy should be considered when:

1. Incomplete excision of malignant polyp


2. Malignant sessile polyp
3. Malignant pedunculated polyp with submucosal invasion
4. Polyps with poorly differentiated carcinoma
5. Familial polyposis coli
-Screening from teenager up to 40 years by 2 yearly sigmoidoscopy/colonoscopy
-Panproctocolectomy and Ileostomy or Restorative Panproctocolectomy.

Rectal polypoidal lesions may be amenable to trans anal endoscopic microsurgery.

References
Rutter MD et al. British Society of Gastroenterology/Association of Coloproctology
of Great Britain and Ireland/Public Health England post- polypectomy and post-
colorectal cancer resection surveillance guidelines. Gut 2019;0:123.
Next question 
gathered by dr. elbarky, for free, not intended for profit by anyone elsewhere.

Display my notes on this topic

          

Save my notes

Question stats

A 56.7%
B 5.5%
C 5.5%
D 9.3%
E 23%

23% of users answered this question correctly

Search eMRCS

Search term Go

 Open MRCS Part A textbook (../review/textbook.php)

External links

+ Suggest a link

Dashboard

3

Question 23 of 74
gathered by dr. elbarky, for free, not intended for profit by anyone elsewhere.

 

A 56 year old man presents with his first attack of diverticulitis. Which of these
complications is least likely to ensue?

Formation of colonic strictures

Malignant transformation

Development of colovesical fistula

Formation of a pericolic abscess

Formation of a phlegmon

Diverticulitis may result in a number of complications. However, whilst malignant


disease may coincide with diverticulitis it is not, in itself, a risk factor for colonic
cancer.

Please rate this question:

 Discuss and give feedback

Next question 

Diverticular disease

Diverticular disease is a common surgical problem. It consists of herniation of


colonic mucosa through the muscular wall of the colon. The usual site is between
the taenia coli where vessels pierce the muscle to supply the mucosa. For this
reason, the rectum, which lacks taenia, is often spared.

Symptoms
Altered bowel habit
Bleeding
Abdominal pain

Complications
Diverticulitis
Haemorrhage
Development of fistula
Perforation and faecal peritonitis
gathered by dr. elbarky, for free, not intended for profit by anyone elsewhere.

Perforation and development of abscess


Development of diverticular phlegmon

Diagnosis
Patients presenting in clinic will typically undergo either a colonoscopy, CT
cologram or barium enema as part of their diagnostic work up. All tests can
identify diverticular disease. It can be far more difficult to confidently exclude
cancer, particularly in diverticular strictures.

Acutely unwell surgical patients should be investigated in a systematic way. Plain


abdominal films and an erect chest x-ray will identify perforation. An abdominal CT
scan (not a CT cologram) with oral and intravenous contrast will help to identify
whether acute inflammation is present but also the presence of local
complications such as abscess formation.

Severity Classification- Hinchey

I Para-colonic abscess

II Pelvic abscess

III Purulent peritonitis

IV Faecal peritonitis

Treatment
Increase dietary fibre intake.
Mild attacks of diverticulitis may be managed conservatively with
antibiotics.
Peri colonic abscesses should be drained either surgically or radiologically.
Recurrent episodes of acute diverticulitis requiring hospitalisation are a
relative indication for a segmental resection.
Hinchey IV perforations (generalised faecal peritonitis) will require a
resection and usually a stoma. This group have a very high risk of post
operative complications and usually require HDU admission.

Next question 
Display my notes on this topic

          
gathered by dr. elbarky, for free, not intended for profit by anyone elsewhere.

Save my notes

Question stats

A 9.7%
B 62.3%
C 8.8%
D 6.2%
E 13.1%

62.3% of users answered this question correctly

Search eMRCS

Search term Go

 Open MRCS Part A textbook (../review/textbook.php)

External links

+ Suggest a link

Dashboard

5

Question 24 of 74
gathered by dr. elbarky, for free, not intended for profit by anyone elsewhere.

 

A 22 year old lady has a long history of severe perianal Crohns disease with
multiple fistulae. She is keen to avoid a stoma. However, she has progressive
disease and multiple episodes of rectal bleeding. A colonoscopy shows rectal
disease only and a small bowel study shows no involvement with Crohns. What is
the best operative strategy?

Abdomino perineal excision of the colon and rectum

Proctectomy and end stoma

Pan proctocolectomy and ileoanal pouch

Loop colostomy alone

Sub total colectomy

Crohns disease is a contra indication to having an ileo-anal pouch as its


associated with very poor pouch function and significant complications.

Whilst the patient wishes to avoid a stoma, that's inevitable here.

Please rate this question:

 Discuss and give feedback

Next question 

Surgery for inflammatory bowel disease

Patients with inflammatory bowel disease (UC and Crohns) frequently present in
surgical practice. Ulcerative colitis may be cured by surgical resection
(Proctocolectomy), this is not the case in Crohns disease which may recur and
affect other areas of the gastrointestinal tract.

Ulcerative colitis
Elective indications for surgery include disease that is requiring maximal
therapy, or prolonged courses of steroids.
Longstanding UC is associated with a risk of malignant transformation.
Dysplastic transformation of the colonic epithelium with associated mass
gathered by dr. elbarky, for free, not intended for profit by anyone elsewhere.

lesions is an absolute indication for a proctocolectomy.


Emergency presentations of poorly controlled colitis that fails to respond to
medical therapy should usually be managed with a sub total colectomy.
Excision of the rectum is a procedure with a higher morbidity and is not
generally performed in the emergency setting. An end ileostomy is usually
created and the rectum either stapled off and left in situ, or, if the bowel is
very oedematous, may be brought to the surface as a mucous fistula.
Patients with IBD have a high incidence of DVT and appropriate
thromboprophylaxis is mandatory.
Restorative options in UC include an ileoanal pouch. This procedure can only
be performed whilst the rectum is in situ and cannot usually be undertaken
as a delayed procedure following proctectomy.
Ileoanal pouch complications include, anastomotic dehiscence, pouchitis
and poor physiological function with seepage and soiling.

Crohns disease
Surgical resection of Crohns disease does not equate with cure, but may
produce substantial symptomatic improvement.
Indications for surgery include complications such as fistulae, abscess
formation and strictures.
Extensive small bowel resections may result in short bowel syndrome and
localised stricturoplasty may allow preservation of intestinal length.
Staging of Crohns will usually involve colonoscopy and a small bowel study
(e.g. MRI enteroclysis).
Complex perianal fistulae are best managed with long term draining seton
sutures, complex attempts at fistula closure e.g. advancement flaps, may be
complicated by non healing and fistula recurrence.
Severe perianal and / or rectal Crohns may require proctectomy. Ileoanal
pouch reconstruction in Crohns carries a high risk of fistula formation and
pouch failure and is not recommended.
Terminal ileal Crohns remains the commonest disease site and these
patients may be treated with limited ileocaecal resections.
Terminal ileal Crohns may affect enterohepatic bile salt recycling and
increase the risk of gallstones.

Next question 

Display my notes on this topic

          
Save my notes
gathered by dr. elbarky, for free, not intended for profit by anyone elsewhere.

Question stats

A 12%
B 39.2%
C 33.6%
D 7%
E 8.3%

39.2% of users answered this question correctly

Search eMRCS

Search term Go

 Open MRCS Part A textbook (../review/textbook.php)

External links

+ Suggest a link

Dashboard

8

Question 25 of 74
gathered by dr. elbarky, for free, not intended for profit by anyone elsewhere.

 

A 19 year old female presents with colicky abdominal pain, bloating and alternating
constipation/diarrhoea. Her grandmother died from colon cancer at the age of 87
years. A digital rectal examination and general physical examination are normal.
What is the best course of action?

Measurement of faecal calprotectin

Arrange a barium enema

Undertake a colonoscopy

Undertake a proctoscopy

Undertake a rigid sigmoidoscopy

The family history is irrelevant, an 87 year old developing colorectal cancer is


sporadic chance. The patients risk of cancer is thus population baseline and
at age 20 this is very low indeed. The main differential would be inflammatory
bowel disease and a faecal calprotectin is a very sensitive screening test for
this.

This patient fulfills the Rome criteria for irritable bowel syndrome. Examination is
normal, therefore it's likely that this patient will have IBS. However, its prudent to
exclude IBD and since endoscopy is poorly tolerated in patients with IBS,
measurement of faecal calprotectin is a reasonable alternative.

Please rate this question:

 Discuss and give feedback

Next question 

Irritable bowel syndrome

The diagnosis of irritable bowel syndrome is made according to the ROME III
diagnostic criteria which state:

Recurrent abdominal pain or discomfort at 3 days per month for the past 3
months associated with two or more of the following:
gathered by dr. elbarky, for free, not intended for profit by anyone elsewhere.

Improvement with defecation.


Onset associated with a change in the frequency of stool.
Onset associated with a change in the form of the stool.

Features such as lethargy, nausea, backache and bladder symptoms may also
support the diagnosis

Red flag features should be inquired about:

Rectal bleeding
Unexplained/unintentional weight loss
Family history of bowel or ovarian cancer
Onset after 60 years of age

Suggested investigations are:

Full blood count


ESR/CRP
Coeliac disease screen (tissue transglutaminase antibodies)
Colonoscopy (if worrying symptoms, positive family history)
Thyroid function tests
Glucose (ensure not diabetic)

The NICE criteria state that blood tests alone will suffice in people fulfilling the
diagnostic criteria. We would point out that luminal colonic studies should be
considered early in patients with altered bowel habit referred to hospital and a
diagnosis of IBS should still be largely one of exclusion.

Treatment
Usually reduce fibre intake.
Tailored prescriptions of laxatives or loperamide according to clinical
picture.
Dietary modification (caffeine avoidance, less carbonated drinks).
Consider low dose tricyclic antidepressants if pain is a dominant symptom.
Biofeedback may help.

Next question 
Display my notes on this topic

          
gathered by dr. elbarky, for free, not intended for profit by anyone elsewhere.

Save my notes

Question stats

A 45.9%
B 6.4%
C 36.5%
D 5.2%
E 6%

45.9% of users answered this question correctly

Search eMRCS

Search term Go

 Open MRCS Part A textbook (../review/textbook.php)

External links

+ Suggest a link

Dashboard

5

Question 26 of 74
gathered by dr. elbarky, for free, not intended for profit by anyone elsewhere.

 

A 77 year old man is admitted with large bowel obstruction and on investigation
with an abdominal CT scan is found to have an obstructing cancer of the sigmoid
colon. What is the most appropriate course of action?

Laparotomy, sigmoid colectomy and formation of end colostomy

Laparotomy and loop colostomy

Laparotomy and loop ileostomy

Laparotomy, high anterior resection and colo-rectal anastomosis

Palliation

Obstructing sigmoid cancers can be resected or stented. If stented, then the


patient may need definitive surgery later. If resected, then a resection and end
colostomy (Hartmann's ) procedure is usually undertaken because of the risks of
anastomotic leak in the setting of anastomosing obstructed colon to rectum.

Please rate this question:

 Discuss and give feedback

Next question 

Large bowel obstruction

Colonic obstruction remains a common surgical problem. It is most commonly due


to malignancy (60%) and diverticular disease (20%). Volvulus affecting the colon
accounts for 5% of cases. Acute colonic pseudo-obstruction remains a potential
differential diagnosis in all cases. Intussusception affecting the colon (most often
due to tumours in the adult population) remains a rare but recognised cause.
The typical patient will present with gradual onset of progressive abdominal
distension, colicky abdominal pain and either obstipation or absolute constipation.
On examination abdominal distension is present, the presence of caecal
tenderness (assuming no overt evidence of peritonitis) is a useful sign to elicit. A
digital rectal examination and rigid sigmoidoscopy should be performed.
A plain abdominal x-ray is the usual first line test and; the caecal diameter and
ileocaecal valve competency should be assessed on this film.

Imaging modalities
gathered by dr. elbarky, for free, not intended for profit by anyone elsewhere.

Debate long surrounds the use of CT versus gastrograffin enemas. The latter
investigation has always been the traditional method of determining whether a
structural lesion is indeed present. However, in the UK the use of this technique
has declined and in most units a CT scan will be offered as the first line
investigation by the majority of radiologists (and is advocated by the ACPGBI). In
most cases this will provide sufficient detail to allow operative planning, and since
malignancy accounts for most presentations may also stage the disease. In the
event that the radiologist cannot provide a clear statement of lesion site, the
surgeon should have no hesitation in requesting a contrast enema.

Surgical options
The decision as to when to operate or not is determined firstly by the patients
physiological status. Unstable patients require resuscitation prior to surgery and
admission to a critical care unit for invasive monitoring and potential inotropic
support may be needed. In patients who are otherwise stable the decision then
rests on the radiological and clinical findings. As a general rule the old adage that
the sun should not rise and set on unrelieved large bowel obstruction still holds
true. A caecal diameter of 12cm or more in the presence of complete obstruction
with a competent ileocaecal valve and caecal tenderness is a sign of impending
perforation and a relative indication for prompt surgery.

Right sided and transverse lesions


Right sided lesions producing large bowel obstruction should generally be treated
by right hemicolectomy or its extended variant if the lesion lies in the distal
transverse colon or splenic flexure. In these cases an ileocolic anastomosis may
be easily constructed and even in the emergency setting has a low risk of
anastomotic leak.

Left sided lesions


The options here lie between sub total colectomy and anastomosis, left
hemicolectomy with on table lavage and primary anastomosis, left hemicolectomy
and end colostomy formation and finally colonic stent insertion.
The usefulness of colonic stents was the subject of a Cochrane review in 2011.
The authors concluded that on the basis of the data that they reviewed, there was
no benefit from the use of colonic stents over conventional surgical resection with
a tendency to better outcomes seen in the surgical group (1). A more recently
conducted meta analysis met with the same conclusion (2). However, the recently
concluded CREST trial has suggested that self expanding metallic stents can
improve outcomes and avoids a stoma.

Rectosigmoid lesions
Lesions below the peritoneal reflection that are causing obstruction should
generally be treated with a loop colostomy. Primary resection of unstaged rectal
cancer would most likely carry a high CRM positivity rate and cannot be condoned.
Where the lesion occupies the distal sigmoid colon the usual practice would be to
perform a high anterior resection. The decision surrounding restoration of
intestinal continuity would lie with the operating surgeon.
gathered by dr. elbarky, for free, not intended for profit by anyone elsewhere.

References
1. Sagar J. Colorectal stents for the management of malignant colonic
obstructions. Cochrane Database of Systematic Reviews 2011, Issue 11. Art. No.:
CD007378. DOI: 10.1002/14651858.CD007378.pub2.
2. Cirrochi et al Safety and efficacy of endoscopic colonic stenting as a bridge to
surgery in the management of intestinal obstruction due to left colon and rectal
cancer: A systematic review and meta-analysis. Surg Oncol. 2013 Mar;22(1):14-21.

Next question 

Display my notes on this topic

          

Save my notes

Question stats

A 47.1%
B 25.9%
C 12.1%
D 9.5%
E 5.4%

47.1% of users answered this question correctly

Search eMRCS

Search term Go

 Open MRCS Part A textbook (../review/textbook.php)


Question 27 of 74
gathered by dr. elbarky, for free, not intended for profit by anyone elsewhere.

 

A 21 year old lady presents with a 6 month history of an offensive discharge from
the anus. She is otherwise well, but is increasingly annoyed at the need to wear
pads. On examination, she has a small epithelial defect in the 5 o'clock position,
approximately 3cm from the anal verge. What is the most likely cause?

Fissure in ano

Fistula in ano

External haemorrhoid

Squamous cell carcinoma of the anus

Proctalgia fugax

Fistulas usually occur following previous ano-rectal sepsis. The discharge may be
foul smelling and troublesome. Patients should be listed for examination under
anaesthesia. Fistulas which are low and have little or no sphincter involvement are
usually laid open.

Please rate this question:

 Discuss and give feedback

Next question 

Anal fistula

Fistula in ano is the most common form of ano rectal sepsis. Fistulae will have
both an internal opening and external opening, these will be connected by tract(s).
Complexity arises because of the potential for multiple entry and exit sites,
together with multiple tracts. Fistulae are classified into four main groups
according to anatomical location and the degree of sphincter involvement. Simple
uncomplicated fistulae are low and do not involve more than 30% of the external
sphincter. Complex fistulae involve the sphincter, have multiple branches or are
non cryptoglandular in origin[1]
Assessment
Examination of the perineum for signs of trauma, external openings or the
stigmata of IBD is important. Digital rectal examination may reveal the cord linking
the internal and external openings. At the same time the integrity of the sphincter
gathered by dr. elbarky, for free, not intended for profit by anyone elsewhere.

mechanism can be assessed. Low, uncomplicated fistulas may not require any
further assessment, other groups will usually require more detailed investigation.
For the fistula, the use of endo-anal USS with instillation of hydrogen peroxide into
the fistula tract may be helpful. Ano-rectal MRI scanning is also a useful tool, it is
sensitive and specific for the identification of fistula anatomy, branching tracts and
identifying occult sphincter involvement[2].

Identification of the internal opening


Fistulas with an external opening less than 3cm from the anal verge will typically
obey Goodsalls rule (see below).

(https://d2zgo9qer4wjf4.cloudfront.net/images_eMRCS/swb142b.png)
Image sourced from Wikipedia (https://d2zgo9qer4wjf4.cloudfront.net
(http://en.wikipedia.org/wiki/Goodsall) /images_eMRCS/swb142b.png)
Therapies
Seton suture
A seton is a piece of material that is passed through the fistula between the
internal and external openings that allows the drainage of sepsis. This is important
as undrained septic foci may drain along the path of least resistance, which may
result in the development of accessory tracts and openings. Their main use is in
treating complex fistula. Two types of seton are recognised, simple and cutting.
Simple setons lie within the fistula tract and encourage both drainage and fibrosis.
A cutting seton is inserted and the skin incised. The suture is tightened and re-
tightened at regular intervals. This may convert a high fistula to a low fistula. Since
the tissue will scar surrounding the fistula it is hoped that this technique will
minimise incontinence[3]. Unfortunately, a large retrospective review of the
literature related to the use of cutting setons has found that they are associated
with a 12% long term incontinence rate [4]

Fistulotomy
Low fistulas, that are simple should be treated by fistulotomy once the acute
sepsis has been controlled. Fistulotomy (where safe) provides the highest healing
rates [5]. Because fistulotomy is regarded as having a high cure rate, there are
some who prefer to use this technique with more extensive sphincter involvement.
gathered by dr. elbarky, for free, not intended for profit by anyone elsewhere.

In these patients the fistulotomy is performed as for a low fistula. However, the
muscle that is encountered is then divided and reconstructed with an overlapping
sphincter repair. A price is paid in terms of incontinence with this technique and up
to 12.5% of patients who were continent pre-operatively will have issues relating to
continence post procedure[6]. The same group also randomised between
fistulotomy and sphincter reconstruction and ano-rectal advancement flaps for the
treatment of complex cryptoglandular fistulas and reported similar outcomes in
terms of recurrence (>90%) and disturbances to continence (20%)[7].
Other authors have found adverse outcomes following fistulotomy in patients who
have undergone previous surgery, are of female gender or who have high internal
openings [8], in these patients careful assessment of pre-operative sphincter
function should be considered mandatory prior to fistulotomy.

Anal fistula plugs and fibrin glue


The desire to avoid injury to the sphincter complex has led to surgeons using both
fibrin glue and plugs to try and improve fistula healing. Meticulous preparation of
the tract and prior use of a draining seton is likely to improve chances of success.
The use of anal fistula plugs in high transphincteric fistula of cryptoglandular
origin is to be discouraged because of the high incidence of non response in
patients treated with such devices [9]In most patients septic complications are the
reasons for failure [10]. Fibrin glue is a popular option for the treatment of fistula.
There is variability of reported healing rates In some cases initial success rates of
up to 50% healing at six months are reported (in patients with complex cryptogenic
fistula). Of these successes 25% suffer a long term recurrence of fistula [11].
There are, however, no obvious cases of damage to the sphincter complex and the
use of the devices does not appear to adversely impact on subsequent surgical
options.

Ano-rectal advancement flaps


This procedure is primarily directed at high fistulae, and is considered attractive as
a sphincter saving operation. The procedure is performed either with the patient in
the prone jack knife position or in lithotomy (depending upon the site of the
fistula). The dissection is commenced in the sub mucosal plane (which may be
infiltrated with dilute adrenaline solution to ease dissection). The dissection is
continued into healthy proximal tissue. This is brought down and sutured over the
defect.
Follow up of patients with cryptoglandular fistulas treated with advancement flaps
shows a success in up to 80% patients[12-14]. With most recurrences occurring in
the first 6 months following surgery[12]. Continence was affected in some
patients, with up to 10% describing major continence issues post operatively.

Ligation of the intersphincteric tract procedure


In this procedure an incision is made in the intersphincteric groove and the fistula
tract dissected out in this plane and divided. A greater than 90% cure rate within 4
weeks was initially reported[15]. Others have subsequently performed similar
studies on larger numbers of patients with similar success rates.

Fistulotomy at the time of abscess drainage?


gathered by dr. elbarky, for free, not intended for profit by anyone elsewhere.

A Cochrane review conducted in 2010 suggests that primary fistulotomy for low,
uncomplicated fistula in ano may be safe and associated with better outcomes in
relation to long term chronic sepsis[16]. However, there is a danger that such
surgery performed by non specialists may result in a higher complication rate and
therefore the traditional teaching is that primary treatment of acute sepsis is
incision and drainage only. All agree that high/ complex fistulae should never be
subject to primary fistulotomy in the acute setting.

References
1. Parks, A.G., P.H. Gordon, and J.D. Hardcastle, A classification of fistula-in-ano. Br
J Surg, 1976. 63(1): p. 1-12.
2. Lunniss, P.J., et al., Magnetic resonance imaging of fistula-in-ano. Dis Colon
Rectum, 1994. 37(7): p. 708-18.
3. Misra, M.C. and B.M. Kapur, A new non-operative approach to fistula in ano. Br J
Surg, 1988. 75(11): p. 1093-4.
4. Ritchie, R.D., J.M. Sackier, and J.P. Hodde, Incontinence rates after cutting seton
treatment for anal fistula. Colorectal Dis, 2009. 11(6): p. 564-71.
5. Tyler, K.M., C.B. Aarons, and S.M. Sentovich, Successful sphincter-sparing
surgery for all anal fistulas. Dis Colon Rectum, 2007. 50(10): p. 1535-9.
6. Perez, F., et al., Prospective clinical and manometric study of fistulotomy with
primary sphincter reconstruction in the management of recurrent complex fistula-
in-ano. Int J Colorectal Dis, 2006. 21(6): p. 522-6.
7. Perez, F., et al., Randomized clinical and manometric study of advancement flap
versus fistulotomy with sphincter reconstruction in the management of complex
fistula-in-ano. Am J Surg, 2006. 192(1): p. 34-40.
8. Garcia-Aguilar, J., et al., Anal fistula surgery. Factors associated with recurrence
and incontinence. Dis Colon Rectum, 1996. 39(7): p. 723-9.
9. Ortiz, H., et al., Randomized clinical trial of anal fistula plug versus endorectal
advancement flap for the treatment of high cryptoglandular fistula in ano. Br J
Surg, 2009. 96(6): p. 608-12.
10. El-Gazzaz, G., M. Zutshi, and T. Hull, A retrospective review of chronic anal
fistulae treated by anal fistulae plug. Colorectal Dis, 2010. 12(5): p. 442-7.
11. Haim, N., et al., Long-term results of fibrin glue treatment for cryptogenic
perianal fistulas: a multicenter study. Dis Colon Rectum, 2011. 54(10): p. 1279-83.
12. Ortiz, H., et al., Length of follow-up after fistulotomy and fistulectomy
associated with endorectal advancement flap repair for fistula in ano. Br J Surg,
2008. 95(4): p. 484-7.
13. Kodner, I.J., et al., Endorectal advancement flap repair of rectovaginal and other
complicated anorectal fistulas. Surgery, 1993. 114(4): p. 682-9; discussion 689-90.
14. Abbas, M.A., R. Lemus-Rangel, and A. Hamadani, Long-term outcome of
endorectal advancement flap for complex anorectal fistulae. Am Surg, 2008.
74(10): p. 921-4.
15. Rojanasakul, A., et al., Total anal sphincter saving technique for fistula-in-ano;
the ligation of intersphincteric fistula tract. J Med Assoc Thai, 2007. 90(3): p.
581-6.
16. Malik, A.I., R.L. Nelson, and S. Tou, Incision and drainage of perianal abscess
with or without treatment of anal fistula. Cochrane Database Syst Rev, 2010(7): p.
CD006827.
gathered by dr. elbarky, for free, not intended for profit by anyone elsewhere.

Next question 

Display my notes on this topic

          

Save my notes

Question stats

A 9.8%
B 72.3%
C 5.6%
D 6.3%
E 5.9%

72.3% of users answered this question correctly

Search eMRCS

Search term Go

 Open MRCS Part A textbook (../review/textbook.php)

External links

+ Suggest a link

Question 28 of 74
gathered by dr. elbarky, for free, not intended for profit by anyone elsewhere.

 

A 68 year old man with ulcerative colitis is admitted with an exacerbation. You are
called to see him because he is having brisk dark PR bleeding. He has been on
intravenous hydrocortisone for 5 days. The gastroenterologists have done an OGD
to exclude a duodenal ulcer, this was normal. What is the best course of action?

Sub total colectomy and end ileostomy

Pan proctocolectomy and end ileostomy

Colonoscopy

CT angiogram

Flexible sigmoidoscopy

This man requires surgery to remove the bleeding segment of bowel. Medical
management has failed here. Note that a pan proctocolectomy is not a suitable
option in the emergency setting because there is increased morbidity from the
pelvic dissection. In the unlikely event that a sub total colectomy did not address
the bleeding then consideration may have to be given to removal of the rectum but
this would not usually be the case. Note that in this case, there is not really any
benefit to be derived from imaging, endoscopy would be very dangerous and risk
perforation as the bowel would be very friable.

Please rate this question:

 Discuss and give feedback

Next question 

Lower Gastrointestinal bleeding

Colonic bleeding
This typically presents as bright red or dark red blood per rectum. Colonic bleeding
rarely presents as malaena type stool, this is because blood in the colon has a
powerful laxative effect and is rarely retained long enough for transformation to
occur and because the digestive enzymes present in the small bowel are not
present in the colon. Up to 15% of patients presenting with haemochezia will have
an upper gastrointestinal source of haemorrhage.
gathered by dr. elbarky, for free, not intended for profit by anyone elsewhere.

As a general rule right sided bleeds tend to present with darker coloured blood
than left sided bleeds. Haemorrhoidal bleeding typically presents as bright red
rectal bleeding that occurs post defecation either onto toilet paper or into the toilet
pan. It is very unusual for haemorrhoids alone to cause any degree of
haemodynamic compromise.

Causes

Cause Presenting features

Colitis Bleeding may be brisk in advanced cases, diarrhoea is


commonly present. Abdominal x-ray may show featureless
colon.

Diverticular Acute diverticulitis often is not complicated by major


disease bleeding and diverticular bleeds often occur sporadically.
75% all will cease spontaneously within 24-48 hours.
Bleeding is often dark and of large volume.

Cancer Colonic cancers often bleed and for many patients this may
be the first sign of the disease. Major bleeding from early
lesions is uncommon

Haemorrhoidal Typically bright red bleeding occurring post defecation.


bleeding Although patients may give graphic descriptions bleeding
of sufficient volume to cause haemodynamic compromise
is rare.

Angiodysplasia Apart from bleeding, which may be massive, these


arteriovenous lesions cause little in the way of symptoms.
The right side of the colon is more commonly affected.

Management
Prompt correction of any haemodynamic compromise is required. Unlike
upper gastrointestinal bleeding the first line management is usually
supportive. This is because in the acute setting endoscopy is rarely helpful.
When haemorrhoidal bleeding is suspected a proctosigmoidoscopy is
reasonable as attempts at full colonoscopy are usually time consuming and
often futile.
In the unstable patient the usual procedure would be an angiogram (either
CT or percutaneous), when these are performed during a period of
haemodynamic instability they may show a bleeding point and may be the
only way of identifying a patch of angiodysplasia.
In others who are more stable the standard procedure would be a
colonoscopy in the elective setting. In patients undergoing angiography
attempts can be made to address the lesion in question such as coiling.
Otherwise surgery will be necessary.
In patients with ulcerative colitis who have significant haemorrhage the
gathered by dr. elbarky, for free, not intended for profit by anyone elsewhere.

standard approach would be a sub total colectomy, particularly if medical


management has already been tried and is not effective.

Indications for surgery


Patients > 60 years
Continued bleeding despite endoscopic intervention
Recurrent bleeding
Known cardiovascular disease with poor response to hypotension

Surgery
Selective mesenteric embolisation if life threatening bleeding. This is most helpful
if conducted during a period of relative haemodynamic instability. If all
haemodynamic parameters are normal then the bleeding is most likely to have
stopped and any angiography normal in appearance. In many units a CT angiogram
will replace selective angiography but the same caveats will apply.

If the source of colonic bleeding is unclear; perform a laparotomy, on table colonic


lavage and following this attempt a resection. A blind sub total colectomy is most
unwise, for example bleeding from an small bowel arterio-venous malformation
will not be treated by this manoeuvre.

Summary of Acute Lower GI bleeding recommendations


Consider admission if:
* Over 60 years
* Haemodynamically unstable/profuse PR bleeding
* On aspirin or NSAID
* Significant co morbidity

Management
All patients should have a history and examination, PR and proctoscopy
Colonoscopic haemostasis aimed for in post polypectomy or diverticular
bleeding

References
http://www.sign.ac.uk/guidelines/fulltext/105/index.html

Next question 
Display my notes on this topic

          
gathered by dr. elbarky, for free, not intended for profit by anyone elsewhere.

Save my notes

Question stats

A 42%
B 17.1%
C 10.6%
D 23.9%
E 6.5%

42% of users answered this question correctly

Search eMRCS

Search term Go

 Open MRCS Part A textbook (../review/textbook.php)

External links

+ Suggest a link

Dashboard

5

Question 29 of 74
gathered by dr. elbarky, for free, not intended for profit by anyone elsewhere.

 

A 78 year old man has undergone a hemi-arthroplasty for an intracapsular hip


fracture. Post operatively he develops electrolyte derangement and receives
intravenous fluids. Over the next 24 hours he develops marked abdominal
distension. On examination, he has a tense, tympanic abdomen which is not
painful. A contrast enema shows flow of contrast through to the caecum and
through the ileocaecal valve. What is the most likely cause?

Ogilvies syndrome

Diverticular stricture

Malignant stricture

Volvulus

Adhesive obstruction

Patients with electrolyte disturbance and previous surgery may develop colonic
pseudo-obstruction (Ogilvies syndrome). The diagnosis is made using a contrast
enema and treatment is usually directed at the underlying cause with colonic
decompression if indicated.

Please rate this question:

 Discuss and give feedback

Next question 

Colonic obstruction

Cause Features Treatment


Cause Features Treatment

Cancer Usually insidious onset Establish diagnosis


History of progressive (e.g. contrast enema/
gathered by dr. elbarky, for free, not intended for profit by anyone elsewhere.

constipation endoscopy)
Systemic features (e.g. Laparotomy and
anaemia) resection, stenting,
Abdominal distension defunctioning
Absence of bowel gas distal colostomy or bypass
to site of obstruction

Diverticular Usually history of previous Once diagnosis


stricture acute diverticulitis established, usually
Long history of altered bowel surgical resection
habit Colonic stenting should
Evidence of diverticulosis on not be performed for
imaging or endoscopy benign disease

Volvulus Twisting of bowel around its Initial treatment is to


mesentery untwist the loop, a
Sigmoid colon affected in flexible sigmoidoscopy
76% cases may be needed
Patients usually present with Those with clinical
abdominal pain, bloating and evidence of ischaemia
constipation should undergo surgery
Examination usually shows Patient with recurrent
asymmetrical distension volvulus should
Plain X-rays usually show undergo resection
massively dilated sigmoid
colon, loss of haustra and U
shape are typical, the loop
may contain fluid levels

Acute colonic Symptoms and signs of large Colonoscopic


pseudo- bowel obstruction with no decompression
obstruction lesion Correct metabolic
Usually associated with disorders
metabolic disorders IV neostigmine
Usually a cut off in the left Surgery
colon (82% cases)
Although abdomen tense
and distended, it is usually
not painful
All patients should undergo
contrast enema (may be
therapeutic)
Next question 
gathered by dr. elbarky, for free, not intended for profit by anyone elsewhere.

Display my notes on this topic

          

Save my notes

Question stats

A 72.5%
B 6%
C 4.7%
D 10%
E 6.8%

72.5% of users answered this question correctly

Search eMRCS

Search term Go

 Open MRCS Part A textbook (../review/textbook.php)

External links

+ Suggest a link

Dashboard

3

Question 30 of 74
gathered by dr. elbarky, for free, not intended for profit by anyone elsewhere.

 

Which of the agents listed below is most likely to help a 22 year old lady with
severe peri anal Crohns disease and multiple anal fistulae. The acute sepsis has
been drained and setons are in place. She is already receiving standard non
biological therapy.

Trastuzumab

Bevacizumab

Imatinib

Cetuximab

Infliximab

Infliximab is a popular choice in managing complex peri anal Crohns. It is


absolutely vital that all sepsis is drained prior to starting therapy.

Please rate this question:

 Discuss and give feedback

Next question 

Biological agents

Agents Target Uses

Adalimumab TNF alpha inhibitor Crohns disease


Infliximab Rheumatoid disease
Etanercept

Bevacizumab Anti VEGF (anti angiogenic) Colorectal cancer


Renal
Glioblastoma
Agents Target Uses

Trastuzumab HER receptor Breast cancer


gathered by dr. elbarky, for free, not intended for profit by anyone elsewhere.

Imatinib Tyrosine kinase inhibitor Gastrointestinal stromal


tumours
Chronic myeloid leukaemia

Basiliximab IL2 binding site Renal transplants

Cetuximab Epidermal growth factor EGF positive colorectal


inhibitor cancers

Detailed understanding of the actions of biological agents is well beyond the


scope of the MRCS syllabus. However, many of these drugs are being frequently
encountered in surgical patients.

Next question 

Display my notes on this topic

          

Save my notes

Question stats

A 8.9%
B 6.1%
C 15.3%
D 6.4%
E 63.2%

63.2% of users answered this question correctly

Search eMRCS

Question 31 of 74
gathered by dr. elbarky, for free, not intended for profit by anyone elsewhere.

 

A 55 year old man is found to have a carcinoma of the sigmoid colon on screening
colonoscopy. How should this be staged?

MRI of the abdomen and CT of the chest

Liver MRI and Chest CT

CT scanning of the chest, abdomen and pelvis alone

MRI of the rectum and CT of the abdomen and chest

Endoluminal USS and CT scanning of the abdomen

Colonic cancers are staged with CT scanning of the chest, abdomen and
pelvis.

Rectal cancer is staged with MRI rectum (and sometimes endolumenal USS for
low T1 lesions) together with CT scanning of the chest, abdomen and pelvis.
Historically, colonic cancer was staged with liver USS and CXR. However, modern
imaging has made this practice obsolete.

Please rate this question:

 Discuss and give feedback

Next question 

Colorectal cancer screening and diagnosis

Overview
Most cancers develop from adenomatous polyps. Screening for colorectal
cancer has been shown to reduce mortality by 16%
The NHS now has a national screening programme offering screening every
2 years to all men and women aged 60 to 69 years. Patients aged over 70
years may request screening
Eligible patients are sent faecal occult blood (FOB) tests through the post.
This is being replaced by FIT testing.
Patients with abnormal results are offered a colonoscopy

- The NHS BOSS flexible sigmoidoscopy screening comprises a single flexible


gathered by dr. elbarky, for free, not intended for profit by anyone elsewhere.

sigmoidoscopy to patients aged 55 years

At colonoscopy, approximately:
5 out of 10 patients will have a normal exam
4 out of 10 patients will be found to have polyps which may be removed due
to their premalignant potential
1 out of 10 patients will be found to have cancer

Diagnosis
Essentially the following patients need referral:
- Altered bowel habit for more than six weeks
- New onset of rectal bleeding
- Symptoms of tenesmus

Colonoscopy is the gold standard, provided it is complete and good mucosal


visualisation is achieved. Other options include double contrast barium enema and
CT colonography.

Staging
Once a malignant diagnosis is made patients with colonic cancer will be staged
using chest / abdomen and pelvic CT. Patients with rectal cancer will also undergo
evaluation of the mesorectum with pelvic MRI scanning.

For examination purposes the Dukes and TNM systems are preferred.

Tumour markers
Carcinoembryonic antigen (CEA) is the main tumour marker in colorectal cancer.
Not all tumours secrete this, and it may be raised in conditions such as IBD.
However, absolute levels do correlate (roughly) with disease burden and it is once
again being used routinely in follow up.

Next question 

Display my notes on this topic

          
Save my notes

Question stats
gathered by dr. elbarky, for free, not intended for profit by anyone elsewhere.

A 7.1%
B 5.7%
C 63.1%
D 16.9%
E 7.2%

63.1% of users answered this question correctly

Search eMRCS

Search term Go

 Open MRCS Part A textbook (../review/textbook.php)

External links

+ Suggest a link

Dashboard

9

Question 32 of 74
gathered by dr. elbarky, for free, not intended for profit by anyone elsewhere.

 

An 83 year old man is admitted on the acute surgical take. His presenting
symptom is of painless, profuse rectal bleeding of dark blood. His medical history
comprises a previous TIA for which he takes clopidogrel and a statin. What is the
most likely underlying cause?

Meckels diverticulum

Colonic cancer

Diverticular bleed

Ischaemic colitis

Diverticulitis

The majority of patients with colonic bleeding will be found to have bleeding
secondary to diverticular disease. Of note, inflammation (i.e. diverticulitis) is not
seen in such cases. Around 70% will stop bleeding spontaneously. Anti platelet and
anti coagulants are sometimes complicating factors and may make bleeding less
likely to cease spontaneously. Ischaemic colitis often has more dominant colitis
symptoms.

Please rate this question:

 Discuss and give feedback

Next question 

Rectal bleeding

Rectal bleeding is a common cause for patients to be referred to the surgical clinic.
In the clinical history it is useful to try and localise the anatomical source of the
blood. Bright red blood is usually of rectal anal canal origin, whilst dark red blood is
more suggestive of a proximally sited bleeding source. Blood which has entered
the GI tract from a gastro-duodenal source will typically resemble malaena due to
the effects of the digestive enzymes on the blood itself.
In the table below we give some typical bleeding scenarios together with physical
examination findings and causation.

Cause Type of Features in history Examination findings


gathered by dr. elbarky, for free, not intended for profit by anyone elsewhere.

bleeding

Fissure in Bright red Painful bleeding that Muco-epithelial defect


ano rectal occurs post defecation usually in the midline
bleeding in small volumes. posteriorly (anterior
Usually antecedent fissures more likely to
features of be due to underlying
constipation disease)

Haemorroids Bright red Post defecation Normal colon and


rectal bleeding noted both on rectum. Proctoscopy
bleeding toilet paper and drips may show internal
into pan. May be haemorrhoids. Internal
alteration of bowel haemorrhoids are
habit and history of usually impalpable.
straining. No blood
mixed with stool. No
local pain.

Crohns Bright red Bleeding that is Perineal inspection may


disease or mixed accompanied by other show fissures or
blood symptoms such as fistulae. Proctoscopy
altered bowel habit, may demonstrate
malaise, history of indurated mucosa and
fissures (especially possibly strictures. Skip
anterior) and lesions may be noted at
abscesses. colonoscopy.

Ulcerative Bright red Diarrhoea, weight loss, Proctitis is the most


colitis bleeding nocturnal marked finding. Peri
often incontinence, passage anal disease is usually
mixed of mucous PR. absent. Colonoscopy
with stool will show continuous
mucosal lesion.

Rectal Bright red Alteration of bowel Usually obvious


cancer blood habit. Tenesmus may mucosal abnormality.
mixed be present. Symptoms Lesion may be fixed or
volumes of metastatic disease. mobile depending upon
disease extent.
Surrounding mucosa
often normal, although
polyps may be present.
Image showing a fissure in ano. Typically these are located posteriorly and in the
midline. Fissures at other sites may be associated with underlying disease.
gathered by dr. elbarky, for free, not intended for profit by anyone elsewhere.

(https://d2zgo9qer4wjf4.cloudfront.net/images_eMRCS/swb054b.jpg)
Image sourced from Wikipedia
(https://d2zgo9qer4wjf4.cloudfront.net
(http://en.wikipedia.org/wiki/Anal
/images_eMRCS/swb054b.jpg)
fissure)

Colonoscopic image of internal haemorroids. Note these may often be impalpable.

(https://d2zgo9qer4wjf4.cloudfront.net/images_eMRCS/swb055b.jpg)
Image sourced from Wikipedia
(https://d2zgo9qer4wjf4.cloudfront.net
(http://en.wikipedia.org
/images_eMRCS/swb055b.jpg)
/wiki/Haemorrhoids)
Investigation
All patients presenting with rectal bleeding require digital rectal examination
gathered by dr. elbarky, for free, not intended for profit by anyone elsewhere.

and procto-sigmoidoscopy as a minimal baseline.


Remember that haemorrhoids are typically impalpable and to attribute
bleeding to these in the absence of accurate internal inspection is
unsatisfactory.
In young patients with no other concerning features in the history a carefully
performed sigmoidoscopy that demonstrates clear haemorrhoidal disease
may be sufficient. If clear views cannot be obtained then patients require
bowel preparation with an enema and a flexible sigmoidscopy performed.
In those presenting with features of altered bowel habit or suspicion of
inflammatory bowel disease a colonoscopy is the best test.
Patients with excessive pain who are suspected of having a fissure may
require an examination under general or local anaesthesia.
In young patients with external stigmata of fissure and a compatible history
it is acceptable to treat medically and defer internal examination until the
fissure is healed. If the fissure fails to heal then internal examination
becomes necessary along the lines suggested above to exclude internal
disease.

Special tests
In patients with a malignancy of the rectum the staging investigations
comprise an MRI of the rectum to identify circumferential resection margin
compromise and to identify mesorectal nodal disease. In addition to this CT
scanning of the chest abdomen and pelvis is necessary to stage for more
distant disease. Some centres will still stage the mesorectum with endo
rectal ultrasound but this is becoming far less common.

Patients with fissure in ano who are being considered for surgical
sphincterotomy and are females who have an obstetric history should
probably have ano rectal manometry testing performed together with endo
anal ultrasound. As this service is not universally available it is not
mandatory but in the absence of such information there are continence
issues that may arise following sphincterotomy.

Management

Disease Management

Fissure in ano GTN ointment 0.2% or diltiazem cream applied topically is


the usual first line treatment. Botulinum toxin for those who
fail to respond. Internal sphincterotomy for those who fail
with botox, can be considered earlier in males.
Haemorroids Lifestyle advice, for small internal haemorrhoids can
consider injection sclerotherapy or rubber band ligation. For
external haemorrhoids consider haemorrhoidectomy.
Modern options include HALO procedure and stapled
gathered by dr. elbarky, for free, not intended for profit by anyone elsewhere.

haemorrhoidectomy.

Inflammatory Medical management- although surgery may be needed for


bowel disease fistulating Crohns (setons).

Rectal cancer Anterior resection or abdomino-perineal excision of the


colon and rectum. Total mesorectal excision is now standard
of care. Most resections below the peritoneal reflection will
require defunctioning ileostomy. Most patients will require
preoperative radiotherapy.

Next question 

Display my notes on this topic

          

Save my notes

Question stats

A 7.2%
B 7.8%
C 44.2%
D 35%
E 5.8%

44.2% of users answered this question correctly

Search eMRCS

Search term Go

Question 33 of 74
gathered by dr. elbarky, for free, not intended for profit by anyone elsewhere.

 

A 55 year old man develops an acute colonic pseudo-obstruction following a


laminectomy. Despite correction of his electrolytes and ongoing supportive care he
fails to settle. Which of the drugs listed below may improve the situation?

Buscopan

Neostigmine

Metoclopramide

Mebevrine

Sodium picosulphate

Neostigmine affects the degradation of acetylcholine and will therefore stimulate


both nicotinic and muscarinic receptors. It may produce symptomatic bradycardia
and should therefore only be administered in a monitored environment. In colonic
pseudo-obstruction it produces generalised colonic contractions and its onset is
usually rapid.

Please rate this question:

 Discuss and give feedback

Next question 

Colonic pseudo-obstruction

Colonic pseudo-obstruction is characterised by the progressive and painless


dilation of the colon. The abdomen may become grossly distended and tympanic.
Unless a complication such as impending bowel necrosis or perforation occurs,
there is usually little pain.
Diagnosis involves excluding a mechanical bowel obstruction with a plain film and
contrast enema. The underlying cause is usually electrolyte imbalance and the
condition will resolve with correction of this and supportive care.
Patients who do not respond to supportive measures should be treated with
attempted colonoscopic decompression and/ or the drug neostigmine. In rare
cases surgery may be required.

Next question 
gathered by dr. elbarky, for free, not intended for profit by anyone elsewhere.

Display my notes on this topic

          

Save my notes

Question stats

A 9.6%
B 39.3%
C 26.4%
D 9.6%
E 15%

39.3% of users answered this question correctly

Search eMRCS

Search term Go

 Open MRCS Part A textbook (../review/textbook.php)

External links

+ Suggest a link

Dashboard

1

Question 34 of 74
gathered by dr. elbarky, for free, not intended for profit by anyone elsewhere.

 

A 73 year old lady is admitted with a brisk rectal bleed. She is otherwise well and
the bleed settles. On examination, her abdomen is soft and non tender. Elective
colonoscopy shows a small erythematous lesion in the right colon, but no other
abnormality. What is the likely cause?

Diverticular bleed

Angiodysplasia

Colonic cancer

Ischaemia

Infective colitis

Angiodysplasia can be difficult to identify and treat. The colonoscopic stigmata


are easily missed by poor bowel preparation.

Please rate this question:

 Discuss and give feedback

Next question 

Lower Gastrointestinal bleeding

Colonic bleeding
This typically presents as bright red or dark red blood per rectum. Colonic bleeding
rarely presents as malaena type stool, this is because blood in the colon has a
powerful laxative effect and is rarely retained long enough for transformation to
occur and because the digestive enzymes present in the small bowel are not
present in the colon. Up to 15% of patients presenting with haemochezia will have
an upper gastrointestinal source of haemorrhage.

As a general rule right sided bleeds tend to present with darker coloured blood
than left sided bleeds. Haemorrhoidal bleeding typically presents as bright red
rectal bleeding that occurs post defecation either onto toilet paper or into the toilet
pan. It is very unusual for haemorrhoids alone to cause any degree of
haemodynamic compromise.

Causes
gathered by dr. elbarky, for free, not intended for profit by anyone elsewhere.

Cause Presenting features

Colitis Bleeding may be brisk in advanced cases, diarrhoea is


commonly present. Abdominal x-ray may show featureless
colon.

Diverticular Acute diverticulitis often is not complicated by major


disease bleeding and diverticular bleeds often occur sporadically.
75% all will cease spontaneously within 24-48 hours.
Bleeding is often dark and of large volume.

Cancer Colonic cancers often bleed and for many patients this may
be the first sign of the disease. Major bleeding from early
lesions is uncommon

Haemorrhoidal Typically bright red bleeding occurring post defecation.


bleeding Although patients may give graphic descriptions bleeding
of sufficient volume to cause haemodynamic compromise
is rare.

Angiodysplasia Apart from bleeding, which may be massive, these


arteriovenous lesions cause little in the way of symptoms.
The right side of the colon is more commonly affected.

Management
Prompt correction of any haemodynamic compromise is required. Unlike
upper gastrointestinal bleeding the first line management is usually
supportive. This is because in the acute setting endoscopy is rarely helpful.
When haemorrhoidal bleeding is suspected a proctosigmoidoscopy is
reasonable as attempts at full colonoscopy are usually time consuming and
often futile.
In the unstable patient the usual procedure would be an angiogram (either
CT or percutaneous), when these are performed during a period of
haemodynamic instability they may show a bleeding point and may be the
only way of identifying a patch of angiodysplasia.
In others who are more stable the standard procedure would be a
colonoscopy in the elective setting. In patients undergoing angiography
attempts can be made to address the lesion in question such as coiling.
Otherwise surgery will be necessary.
In patients with ulcerative colitis who have significant haemorrhage the
standard approach would be a sub total colectomy, particularly if medical
management has already been tried and is not effective.
Indications for surgery
Patients > 60 years
gathered by dr. elbarky, for free, not intended for profit by anyone elsewhere.

Continued bleeding despite endoscopic intervention


Recurrent bleeding
Known cardiovascular disease with poor response to hypotension

Surgery
Selective mesenteric embolisation if life threatening bleeding. This is most helpful
if conducted during a period of relative haemodynamic instability. If all
haemodynamic parameters are normal then the bleeding is most likely to have
stopped and any angiography normal in appearance. In many units a CT angiogram
will replace selective angiography but the same caveats will apply.

If the source of colonic bleeding is unclear; perform a laparotomy, on table colonic


lavage and following this attempt a resection. A blind sub total colectomy is most
unwise, for example bleeding from an small bowel arterio-venous malformation
will not be treated by this manoeuvre.

Summary of Acute Lower GI bleeding recommendations


Consider admission if:
* Over 60 years
* Haemodynamically unstable/profuse PR bleeding
* On aspirin or NSAID
* Significant co morbidity

Management
All patients should have a history and examination, PR and proctoscopy
Colonoscopic haemostasis aimed for in post polypectomy or diverticular
bleeding

References
http://www.sign.ac.uk/guidelines/fulltext/105/index.html

Next question 

Display my notes on this topic

          

Save my notes
Question stats

A 15.6%
gathered by dr. elbarky, for free, not intended for profit by anyone elsewhere.

B 66.9%
C 6.5%
D 4.9%
E 6.1%

66.9% of users answered this question correctly

Search eMRCS

Search term Go

 Open MRCS Part A textbook (../review/textbook.php)

External links

+ Suggest a link

Dashboard

10

Question 35 of 74
gathered by dr. elbarky, for free, not intended for profit by anyone elsewhere.

 

A 23 year old lady has suffered from diarrhoea for 8 months, she has also lost 2 Kg
in weight. At colonoscopy, appearances of melanosis coli are identified and
confirmed on biopsy. What is the most likely cause?

Ischaemic colitis

Salmonella gastroenteritis infection

Laxative abuse

Irritable bowel syndrome

Clostridium difficile infection

This may occur as a result of laxative abuse and consists of lipofuschin laden
macrophages that appear brown.

Please rate this question:

 Discuss and give feedback

Next question 

Diarrhoea

World Health Organisation definitions


Diarrhoea: > 3 loose or watery stool per day
Acute diarrhoea < 14 days
Chronic diarrhoea > 14 days

Acute Diarrhoea

Gastroenteritis May be accompanied by abdominal pain or


nausea/vomiting
Acute Diarrhoea

Diverticulitis Classically causes left lower quadrant pain,


diarrhoea and fever
gathered by dr. elbarky, for free, not intended for profit by anyone elsewhere.

Antibiotic therapy More common with broad spectrum antibiotics


Clostridium difficile is also seen with antibiotic use

Constipation causing A history of alternating diarrhoea and constipation


overflow may be given
May lead to faecal incontinence in the elderly

Chronic
Diarrhoea

Irritable Extremely common. The most consistent features are


bowel abdominal pain, bloating and change in bowel habit. Patients
syndrome may be divided into those with diarrhoea predominant IBS and
those with constipation predominant IBS.
Features such as lethargy, nausea, backache and bladder
symptoms may also be present

Ulcerative Bloody diarrhoea may be seen. Crampy abdominal pain and


colitis weight loss are also common. Faecal urgency and tenesmus
may occur

Crohn's Crampy abdominal pains and diarrhoea. Bloody diarrhoea less


disease common than in ulcerative colitis. Other features include
malabsorption, mouth ulcers perianal disease and intestinal
obstruction

Colorectal Symptoms depend on the site of the lesion but include


cancer diarrhoea, rectal bleeding, anaemia and constitutional
symptoms e.g. Weight loss and anorexia

Coeliac In children may present with failure to thrive, diarrhoea


disease and abdominal distension
In adults lethargy, anaemia, diarrhoea and weight loss
are seen. Other autoimmune conditions may coexist

Other conditions associated with diarrhoea include:


Thyrotoxicosis
Laxative abuse
Appendicitis with pelvic abscess or pelvic appendix
Radiation enteritis
Diagnosis
Stool culture
Abdominal and digital rectal examination
gathered by dr. elbarky, for free, not intended for profit by anyone elsewhere.

Consider colonoscopy (radiological studies unhelpful)


Thyroid function tests, serum calcium, anti endomysial antibodies, glucose

Next question 

Display my notes on this topic

          

Save my notes

Question stats

A 10.6%
B 8%
C 64.1%
D 9.8%
E 7.5%

64.1% of users answered this question correctly

Search eMRCS

Search term Go

 Open MRCS Part A textbook (../review/textbook.php)

External links

+ Suggest a link

Question 36 of 74
gathered by dr. elbarky, for free, not intended for profit by anyone elsewhere.

 

A 20 year old man is admitted with bloody diarrhoea. He has been passing 10
stools per day, Hb-8.0, albumin-20. Stool culture negative. Evidence of colitis on
endoscopy. He has been on intravenous steroids for 5 days and has now
developed megacolon. His haemoglobin is falling and inflammatory markers are
static. What is the most appropriate course of action?

Double the steroid dose

Undertake a loop ileostomy

Undertake a sub total colectomy and end ileostomy

Undertake a sub total colectomy and ileo-rectal anastomosis

Undertake a pan proctocolectomy

Pan proctocolectomy should not be performed in acute unwell patients


Loop ileostomy is not conventional management of UC

This man requires a sub total colectomy. Conservative management has failed.
Patients with ulcerative colitis should undergo colectomy if there is no significant
improvement in 5-7 days after initiating medical therapy if they have a severe
attack of the disease.

Please rate this question:

 Discuss and give feedback

Next question 

IBD

Ulcerative colitis Vs Crohns


Crohn's disease Ulcerative colitis

Distribution Mouth to anus Rectum and colon


gathered by dr. elbarky, for free, not intended for profit by anyone elsewhere.

Macroscopic Cobblestone appearance, Contact bleeding


changes apthoid ulceration

Depth of Transmural inflammation Superficial inflammation


disease

Distribution Patchy Continuous


pattern

Histological Granulomas (non caseating Crypt abscesses,


features epithelioid cell aggregates with Inflammatory cells in the
Langerhans' giant cells) lamina propria

Surgical treatment

Ulcerative colitis
In UC the main place for surgery is when medical treatment has failed, in the
emergency setting this will be a sub total colectomy, end ileostomy and a mucous
fistula. Electively it will be a pan proctocolectomy, an ileoanal pouch may be a
selected option for some. Remember that longstanding UC increases colorectal
cancer risk.

(https://d2zgo9qer4wjf4.cloudfront.net/images_eMRCS/swb042b.jpg)
Image sourced from Wikipedia
(https://d2zgo9qer4wjf4.cloudfront.net
(http://en.wikipedia.org/wiki/Ulcerative
/images_eMRCS/swb042b.jpg)
colitis)
gathered by dr. elbarky, for free, not intended for profit by anyone elsewhere.

Crohn's disease
Unlike UC Crohn's patients need to avoid surgeons, minimal resections are the rule.
They should not have ileoanal pouches as they will do poorly with them.
Management of Crohn's ano rectal sepsis is with a minimal approach, simply drain
sepsis and use setons to facilitate drainage. Definitive fistula surgery should be
avoided.

(https://d2zgo9qer4wjf4.cloudfront.net/images_eMRCS/swb043b.jpg)
Image sourced from Wikipedia (https://d2zgo9qer4wjf4.cloudfront.net
(http://en.wikipedia.org/wiki/Crohn) /images_eMRCS/swb043b.jpg)

Next question 

Display my notes on this topic

          

Save my notes
Question stats

A 7%
gathered by dr. elbarky, for free, not intended for profit by anyone elsewhere.

B 10.1%
C 45.2%
D 8.6%
E 29.2%

45.2% of users answered this question correctly

Search eMRCS

Search term Go

 Open MRCS Part A textbook (../review/textbook.php)

External links

+ Suggest a link

Dashboard

10

Question 37 of 74
gathered by dr. elbarky, for free, not intended for profit by anyone elsewhere.

 

A 31 year old male presents with recurrent episodes of knife like pain within his
rectum. On examination, there is no abnormality to find on either proctoscopy or
palpation. What is the most likely diagnosis?

Proctalgia fugax

Fissure in ano

Fistula in ano

Anal cancer

Intersphincteric abscess

Proctalgia fugax is a functional anorectal disorder characterized by severe,


intermittent episodes of rectal pain that are self-limited. The diagnosis of
proctalgia fugax requires exclusion of other causes of rectal or anal pain.
Diagnostic work up should also include imaging (for example with MRI) to exclude
occult pathology.

Please rate this question:

 Discuss and give feedback

Next question 

Ano rectal disease

Location: 3, 7, 11 o'clock position


Haemorrhoids Internal or external
Treatment: Conservative, Rubber band ligation,
Haemorrhoidectomy
Fissure in ano Location: midline 6 (posterior midline 90%) and 12 o'clock
position. Distal to the dentate line
Chronic fissure > 6/52: triad: Ulcer, sentinel pile, enlarged
anal papillae
gathered by dr. elbarky, for free, not intended for profit by anyone elsewhere.

Proctitis Causes: Crohn's, ulcerative colitis, Clostridium difficile

Ano rectal E.coli, staph aureus


abscess Positions: Perianal, Ischiorectal, Pelvirectal, Intersphincteric

Anal fistula Usually due to previous ano-rectal abscess


Intersphincteric, transsphincteric, suprasphincteric, and
extrasphincteric. Goodsalls rule determines location

Rectal prolapse Associated with childbirth and rectal intussceception. May


be internal or external

Pruritus ani Systemic and local causes

Anal neoplasm Squamous cell carcinoma commonest unlike


adenocarcinoma in rectum

Solitary rectal Associated with chronic straining and constipation.


ulcer Histology shows mucosal thickening, lamina propria
replaced with collagen and smooth muscle (fibromuscular
obliteration)

Rectal prolapse
Common especially in multiparous women.
May be internal or external.
Internal rectal prolapse can present insidiously.
External prolapse can ulcerate and in long term impair continence.
Diagnostic work up includes colonoscopy, defecating proctogram, ano rectal
manometry studies and if doubt exists an examination under anaesthesia.

Treatments for prolapse


In the acute setting reduce it (covering it with sugar may reduce swelling.
Delormes procedure which excises mucosa and plicates the rectum (high
recurrence rates) may be used for external prolapse.
Altmeirs procedure which resects the colon via the perineal route has lower
recurrence rates but carries the risk of anastamotic leak.
Rectopexy is an abdominal procedure in which the rectum is elevated and
usually supported at the level of the sacral promontory. Post operative
constipation may be reduced by limiting the dissection to the anterior plane
(laparoscopic ventral mesh rectopexy).
Pruritus ani
Extremely common.
Check not secondary to altered bowel habits (e.g. Diarrhoea)
Associated with underlying diseases such as haemorrhoids.
gathered by dr. elbarky, for free, not intended for profit by anyone elsewhere.

Examine to look for causes such as worms.


Proctosigmoidoscopy to identify associated haemorrhoids and exclude
cancer.
Treatment is largely supportive and patients should avoid using perfumed
products around the area.

Fissure in ano
Typically painful PR bleeding (bright red).
Nearly always in the posterior midline.
Usually solitary.

Treatment
Stool softeners.
Topical diltiazem (or GTN).
If topical treatments fail then botulinum toxin should be injected.
If botulinum toxin fails then males should probably undergo lateral internal
sphincterotomy.
Females who do not respond to botulinum toxin should undergo ano rectal
manometry studies and endo anal USS prior to being offered surgery such
as sphincterotomy.

Next question 

Display my notes on this topic

          

Save my notes

Question stats

A 70.5%
B 10.4%
C 5.2%
D 5%
E 8.9%

Question 38 of 74
gathered by dr. elbarky, for free, not intended for profit by anyone elsewhere.

 

A 30 year old lady presents with painful bright red bleeding that occurs post
defecation. Digital rectal examination is too uncomfortable for the patient, perineal
inspection shows a prominent posterior skin tag. What is the best course of
action?

Arrange for removal of the skin tag

Arrange a haemorrhoidectomy

Prescribe topical diltiazem

Injection of 88% aqueous phenol

Arrange a sphincterotomy

The skin tag will be the sentinel pile of a posterior fissure and removal would be
unwise. Fissures should be treated medically in the first instance.

Please rate this question:

 Discuss and give feedback

Next question 

Benign proctology

Condition Features Treatment

Fissure in Painful, bright red rectal Stool softeners, topical


ano bleeding diltiazem or GTN, botulinum
toxin, Sphincterotomy
Haemorroids Painless, bright red rectal Stool softeners, avoid straining,
bleeding occurs following surgery (see below)
defecation and bleeds onto
the toilet paper and into the
gathered by dr. elbarky, for free, not intended for profit by anyone elsewhere.

toilet pan

Fistula in ano May initially present with an Lay open if low, no sphincter
abscess and then involvement or IBD, if complex,
persisting discharge onto high or IBD insert seton and
the perineum, separate consider other options (see
from the anus below)

Peri anal Peri anal swelling and Incision and drainage, leave the
abscess surrounding erythema cavity open to heal by
secondary intention

Pruritus ani Peri anal itching, occasional Avoid scented products, use
mild bleeding (if severe skin wet wipes rather than tissue,
damage) avoidance of scratching, ensure
no underlying faecal
incontinence

Overview of surgical therapies


Haemorroidal disease
The treatment of haemorroids is usually conservative. Acutely thrombosed
haemorroids may be extremely painful. Treatment of this acute condition is usually
conservative and consists of stool softeners, ice compressions and topical GTN or
diltiazem to reduce sphincter spasm. Most cases managed with this approach will
settle over the next 5-7 days. After this period there may be residual skin tags that
merit surgical excision or indeed residual haemorroidal disease that may
necessitate haemorroidectomy.
Patients with more chronic symptoms are managed according to the stage of their
disease, small mild internal haemorroids causing little symptoms are best
managed conservatively. More marked symptoms of bleeding and occasional
prolapse, where the haemorroidal complex is largely internal may benefit from
stapled haemorroidopexy. This procedure excises rectal tissue above the dentate
line and disrupts the haemorroidal blood supply. At the same time the excisional
component of the procedure means that the haemorroids are less prone to
prolapse. Adverse effects of this procedure include urgency, which can affect up to
40% of patients (but settles over 6-12 months) and recurrence. The procedure
does not address skin tags and therefore this procedure is unsuitable if this is the
dominant symptom.
Large haemorroids with a substantial external component may be best managed
with a Milligan Morgan style conventional haemorroidectomy. In this procedure
three haemorroidal cushions are excised, together with their vascular pedicle.
Excision of excessive volumes of tissue may result in anal stenosis. The procedure
is quite painful and most surgeons prescribe metronidazole post operatively as it
decreases post operative pain.

Fissure in ano
Probably the most efficient and definitive treatment for fissure in ano is lateral
gathered by dr. elbarky, for free, not intended for profit by anyone elsewhere.

internal sphincterotomy. The treatment is permanent and nearly all patients will
recover. Up to 30% will develop incontinence to flatus. There are justifiable
concerns about using this procedure in females as pregnancy and pelvic floor
damage together with a sphincterotomy may result in faecal incontinence. The
usual first line therapy is relaxation of the internal sphincter with either GTN or
diltiazem (the latter being better tolerated) applied topically for 6 weeks. Treatment
failures with topical therapy will usually go on to have treatment with botulinum
toxin. This leads to more permanent changes in the sphincter and this may
facilitate healing.
Typical fissures usually present in the posterior midline, multiple or unusually
located fissures should prompt a search for an underlying cause such as
inflammatory bowel disease or internal prolapse.
Refractory cases where the above treatments have failed may be considered for
advancement flaps.

Fistula in ano
The most effective treatment for fistula is laying it open (fistulotomy). When the
fistula is below the sphincter and uncomplicated, this is a reasonable option.
Sphincter involvement and complex underlying disease should be assessed both
surgically and ideally with imaging (either MRI or endoanal USS). Surgery is then
usually staged, in the first instance a draining seton suture may be inserted. This
avoids the development of recurrent sepsis and may allow resolution. In patients
with Crohns disease the seton should be left in situ long term and the patient
managed medically, as in these cases attempts at complex surgical repair nearly
always fail. Fistulas not associated with IBD may be managed by advancement
flaps, instillation of plugs and glue is generally unsuccessful. A newer technique of
ligation of intersphincteric tract (LIFT procedure) is reported to have good results
in selected centres.

Next question 

Display my notes on this topic

          

Save my notes

Question stats
A 7.2%
B 8%
gathered by dr. elbarky, for free, not intended for profit by anyone elsewhere.

C 70.3%
D 6.1%
E 8.4%

70.3% of users answered this question correctly

Search eMRCS

Search term Go

 Open MRCS Part A textbook (../review/textbook.php)

External links

+ Suggest a link

Dashboard

10

11

12

Question 39 of 74
gathered by dr. elbarky, for free, not intended for profit by anyone elsewhere.

 

A patient has an appendicectomy and a 1.2cm carcinoid tumour is identified in the


tip of the appendix. What is the most appropriate management?

Watchful waiting

Discharge

Right hemicolectomy

Limited ileocaecal resection

Radioisotope scan

Individuals with small carcinoids can be discharged (<2cm and limited to the
appendix). Larger tumours should have a radioisotope scan. Where the resection
margin is positive or where the isotope scan suggests lymphatic metastasis a right
hemicolectomy should be performed.

Please rate this question:

 Discuss and give feedback

Next question 

Carcinoid syndrome

Carcinoid tumours secrete serotonin


Originate in neuroendocrine cells mainly in the intestine (midgut-distal
ileum/appendix)
Can occur in the rectum, bronchi
Hormonal symptoms mainly occur when disease spreads outside the bowel

Clinical features
Onset: insidious over many years
Flushing face
Palpitations
Pulmonary valve stenosis and tricuspid regurgitation causing dyspnoea
Asthma
Severe diarrhoea (secretory, persists despite fasting)
gathered by dr. elbarky, for free, not intended for profit by anyone elsewhere.

Investigation
5-HIAA in a 24-hour urine collection
Somatostatin receptor scintigraphy
CT scan
Blood testing for chromogranin A

Treatment
Octreotide
Surgical removal

Next question 

Display my notes on this topic

          

Save my notes

Question stats

A 18.2%
B 25.7%
C 26.9%
D 13.3%
E 15.9%

25.7% of users answered this question correctly

Search eMRCS

Search term Go

Question 40 of 74
gathered by dr. elbarky, for free, not intended for profit by anyone elsewhere.

 

A 53 year old man presents with a full thickness external rectal prolapse. Which of
the following procedures would be the most suitable surgical option?

Rectopexy

Delormes

Altmeirs

Thirsch tape

Abdomino-perineal excision of the rectum

As this man is relatively young and has full thickness prolapse a rectopexy is the
most appropriate procedure. It will give the lowest recurrence rates.

Please rate this question:

 Discuss and give feedback

Next question 

Rectal prolapse

Rectal prolapse may be divided into internal and external prolapse. Patients with
the former condition may have internal intussceception of the rectum and present
with constipation, obstructed defecation and occasionally faecal incontinence.
Patients with external rectal prolapse have a full thickness external protrusion of
the rectum. Risk factors for the condition include multiparity, pelvic floor trauma
and connective tissue disorders.

Diagnosis
External prolapse is usually evident. Internal prolapse may be identified by
defecating proctography and examination under anaesthesia.
Sinister pathology should be excluded with endoscopy

Treatment
Perineal approaches include the Delormes operation, this avoids resection
and is relatively safe but is associated with high recurrence rates. An
Altmeirs operation involves a perineal excision of the sigmoid colon and
rectum, it may be a more effective procedure than a Delormes but carries
gathered by dr. elbarky, for free, not intended for profit by anyone elsewhere.

the risk of anastomotic leak.


Rectopexy - this is an abdominal procedure. The rectum is mobilised and
fixed onto the sacral promontary. A prosthetic mesh may be inserted. The
recurrence rates are low and the procedure is well tolerated (particularly if
performed laparoscopically).
Thirsch tape- this is a largely historical procedure and involves encircling the
rectum with tape or wire. It may be of use in a palliative setting.

Next question 

Display my notes on this topic

          

Save my notes

Question stats

A 55.4%
B 19.3%
C 10.1%
D 7%
E 8.2%

55.4% of users answered this question correctly

Search eMRCS

Search term Go

 Open MRCS Part A textbook (../review/textbook.php)


Question 41 of 74
gathered by dr. elbarky, for free, not intended for profit by anyone elsewhere.

 

A 19 year old man presents with painful rectal bleeding and is found to have an
anal fissure. Which of the following is least associated with this condition?

Leukaemia

Syphilis

Tuberculosis

Sickle cell disease

Crohn's disease

Anal fissures are associated with:

Sexually transmitted diseases (syphilis, HIV)


Inflammatory bowel disease (Crohn's up to 50%)
Leukaemia (25% of patients)
Tuberculosis
Previous anal surgery

Please rate this question:

 Discuss and give feedback

Next question 

Ano rectal disease

Location: 3, 7, 11 o'clock position


Haemorrhoids Internal or external
Treatment: Conservative, Rubber band ligation,
Haemorrhoidectomy
Fissure in ano Location: midline 6 (posterior midline 90%) and 12 o'clock
position. Distal to the dentate line
Chronic fissure > 6/52: triad: Ulcer, sentinel pile, enlarged
anal papillae
gathered by dr. elbarky, for free, not intended for profit by anyone elsewhere.

Proctitis Causes: Crohn's, ulcerative colitis, Clostridium difficile

Ano rectal E.coli, staph aureus


abscess Positions: Perianal, Ischiorectal, Pelvirectal, Intersphincteric

Anal fistula Usually due to previous ano-rectal abscess


Intersphincteric, transsphincteric, suprasphincteric, and
extrasphincteric. Goodsalls rule determines location

Rectal prolapse Associated with childbirth and rectal intussceception. May


be internal or external

Pruritus ani Systemic and local causes

Anal neoplasm Squamous cell carcinoma commonest unlike


adenocarcinoma in rectum

Solitary rectal Associated with chronic straining and constipation.


ulcer Histology shows mucosal thickening, lamina propria
replaced with collagen and smooth muscle (fibromuscular
obliteration)

Rectal prolapse
Common especially in multiparous women.
May be internal or external.
Internal rectal prolapse can present insidiously.
External prolapse can ulcerate and in long term impair continence.
Diagnostic work up includes colonoscopy, defecating proctogram, ano rectal
manometry studies and if doubt exists an examination under anaesthesia.

Treatments for prolapse


In the acute setting reduce it (covering it with sugar may reduce swelling.
Delormes procedure which excises mucosa and plicates the rectum (high
recurrence rates) may be used for external prolapse.
Altmeirs procedure which resects the colon via the perineal route has lower
recurrence rates but carries the risk of anastamotic leak.
Rectopexy is an abdominal procedure in which the rectum is elevated and
usually supported at the level of the sacral promontory. Post operative
constipation may be reduced by limiting the dissection to the anterior plane
(laparoscopic ventral mesh rectopexy).
Pruritus ani
Extremely common.
Check not secondary to altered bowel habits (e.g. Diarrhoea)
Associated with underlying diseases such as haemorrhoids.
gathered by dr. elbarky, for free, not intended for profit by anyone elsewhere.

Examine to look for causes such as worms.


Proctosigmoidoscopy to identify associated haemorrhoids and exclude
cancer.
Treatment is largely supportive and patients should avoid using perfumed
products around the area.

Fissure in ano
Typically painful PR bleeding (bright red).
Nearly always in the posterior midline.
Usually solitary.

Treatment
Stool softeners.
Topical diltiazem (or GTN).
If topical treatments fail then botulinum toxin should be injected.
If botulinum toxin fails then males should probably undergo lateral internal
sphincterotomy.
Females who do not respond to botulinum toxin should undergo ano rectal
manometry studies and endo anal USS prior to being offered surgery such
as sphincterotomy.

Next question 

Display my notes on this topic

          

Save my notes

Question stats

A 23%
B 9.7%
C 20.3%
D 37%
E 10%

Question 42 of 74
gathered by dr. elbarky, for free, not intended for profit by anyone elsewhere.

 

A 34 year old man presents with symptoms attributable to a fistula in ano. He is


examined in the lithotomy position and the external opening of the fistula is
identified in the 7 o'clock position. At which of the following locations is the
internal opening most likely to be identified?

7 o'clock

12 o'clock

9 o'clock

3 o'clock

6 o'clock

Goodsals rule:
Anterior fistulae will tend to have an internal opening opposite the external
opening.
Posterior fistulae will tend to have a curved track that passes towards the
midline.

According to Goodsalls rule the track of a posteriorly sited fistula will track to the
posterior midline (i.e. 6 o'clock)

Please rate this question:

 Discuss and give feedback

Next question 

Fistulas

A fistula is defined as an abnormal connection between two epithelial


surfaces.
There are many types ranging from Branchial fistulae in the neck to entero-
cutaneous fistulae abdominally.
In general surgical practice the abdominal cavity generates the majority and
most of these arise from diverticular disease and Crohn's.
As a general rule all fistulae will resolve spontaneously as long as there is no
gathered by dr. elbarky, for free, not intended for profit by anyone elsewhere.

distal obstruction. This is particularly true of intestinal fistulae.

The four types of fistulae are:

Enterocutaneous
These link the intestine to the skin. They may be high (>500ml) or low output
(<250ml) depending upon source. Duodenal /jejunal fistulae will tend to produce
high volume, electrolyte rich secretions which can lead to severe excoriation of the
skin. Colo-cutaneous fistulae will tend to leak faeculent material. Both fistulae may
result from the spontaneous rupture of an abscess cavity onto the skin (such as
following perianal abscess drainage) or may occur as a result of iatrogenic input.
In some cases it may even be surgically desirable e.g. mucous fistula following
sub total colectomy for colitis.

Suspect if there is excess fluid in the drain.

Enteroenteric or Enterocolic
This is a fistula that involves the large or small intestine. They may originate in a
similar manner to enterocutaneous fistulae. A particular problem with this fistula
type is that bacterial overgrowth may precipitate malabsorption syndromes. This
may be particularly serious in inflammatory bowel disease.

Enterovaginal
Aetiology as above.

Enterovesical
This type of fistula goes to the bladder. These fistulas may result in frequent
urinary tract infections, or the passage of gas from the urethra during urination.

Management
Some rules relating to fistula management:
They will heal provided there is no underlying inflammatory bowel disease
and no distal obstruction, so conservative measures may be the best option
Where there is skin involvement, protect the overlying skin, often using a
well fitted stoma bag- skin damage is difficult to treat
A high output fistula may be rendered more easily managed by the use of
octreotide, this will tend to reduce the volume of pancreatic secretions.
Nutritional complications are common especially with high fistula (e.g. high
jejunal or duodenal) these may necessitate the use of TPN to provide
nutritional support together with the concomitant use of octreotide to
reduce volume and protect skin.
When managing perianal fistulae surgeons should avoid probing the fistula
where acute inflammation is present, this almost always worsens
outcomes.
When perianal fistulae occur secondary to Crohn's disease the best
management option is often to drain acute sepsis and maintain that
drainage through the judicious use of setons whilst medical management is
implemented.
gathered by dr. elbarky, for free, not intended for profit by anyone elsewhere.

Always attempt to delineate the fistula anatomy, for abscesses and fistulae
that have an intra abdominal source the use of barium and CT studies
should show a track. For perianal fistulae surgeons should recall Goodsall's
rule in relation to internal and external openings.

Next question 

Display my notes on this topic

          

Save my notes

Question stats

A 15.8%
B 13.9%
C 13.8%
D 12.6%
E 43.8%

43.8% of users answered this question correctly

Search eMRCS

Search term Go

 Open MRCS Part A textbook (../review/textbook.php)

External links

Question 43 of 74
gathered by dr. elbarky, for free, not intended for profit by anyone elsewhere.

 

A 24 year old woman presents with a long history of obstructed defecation and
chronic constipation. She often strains to open her bowels for long periods and
occasionally notices that she has passed a small amount of blood. On
examination, she has an indurated area located anteriorly approximately 3cm
proximal to the anal verge. What is the most likely diagnosis?

Haemorrhoids

Rectal cancer

Ulcerative colitis

Solitary rectal ulcer syndrome

Fissure in ano

Solitary rectal ulcers are associated with chronic constipation and straining. It will
need to be biopsied to exclude malignancy (the histological appearances are
characteristic). Diagnostic work up should include endoscopy and probably
defecating proctogram and ano-rectal manometry studies.

Please rate this question:

 Discuss and give feedback

Next question 

Rectal bleeding

Rectal bleeding is a common cause for patients to be referred to the surgical clinic.
In the clinical history it is useful to try and localise the anatomical source of the
blood. Bright red blood is usually of rectal anal canal origin, whilst dark red blood is
more suggestive of a proximally sited bleeding source. Blood which has entered
the GI tract from a gastro-duodenal source will typically resemble malaena due to
the effects of the digestive enzymes on the blood itself.

In the table below we give some typical bleeding scenarios together with physical
examination findings and causation.

Cause Type of Features in history Examination findings


bleeding
gathered by dr. elbarky, for free, not intended for profit by anyone elsewhere.

Fissure in Bright red Painful bleeding that Muco-epithelial defect


ano rectal occurs post defecation usually in the midline
bleeding in small volumes. posteriorly (anterior
Usually antecedent fissures more likely to
features of be due to underlying
constipation disease)

Haemorroids Bright red Post defecation Normal colon and


rectal bleeding noted both on rectum. Proctoscopy
bleeding toilet paper and drips may show internal
into pan. May be haemorrhoids. Internal
alteration of bowel haemorrhoids are
habit and history of usually impalpable.
straining. No blood
mixed with stool. No
local pain.

Crohns Bright red Bleeding that is Perineal inspection may


disease or mixed accompanied by other show fissures or
blood symptoms such as fistulae. Proctoscopy
altered bowel habit, may demonstrate
malaise, history of indurated mucosa and
fissures (especially possibly strictures. Skip
anterior) and lesions may be noted at
abscesses. colonoscopy.

Ulcerative Bright red Diarrhoea, weight loss, Proctitis is the most


colitis bleeding nocturnal marked finding. Peri
often incontinence, passage anal disease is usually
mixed of mucous PR. absent. Colonoscopy
with stool will show continuous
mucosal lesion.

Rectal Bright red Alteration of bowel Usually obvious


cancer blood habit. Tenesmus may mucosal abnormality.
mixed be present. Symptoms Lesion may be fixed or
volumes of metastatic disease. mobile depending upon
disease extent.
Surrounding mucosa
often normal, although
polyps may be present.
Image showing a fissure in ano. Typically these are located posteriorly and in the
midline. Fissures at other sites may be associated with underlying disease.
gathered by dr. elbarky, for free, not intended for profit by anyone elsewhere.

(https://d2zgo9qer4wjf4.cloudfront.net/images_eMRCS/swb054b.jpg)
Image sourced from Wikipedia
(https://d2zgo9qer4wjf4.cloudfront.net
(http://en.wikipedia.org/wiki/Anal
/images_eMRCS/swb054b.jpg)
fissure)

Colonoscopic image of internal haemorroids. Note these may often be impalpable.

(https://d2zgo9qer4wjf4.cloudfront.net/images_eMRCS/swb055b.jpg)
Image sourced from Wikipedia
(https://d2zgo9qer4wjf4.cloudfront.net
(http://en.wikipedia.org
/images_eMRCS/swb055b.jpg)
/wiki/Haemorrhoids)
Investigation
All patients presenting with rectal bleeding require digital rectal examination
and procto-sigmoidoscopy as a minimal baseline.
gathered by dr. elbarky, for free, not intended for profit by anyone elsewhere.

Remember that haemorrhoids are typically impalpable and to attribute


bleeding to these in the absence of accurate internal inspection is
unsatisfactory.
In young patients with no other concerning features in the history a carefully
performed sigmoidoscopy that demonstrates clear haemorrhoidal disease
may be sufficient. If clear views cannot be obtained then patients require
bowel preparation with an enema and a flexible sigmoidscopy performed.
In those presenting with features of altered bowel habit or suspicion of
inflammatory bowel disease a colonoscopy is the best test.
Patients with excessive pain who are suspected of having a fissure may
require an examination under general or local anaesthesia.
In young patients with external stigmata of fissure and a compatible history
it is acceptable to treat medically and defer internal examination until the
fissure is healed. If the fissure fails to heal then internal examination
becomes necessary along the lines suggested above to exclude internal
disease.

Special tests
In patients with a malignancy of the rectum the staging investigations
comprise an MRI of the rectum to identify circumferential resection margin
compromise and to identify mesorectal nodal disease. In addition to this CT
scanning of the chest abdomen and pelvis is necessary to stage for more
distant disease. Some centres will still stage the mesorectum with endo
rectal ultrasound but this is becoming far less common.

Patients with fissure in ano who are being considered for surgical
sphincterotomy and are females who have an obstetric history should
probably have ano rectal manometry testing performed together with endo
anal ultrasound. As this service is not universally available it is not
mandatory but in the absence of such information there are continence
issues that may arise following sphincterotomy.

Management

Disease Management

Fissure in ano GTN ointment 0.2% or diltiazem cream applied topically is


the usual first line treatment. Botulinum toxin for those who
fail to respond. Internal sphincterotomy for those who fail
with botox, can be considered earlier in males.
Haemorroids Lifestyle advice, for small internal haemorrhoids can
consider injection sclerotherapy or rubber band ligation. For
external haemorrhoids consider haemorrhoidectomy.
Modern options include HALO procedure and stapled
gathered by dr. elbarky, for free, not intended for profit by anyone elsewhere.

haemorrhoidectomy.

Inflammatory Medical management- although surgery may be needed for


bowel disease fistulating Crohns (setons).

Rectal cancer Anterior resection or abdomino-perineal excision of the


colon and rectum. Total mesorectal excision is now standard
of care. Most resections below the peritoneal reflection will
require defunctioning ileostomy. Most patients will require
preoperative radiotherapy.

Next question 

Display my notes on this topic

          

Save my notes

Question stats

A 20.7%
B 7.9%
C 7.1%
D 46.7%
E 17.6%

46.7% of users answered this question correctly

Search eMRCS

Search term Go

Question 44 of 74
gathered by dr. elbarky, for free, not intended for profit by anyone elsewhere.

 

A 67 year old man has had multiple episodes of fever and left iliac fossa pain.
These have usually resolved with courses of intravenous antibiotics. He is
admitted with a history of increasing constipation and abdominal distension. A
contrast x-ray is performed which shows flow of contrast to the sigmoid colon,
here the contrast flows through a long narrow segment of colon into dilated
proximal bowel. What is the most likely cause?

Diverticular stricture

Malignant stricture

Ischaemic stricture

Volvulus

Crohns stricture

The long history of left iliac fossa pain and development of bowel obstruction
suggests a diverticular stricture. These may contain a malignancy and most will
require resection. Whilst colonic Crohns strictures can occur, they would be quite
rare in this age group, with this history as an isolated finding.

Please rate this question:

 Discuss and give feedback

Next question 

Colonic obstruction

Cause Features Treatment


Cause Features Treatment

Cancer Usually insidious onset Establish diagnosis


History of progressive (e.g. contrast enema/
gathered by dr. elbarky, for free, not intended for profit by anyone elsewhere.

constipation endoscopy)
Systemic features (e.g. Laparotomy and
anaemia) resection, stenting,
Abdominal distension defunctioning
Absence of bowel gas distal colostomy or bypass
to site of obstruction

Diverticular Usually history of previous Once diagnosis


stricture acute diverticulitis established, usually
Long history of altered bowel surgical resection
habit Colonic stenting should
Evidence of diverticulosis on not be performed for
imaging or endoscopy benign disease

Volvulus Twisting of bowel around its Initial treatment is to


mesentery untwist the loop, a
Sigmoid colon affected in flexible sigmoidoscopy
76% cases may be needed
Patients usually present with Those with clinical
abdominal pain, bloating and evidence of ischaemia
constipation should undergo surgery
Examination usually shows Patient with recurrent
asymmetrical distension volvulus should
Plain X-rays usually show undergo resection
massively dilated sigmoid
colon, loss of haustra and U
shape are typical, the loop
may contain fluid levels

Acute colonic Symptoms and signs of large Colonoscopic


pseudo- bowel obstruction with no decompression
obstruction lesion Correct metabolic
Usually associated with disorders
metabolic disorders IV neostigmine
Usually a cut off in the left Surgery
colon (82% cases)
Although abdomen tense
and distended, it is usually
not painful
All patients should undergo
contrast enema (may be
therapeutic)
Next question 
gathered by dr. elbarky, for free, not intended for profit by anyone elsewhere.

Display my notes on this topic

          

Save my notes

Question stats

A 68.2%
B 9.8%
C 6%
D 8.4%
E 7.7%

68.2% of users answered this question correctly

Search eMRCS

Search term Go

 Open MRCS Part A textbook (../review/textbook.php)

External links

+ Suggest a link

Dashboard

3

Question 45 of 74
gathered by dr. elbarky, for free, not intended for profit by anyone elsewhere.

 

A 32 year old man is diagnosed as having a carcinoma of the caecum. On


questioning, his mother developed uterine cancer at the age of 39 and his maternal
uncle died from colonic cancer aged 38. His older brother developed a colonic
cancer with micro satellite instability aged 37. What is the most appropriate
operative treatment?

Limited ileocaecal resection

Right hemicolectomy

Extended right hemicolectomy

Panproctocolectomy

Sub total colectomy

The likely diagnosis is one of a familial cancer syndrome and now that he has
developed a colonic cancer the safest operative strategy is a total colectomy and
end ileostomy.

Please rate this question:

 Discuss and give feedback

Next question 

Polyposis syndromes

Genetic Screening and Associated


Syndrome defect Features management disorders
Genetic Screening and Associated
Syndrome defect Features management disorders

Familial Mutation of Typically over If known to be Gastric


gathered by dr. elbarky, for free, not intended for profit by anyone elsewhere.

adenomatous APC gene 100 colonic at risk then fundal


polyposis (80%) cases, adenomas predictive polyps
dominant Cancer risk of genetic testing (50%).
100% as teenager Duodenal
20% are new Annual flexible polyps 90%.
mutations sigmoidoscopy If severe
from 15 years duodenal
If no polyps polyposis
found then 5 cancer risk
yearly of 30% at 10
colonoscopy years.
started at age Abdominal
20 desmoid
Polyps found = tumours.
resectional
surgery
(resection and
pouch Vs sub
total
colectomy and
IRA)

MYH Biallelic Multiple colonic Once identified Duodenal


associated mutation of polyps resection and polyposis in
polyposis mut Y Later onset ileoanal pouch 30%
human right sided reconstruction Associated
homologue cancers more is with
(MYH) on common than in recommended increased
chromosome FAP Attenuated risk of breast
1p, recessive 100% cancer phenotype - cancer (self
risk by age 60 regular examination)
colonoscopy
Genetic Screening and Associated
Syndrome defect Features management disorders

Peutz STK11 Multiple benign Annual Malignancies


gathered by dr. elbarky, for free, not intended for profit by anyone elsewhere.

-Jeghers (LKB1) intestinal examination at other sites


syndrome mutation on hamartomas Pan intestinal Classical
chromosome Episodic endoscopy pigmentation
19 in some obstruction and every 2-3 years pattern
(but not all) intussceception
cases, Increased risk
dominant of GI cancers
(colorectal
cancer 20%,
gastric 5%)
Increased risk
of breast,
ovarian, cervical
pancreatic and
testicular
cancers

Cowden Mutation of Macrocephaly Targeted Breast


disease PTEN gene Multiple individualised cancer (81%
on intestinal screening risk)
chromosome hamartomas Thyroid
10q22, Multiple cancer and
dominant trichilemmomas non toxic
89% risk of goitre
cancer at any Uterine
site cancer
16% risk of
colorectal
cancer

HNPCC Germline Colo rectal Colonoscopy Extra colonic


(Lynch mutations of cancer 30-70% every 1-2 years cancers
syndrome) DNA Endometrial from age 25
mismatch cancer 30-70% Consideration
repair genes Gastric cancer of prophylactic
5-10% surgery
Scanty colonic Extra colonic
polyps may be surveillance
present recommended
Colonic
tumours likely
to be right sided
and mucinous
Next question 
gathered by dr. elbarky, for free, not intended for profit by anyone elsewhere.

Display my notes on this topic

          

Save my notes

Question stats

A 5.9%
B 16.4%
C 10%
D 54.4%
E 13.3%

54.4% of users answered this question correctly

Search eMRCS

Search term Go

 Open MRCS Part A textbook (../review/textbook.php)

External links

+ Suggest a link

Dashboard

3

Question 46 of 74
gathered by dr. elbarky, for free, not intended for profit by anyone elsewhere.

 

A 78 year old lady is admitted with a lower GI bleed and on investigation with a CT
angiogram is found to have bleeding sigmoid diverticular disease. She is otherwise
well and apart from tachycardia, she is stable. What is the most appropriate
course of action?

Laparotomy and Hartmanns procedure

Laparotomy, sigmoid colectomy and colorectal anastomosis

Colonoscopy and application of endoscopic clips to the area

Conservative management with close observation

Laparotomy and sub total colectomy

Most lower GI bleeds occur secondary to diverticular disease and will settle with
conservative management. Attempts at endoscopic haemostasis are usually
unsuccessful.

Please rate this question:

 Discuss and give feedback

Next question 

Diverticular disease

Diverticular disease is a common surgical problem. It consists of herniation of


colonic mucosa through the muscular wall of the colon. The usual site is between
the taenia coli where vessels pierce the muscle to supply the mucosa. For this
reason, the rectum, which lacks taenia, is often spared.

Symptoms
Altered bowel habit
Bleeding
Abdominal pain
Complications
Diverticulitis
Haemorrhage
gathered by dr. elbarky, for free, not intended for profit by anyone elsewhere.

Development of fistula
Perforation and faecal peritonitis
Perforation and development of abscess
Development of diverticular phlegmon

Diagnosis
Patients presenting in clinic will typically undergo either a colonoscopy, CT
cologram or barium enema as part of their diagnostic work up. All tests can
identify diverticular disease. It can be far more difficult to confidently exclude
cancer, particularly in diverticular strictures.

Acutely unwell surgical patients should be investigated in a systematic way. Plain


abdominal films and an erect chest x-ray will identify perforation. An abdominal CT
scan (not a CT cologram) with oral and intravenous contrast will help to identify
whether acute inflammation is present but also the presence of local
complications such as abscess formation.

Severity Classification- Hinchey

I Para-colonic abscess

II Pelvic abscess

III Purulent peritonitis

IV Faecal peritonitis

Treatment
Increase dietary fibre intake.
Mild attacks of diverticulitis may be managed conservatively with
antibiotics.
Peri colonic abscesses should be drained either surgically or radiologically.
Recurrent episodes of acute diverticulitis requiring hospitalisation are a
relative indication for a segmental resection.
Hinchey IV perforations (generalised faecal peritonitis) will require a
resection and usually a stoma. This group have a very high risk of post
operative complications and usually require HDU admission.

Next question 
Display my notes on this topic

          
gathered by dr. elbarky, for free, not intended for profit by anyone elsewhere.

Save my notes

Question stats

A 17.8%
B 13%
C 13.6%
D 49.1%
E 6.5%

49.1% of users answered this question correctly

Search eMRCS

Search term Go

 Open MRCS Part A textbook (../review/textbook.php)

External links

+ Suggest a link

Dashboard

5

Question 47 of 74
gathered by dr. elbarky, for free, not intended for profit by anyone elsewhere.

 

A 56 year old man is admitted with passage of a large volume of blood per rectum.
On examination, he is tachycardic, his abdomen is soft, although he has marked
dilated veins on his abdominal wall. Proctoscopy reveals large dilated veins with
stigmata of recent haemorrhage. What is the most appropriate treatment?

IV terlipressin

Excisional haemorrhoidectomy

Injection sclerotherapy

Proctectomy

Rectal pack insertion

Rectal varices are a recognised complication of portal hypertension. In the first


instance they can be managed with medical therapy to lower pressure in the portal
venous system. TIPSS may be considered. Whilst band ligation is an option,
attempting to inject these in same way as haemorroids would carry a high risk of
precipitating further haemorrhage.

Please rate this question:

 Discuss and give feedback

Next question 

Lower Gastrointestinal bleeding

Colonic bleeding
This typically presents as bright red or dark red blood per rectum. Colonic bleeding
rarely presents as malaena type stool, this is because blood in the colon has a
powerful laxative effect and is rarely retained long enough for transformation to
occur and because the digestive enzymes present in the small bowel are not
present in the colon. Up to 15% of patients presenting with haemochezia will have
an upper gastrointestinal source of haemorrhage.
As a general rule right sided bleeds tend to present with darker coloured blood
than left sided bleeds. Haemorrhoidal bleeding typically presents as bright red
rectal bleeding that occurs post defecation either onto toilet paper or into the toilet
pan. It is very unusual for haemorrhoids alone to cause any degree of
gathered by dr. elbarky, for free, not intended for profit by anyone elsewhere.

haemodynamic compromise.

Causes

Cause Presenting features

Colitis Bleeding may be brisk in advanced cases, diarrhoea is


commonly present. Abdominal x-ray may show featureless
colon.

Diverticular Acute diverticulitis often is not complicated by major


disease bleeding and diverticular bleeds often occur sporadically.
75% all will cease spontaneously within 24-48 hours.
Bleeding is often dark and of large volume.

Cancer Colonic cancers often bleed and for many patients this may
be the first sign of the disease. Major bleeding from early
lesions is uncommon

Haemorrhoidal Typically bright red bleeding occurring post defecation.


bleeding Although patients may give graphic descriptions bleeding
of sufficient volume to cause haemodynamic compromise
is rare.

Angiodysplasia Apart from bleeding, which may be massive, these


arteriovenous lesions cause little in the way of symptoms.
The right side of the colon is more commonly affected.

Management
Prompt correction of any haemodynamic compromise is required. Unlike
upper gastrointestinal bleeding the first line management is usually
supportive. This is because in the acute setting endoscopy is rarely helpful.
When haemorrhoidal bleeding is suspected a proctosigmoidoscopy is
reasonable as attempts at full colonoscopy are usually time consuming and
often futile.
In the unstable patient the usual procedure would be an angiogram (either
CT or percutaneous), when these are performed during a period of
haemodynamic instability they may show a bleeding point and may be the
only way of identifying a patch of angiodysplasia.
In others who are more stable the standard procedure would be a
colonoscopy in the elective setting. In patients undergoing angiography
attempts can be made to address the lesion in question such as coiling.
Otherwise surgery will be necessary.
In patients with ulcerative colitis who have significant haemorrhage the
standard approach would be a sub total colectomy, particularly if medical
management has already been tried and is not effective.
gathered by dr. elbarky, for free, not intended for profit by anyone elsewhere.

Indications for surgery


Patients > 60 years
Continued bleeding despite endoscopic intervention
Recurrent bleeding
Known cardiovascular disease with poor response to hypotension

Surgery
Selective mesenteric embolisation if life threatening bleeding. This is most helpful
if conducted during a period of relative haemodynamic instability. If all
haemodynamic parameters are normal then the bleeding is most likely to have
stopped and any angiography normal in appearance. In many units a CT angiogram
will replace selective angiography but the same caveats will apply.

If the source of colonic bleeding is unclear; perform a laparotomy, on table colonic


lavage and following this attempt a resection. A blind sub total colectomy is most
unwise, for example bleeding from an small bowel arterio-venous malformation
will not be treated by this manoeuvre.

Summary of Acute Lower GI bleeding recommendations


Consider admission if:
* Over 60 years
* Haemodynamically unstable/profuse PR bleeding
* On aspirin or NSAID
* Significant co morbidity

Management
All patients should have a history and examination, PR and proctoscopy
Colonoscopic haemostasis aimed for in post polypectomy or diverticular
bleeding

References
http://www.sign.ac.uk/guidelines/fulltext/105/index.html

Next question 

Display my notes on this topic

          
Save my notes
gathered by dr. elbarky, for free, not intended for profit by anyone elsewhere.

Question stats

A 55.9%
B 9.4%
C 18.4%
D 6%
E 10.2%

55.9% of users answered this question correctly

Search eMRCS

Search term Go

 Open MRCS Part A textbook (../review/textbook.php)

External links

+ Suggest a link

Dashboard

8

Question 48 of 74
gathered by dr. elbarky, for free, not intended for profit by anyone elsewhere.

 

During a colonoscopy, a patient is found to have a colonic cancer in the caecum


and a 1cm polyp (which looks adenomatous) in the sigmoid colon. What is the
correct management of the sigmoid polyp?

Undertake a snare polypectomy

Leave in situ until the cancer has been resected

Perform a hot biopsy

Perform a cold biopsy

Resect the sigmoid at the same time as the cancer resection

Dysplasia and cancer are not the same disease. All colonic adenomas are
dysplastic. Adenomas greater than 2cm may harbor foci of malignancy within
them. However, many have dysplastic cells only. These do not require
segmental resection.

When a cancer has been identified during endoscopy, it is safest to avoid


undertaking polyp interventions as there is a risk of seeding.
In summary, do NOT remove polyps until after the cancer has been resected.

Please rate this question:

 Discuss and give feedback

Next question 

Colonic polyps

Colonic Polyps
May occur in isolation, or greater numbers as part of the polyposis syndromes. In
FAP greater than 100 polyps are typically present. The risk of malignancy in
association with adenomas is related to size, and is the order of 10% in a 1cm
adenoma. Isolated adenomas seldom give risk of symptoms (unless large and
distal). Distally sited villous lesions may produce mucous and if very large,
electrolyte disturbances may occur.
gathered by dr. elbarky, for free, not intended for profit by anyone elsewhere.

Follow up of colonic polyps

Group Action

Colorectal cancer Colonoscopy 1 year post resection

Large non pedunculated colorectal One off scope at 3 years


polyps (LNPCP), R0 resection

Large non pedunculated colorectal Site check at 2-6 months and then a
polyps (LNPCP) R1 or non en bloc further scope at 12 months
resection

High risk findings at baseline One off surveillance at 3 years


colonoscopy

No high risk findings at baseline No colonoscopic surveillance and invite


colonoscopy participation in NHSBCSP programme
when due

High risk findings


More than 2 premalignant polyps including 1 or more advanced
colorectal polyps
OR
More than 5 pre malignant polyps

Exceptions to guidelines
If patient more than 10 years younger than lower screening age and has polyps but
no high risk findings, consider colonoscopy at 5 or 10 years.

Segmental resection or complete colectomy should be considered when:

1. Incomplete excision of malignant polyp


2. Malignant sessile polyp
3. Malignant pedunculated polyp with submucosal invasion
4. Polyps with poorly differentiated carcinoma
5. Familial polyposis coli
-Screening from teenager up to 40 years by 2 yearly sigmoidoscopy/colonoscopy
-Panproctocolectomy and Ileostomy or Restorative Panproctocolectomy.

Rectal polypoidal lesions may be amenable to trans anal endoscopic microsurgery.


References
Rutter MD et al. British Society of Gastroenterology/Association of Coloproctology
of Great Britain and Ireland/Public Health England post- polypectomy and post-
gathered by dr. elbarky, for free, not intended for profit by anyone elsewhere.

colorectal cancer resection surveillance guidelines. Gut 2019;0:123.

Next question 

Display my notes on this topic

          

Save my notes

Question stats

A 39.8%
B 29.6%
C 8.9%
D 8%
E 13.7%

29.6% of users answered this question correctly

Search eMRCS

Search term Go

 Open MRCS Part A textbook (../review/textbook.php)

External links

+ Suggest a link

Question 49 of 74
gathered by dr. elbarky, for free, not intended for profit by anyone elsewhere.

 

A 28 year old male presents with painful, bright red, rectal bleeding. On
examination he is found to have a posteriorly sited, midline, fissure in ano. What is
the most appropriate treatment?

Topical GTN paste

Sub lingual GTN paste

Anal stretch

Advancement flap

Tailored division of the external anal sphincter

Topical vasodilator therapy is the most commonly utilised treatment for fissure in
ano. Surgical division of the internal anal sphincter is a reasonable treatment
option in a young male. Division of the external sphincter will almost certainly
result in incontinence and is not performed. Anal stretches were associated with a
high rate of external sphincter injuries and have been discontinued for this reason.

Please rate this question:

 Discuss and give feedback

Next question 

Anal fissure
Anal fissures are a common cause of painful, bright red, rectal bleeding.
Most fissures are idiopathic and present as a painful mucocutaneous defect in the
posterior midline (90% cases). Fissures are more likely to be anteriorly located in
females, particularly if they are multiparous. Multiple fissures and those which are
gathered by dr. elbarky, for free, not intended for profit by anyone elsewhere.

located at other sites are more likely to be due to an underlying cause.


Diseases associated with fissure in ano include:
Crohns disease
Tuberculosis
Internal rectal prolapse

Diagnosis
In most cases the defect can be visualised as a posterior midline epithelial defect.
Where symptoms are highly suggestive of the condition and examination findings
are unclear an examination under anaesthesia may be helpful. Atypical disease
presentation should be investigated with colonoscopy and EUA with biopsies of
the area.

Treatment
Stool softeners are important as the hard stools may tear the epithelium and result
in recurrent symptoms. The most effective first line agents are topically applied
GTN (0.2%) or Diltiazem (2%) paste. Side effects of diltiazem are better tolerated.
Resistant cases may benefit from injection of botulinum toxin or lateral internal
sphincterotomy (beware in females). Advancement flaps may be used to treat
resistant cases.
Sphincterotomy produces the best healing rates. It is associated with incontinence
to flatus in up to 10% of patients in the long term.

Next question 

Display my notes on this topic

          

Save my notes

Question stats

A 73.9%
B 6.3%
C 5.9%
D 6.8%

Question 50 of 74
gathered by dr. elbarky, for free, not intended for profit by anyone elsewhere.

 

A 23 year old male presents with bright red rectal bleeding that occurs post
defecation onto the toilet paper. He has been suffering from severe pain
associated with this. On external anal examination there is a skin tag located at the
6 O'clock position. Which of the treatments listed below is most likely to be
helpful?

Topical GTN

Rubber band ligation

Injections of oily phenol

Milligan Morgan haemorrhoidectomy

Lords anal dilatation

Since the most likely diagnosis is a fissure, the correct treatment is topical
nitrates. Haemorrhoidal treatments are not going to be helpful. Whilst a Lords anal
dilation was the traditional treatment, there are few surgeons (and even fewer
patients!) that would advocate a significant anal stretch these days as there are
significant long term continence risks.

Please rate this question:

 Discuss and give feedback

Next question 

Ano rectal disease

Location: 3, 7, 11 o'clock position


Haemorrhoids Internal or external
Treatment: Conservative, Rubber band ligation,
Haemorrhoidectomy
Fissure in ano Location: midline 6 (posterior midline 90%) and 12 o'clock
position. Distal to the dentate line
Chronic fissure > 6/52: triad: Ulcer, sentinel pile, enlarged
anal papillae
gathered by dr. elbarky, for free, not intended for profit by anyone elsewhere.

Proctitis Causes: Crohn's, ulcerative colitis, Clostridium difficile

Ano rectal E.coli, staph aureus


abscess Positions: Perianal, Ischiorectal, Pelvirectal, Intersphincteric

Anal fistula Usually due to previous ano-rectal abscess


Intersphincteric, transsphincteric, suprasphincteric, and
extrasphincteric. Goodsalls rule determines location

Rectal prolapse Associated with childbirth and rectal intussceception. May


be internal or external

Pruritus ani Systemic and local causes

Anal neoplasm Squamous cell carcinoma commonest unlike


adenocarcinoma in rectum

Solitary rectal Associated with chronic straining and constipation.


ulcer Histology shows mucosal thickening, lamina propria
replaced with collagen and smooth muscle (fibromuscular
obliteration)

Rectal prolapse
Common especially in multiparous women.
May be internal or external.
Internal rectal prolapse can present insidiously.
External prolapse can ulcerate and in long term impair continence.
Diagnostic work up includes colonoscopy, defecating proctogram, ano rectal
manometry studies and if doubt exists an examination under anaesthesia.

Treatments for prolapse


In the acute setting reduce it (covering it with sugar may reduce swelling.
Delormes procedure which excises mucosa and plicates the rectum (high
recurrence rates) may be used for external prolapse.
Altmeirs procedure which resects the colon via the perineal route has lower
recurrence rates but carries the risk of anastamotic leak.
Rectopexy is an abdominal procedure in which the rectum is elevated and
usually supported at the level of the sacral promontory. Post operative
constipation may be reduced by limiting the dissection to the anterior plane
(laparoscopic ventral mesh rectopexy).
Pruritus ani
Extremely common.
Check not secondary to altered bowel habits (e.g. Diarrhoea)
Associated with underlying diseases such as haemorrhoids.
gathered by dr. elbarky, for free, not intended for profit by anyone elsewhere.

Examine to look for causes such as worms.


Proctosigmoidoscopy to identify associated haemorrhoids and exclude
cancer.
Treatment is largely supportive and patients should avoid using perfumed
products around the area.

Fissure in ano
Typically painful PR bleeding (bright red).
Nearly always in the posterior midline.
Usually solitary.

Treatment
Stool softeners.
Topical diltiazem (or GTN).
If topical treatments fail then botulinum toxin should be injected.
If botulinum toxin fails then males should probably undergo lateral internal
sphincterotomy.
Females who do not respond to botulinum toxin should undergo ano rectal
manometry studies and endo anal USS prior to being offered surgery such
as sphincterotomy.

Next question 

Display my notes on this topic

          

Save my notes

Question stats

A 67.2%
B 13.1%
C 6.2%
D 7.2%
E 6.2%

Question 51 of 74
gathered by dr. elbarky, for free, not intended for profit by anyone elsewhere.

 

You embark on a laparoscopic appendicectomy and find an inflammatory


appendix mass. There is no free fluid and the patient has no evidence of
peritonitis. Which is the best option?

Convert to a midline laparotomy and perform a limited right


hemicolectomy and end ileostomy

Convert to midline laparotomy and perform and appendicectomy after


taking down the adhesions

Place a drain laparoscopically and administer parenteral antibiotics

Send the patient for CT guided drainage

Wrap omentum around the area and avoid drainage

Attempt conservative management for appendix mass without peritonitis.

Dissection of appendix masses can be associated with a considerable degree of


morbidity, the gains of formally dissecting them over simple drainage and
antibiotics are minimal.
This was initially described as the Ochsner-Sherren regime and was based on the
teachings of Albert Ochsner of Chicago and James Sherren of the London
hospital. The key facts of both methods (which essentially consisted of non
surgical management and careful observation) were combined and published by
Hamilton Bailey in 1930 (Bailey H. The Oschner- Sherren treatment of acute
appendicitis. BMJ 1930 Jan 25; 1(3603): 140143.)

Please rate this question:

 Discuss and give feedback

Next question 

Appendicitis
History
Peri umbilical abdominal pain (visceral stretching of appendix lumen and
appendix is mid gut structure) radiating to the right iliac fossa due to
localised parietal peritoneal inflammation.
gathered by dr. elbarky, for free, not intended for profit by anyone elsewhere.

Vomit once or twice but marked and persistent vomiting is unusual.


Diarrhoea is rare. However, pelvic appendicitis may cause localised rectal
irritation and some loose stools. A pelvic abscess may also cause
diarrhoea.
Mild pyrexia is common - temperature is usually 37.5 -38oC. Higher
temperatures are more typical of conditions like mesenteric adenitis.
Anorexia is very common. It is very unusual for patients with appendicitis to
be hungry.

Examination
Generalised peritonitis if perforation has occurred or localised peritonism.
Retrocaecal appendicitis may have relatively few signs.
Digital rectal examination may reveal boggy sensation if pelvic abscess is
present, or even tenderness with a pelvic appendix.

Diagnosis
Typically raised inflammatory markers coupled with compatible history and
examination findings should be enough to justify appendicectomy.
Urine analysis may show mild leucocytosis but no nitrites.
Ultrasound is useful in females where pelvic organ pathology is suspected.
Although it is not always possible to visualise the appendix on ultrasound,
the presence of free fluid (always pathological in males) should raise
suspicion.

Ultrasound examination may show evidence of luminal obstruction and thickening of


the appendiceal wall as shown below
gathered by dr. elbarky, for free, not intended for profit by anyone elsewhere.

(https://d2zgo9qer4wjf4.cloudfront.net/images_eMRCS/swb061b.jpg)
Image sourced from Wikipedia
(https://d2zgo9qer4wjf4.cloudfront.net
(http://en.wikipedia.org
/images_eMRCS/swb061b.jpg)
/wiki/Appendicitis)

Treatment
Appendicectomy which can be performed via either an open or laparoscopic
approach.
Administration of metronidazole reduces wound infection rates.
Patients with perforated appendicitis require copious abdominal lavage.
Patients without peritonitis who have an appendix mass should receive
broad spectrum antibiotics and consideration given to performing an
interval appendicectomy.
Be wary in the older patients who may have either an underlying caecal
malignancy or perforated sigmoid diverticular disease.

Laparoscopic appendicectomy is becoming increasing popular as demonstrated


below
gathered by dr. elbarky, for free, not intended for profit by anyone elsewhere.

(https://d2zgo9qer4wjf4.cloudfront.net/images_eMRCS/swb062b.jpg)
Image sourced from Wikipedia
(https://d2zgo9qer4wjf4.cloudfront.net
(http://en.wikipedia.org
/images_eMRCS/swb062b.jpg)
/wiki/Appendicitis)

Next question 

Display my notes on this topic

          

Save my notes

Question stats

A 13.9%
B 18.2%
C 46.6%
D 8%
E 13.4%

46.6% of users answered this question correctly

Search eMRCS

Question 52 of 74
gathered by dr. elbarky, for free, not intended for profit by anyone elsewhere.

 

A 53 year old man is due to undergo a right hemicolectomy for a caecal


carcinoma. Which of the following would be usual practice prior to surgery?

Oral carbohydrate loading drink 2 hours pre operatively

Mechanical bowel preparation with oral sodium picosulphate

Mechanical bowel preparation with oral mannitol

Total gut cleansing with oral antibiotics 3 days pre-operatively

Iodine rectal washout pre-operatively

Of the options presented here, only the oral carbohydrate drink would be standard
practice prior to a right sided colonic resection. Whilst some surgeons may
administer phosphate enemas before surgery, total gut clearance confers no
benefit for right sided resections and delays recovery. In contrast, the carbohydrate
loading drink is part of enhanced recovery protocols.

Please rate this question:

 Discuss and give feedback

Next question 

Preparation for surgery

Elective and emergency patients require different preparation.

Elective cases
Consider pre admission clinic to address medical issues.
Blood tests including FBC, U+E, LFT's, Clotting, Group and Save
Urine analysis
Pregnancy test
Sickle cell test
ECG/ Chest x-ray
Exact tests to be performed will depend upon the proposed procedure and patient
fitness.
gathered by dr. elbarky, for free, not intended for profit by anyone elsewhere.

Risk factors for development of deep vein thrombosis should be assessed and a
plan for thromboprophylaxis formulated.

Diabetes
Diabetic patients have greater risk of complications.
Poorly controlled diabetes carries high risk of wound infections.
Patients with diet or tablet controlled diabetes may be managed using a policy of
omitting medication and checking blood glucose levels regularly. Diabetics who
are poorly controlled or who take insulin may require a intravenous sliding scale.
Potassium supplementation should also be given.
Diabetic cases should be operated on first.

Emergency cases
Stabilise and resuscitate where needed.
Consider whether antibiotics are needed and when and how they should be
administered.
Inform blood bank if major procedures planned particularly where coagulopathies
are present at the outset or anticipated (e.g. Ruptured AAA repair)
Don't forget to consent and inform relatives.

Special preparation
Some procedures require special preparation:
Thyroid surgery; vocal cord check.
Parathyroid surgery; consider methylene blue to identify gland.
Sentinel node biopsy; radioactive marker/ patent blue dye.
Surgery involving the thoracic duct; consider administration of cream.
Pheochromocytoma surgery; will need alpha and beta blockade.
Surgery for carcinoid tumours; will need covering with octreotide.
Colorectal cases; bowel preparation (especially left sided surgery)
Thyrotoxicosis; lugols iodine/ medical therapy.

References
Management of adults with diabetes undergoing surgery and elective procedures.
NHS Diabetes. April 2011.

Next question 

Display my notes on this topic

          
Save my notes
gathered by dr. elbarky, for free, not intended for profit by anyone elsewhere.

Question stats

A 60.1%
B 20.3%
C 7.1%
D 7.1%
E 5.5%

60.1% of users answered this question correctly

Search eMRCS

Search term Go

 Open MRCS Part A textbook (../review/textbook.php)

External links

+ Suggest a link

Dashboard

8

Question 53 of 74
gathered by dr. elbarky, for free, not intended for profit by anyone elsewhere.

 

What is the most likely diagnosis in a 17 year old man who presents with painful
bright red ano- rectal bleeding that has been noticed to occur in past 2 weeks?

Fistula in ano

Fissure in ano

External haemorrhoids

Internal haemorrhoids

Ulcerative colitis

Painful rectal bleeding is typically seen with fissure in ano (most will be posterior).
The initial history is often short (as in this case). A fistula is more likely to present
with discharge than just blood. Haemorrhoidal disease bleeding is usually
painless. Although thrombosed haemorrhoids may be painful, they typically occur
in patients with a longer history.

Please rate this question:

 Discuss and give feedback

Next question 

Rectal bleeding

Rectal bleeding is a common cause for patients to be referred to the surgical clinic.
In the clinical history it is useful to try and localise the anatomical source of the
blood. Bright red blood is usually of rectal anal canal origin, whilst dark red blood is
more suggestive of a proximally sited bleeding source. Blood which has entered
the GI tract from a gastro-duodenal source will typically resemble malaena due to
the effects of the digestive enzymes on the blood itself.

In the table below we give some typical bleeding scenarios together with physical
examination findings and causation.
Cause Type of Features in history Examination findings
bleeding

Fissure in Bright red Painful bleeding that Muco-epithelial defect


gathered by dr. elbarky, for free, not intended for profit by anyone elsewhere.

ano rectal occurs post defecation usually in the midline


bleeding in small volumes. posteriorly (anterior
Usually antecedent fissures more likely to
features of be due to underlying
constipation disease)

Haemorroids Bright red Post defecation Normal colon and


rectal bleeding noted both on rectum. Proctoscopy
bleeding toilet paper and drips may show internal
into pan. May be haemorrhoids. Internal
alteration of bowel haemorrhoids are
habit and history of usually impalpable.
straining. No blood
mixed with stool. No
local pain.

Crohns Bright red Bleeding that is Perineal inspection may


disease or mixed accompanied by other show fissures or
blood symptoms such as fistulae. Proctoscopy
altered bowel habit, may demonstrate
malaise, history of indurated mucosa and
fissures (especially possibly strictures. Skip
anterior) and lesions may be noted at
abscesses. colonoscopy.

Ulcerative Bright red Diarrhoea, weight loss, Proctitis is the most


colitis bleeding nocturnal marked finding. Peri
often incontinence, passage anal disease is usually
mixed of mucous PR. absent. Colonoscopy
with stool will show continuous
mucosal lesion.

Rectal Bright red Alteration of bowel Usually obvious


cancer blood habit. Tenesmus may mucosal abnormality.
mixed be present. Symptoms Lesion may be fixed or
volumes of metastatic disease. mobile depending upon
disease extent.
Surrounding mucosa
often normal, although
polyps may be present.

Image showing a fissure in ano. Typically these are located posteriorly and in the
midline. Fissures at other sites may be associated with underlying disease.
gathered by dr. elbarky, for free, not intended for profit by anyone elsewhere.

(https://d2zgo9qer4wjf4.cloudfront.net/images_eMRCS/swb054b.jpg)
Image sourced from Wikipedia
(https://d2zgo9qer4wjf4.cloudfront.net
(http://en.wikipedia.org/wiki/Anal
/images_eMRCS/swb054b.jpg)
fissure)

Colonoscopic image of internal haemorroids. Note these may often be impalpable.

(https://d2zgo9qer4wjf4.cloudfront.net/images_eMRCS/swb055b.jpg)
Image sourced from Wikipedia
(https://d2zgo9qer4wjf4.cloudfront.net
(http://en.wikipedia.org
/images_eMRCS/swb055b.jpg)
/wiki/Haemorrhoids)

Investigation
All patients presenting with rectal bleeding require digital rectal examination
and procto-sigmoidoscopy as a minimal baseline.
Remember that haemorrhoids are typically impalpable and to attribute
bleeding to these in the absence of accurate internal inspection is
gathered by dr. elbarky, for free, not intended for profit by anyone elsewhere.

unsatisfactory.
In young patients with no other concerning features in the history a carefully
performed sigmoidoscopy that demonstrates clear haemorrhoidal disease
may be sufficient. If clear views cannot be obtained then patients require
bowel preparation with an enema and a flexible sigmoidscopy performed.
In those presenting with features of altered bowel habit or suspicion of
inflammatory bowel disease a colonoscopy is the best test.
Patients with excessive pain who are suspected of having a fissure may
require an examination under general or local anaesthesia.
In young patients with external stigmata of fissure and a compatible history
it is acceptable to treat medically and defer internal examination until the
fissure is healed. If the fissure fails to heal then internal examination
becomes necessary along the lines suggested above to exclude internal
disease.

Special tests
In patients with a malignancy of the rectum the staging investigations
comprise an MRI of the rectum to identify circumferential resection margin
compromise and to identify mesorectal nodal disease. In addition to this CT
scanning of the chest abdomen and pelvis is necessary to stage for more
distant disease. Some centres will still stage the mesorectum with endo
rectal ultrasound but this is becoming far less common.

Patients with fissure in ano who are being considered for surgical
sphincterotomy and are females who have an obstetric history should
probably have ano rectal manometry testing performed together with endo
anal ultrasound. As this service is not universally available it is not
mandatory but in the absence of such information there are continence
issues that may arise following sphincterotomy.

Management

Disease Management

Fissure in ano GTN ointment 0.2% or diltiazem cream applied topically is


the usual first line treatment. Botulinum toxin for those who
fail to respond. Internal sphincterotomy for those who fail
with botox, can be considered earlier in males.
Haemorroids Lifestyle advice, for small internal haemorrhoids can
consider injection sclerotherapy or rubber band ligation. For
external haemorrhoids consider haemorrhoidectomy.
Modern options include HALO procedure and stapled
gathered by dr. elbarky, for free, not intended for profit by anyone elsewhere.

haemorrhoidectomy.

Inflammatory Medical management- although surgery may be needed for


bowel disease fistulating Crohns (setons).

Rectal cancer Anterior resection or abdomino-perineal excision of the


colon and rectum. Total mesorectal excision is now standard
of care. Most resections below the peritoneal reflection will
require defunctioning ileostomy. Most patients will require
preoperative radiotherapy.

Next question 

Display my notes on this topic

          

Save my notes

Question stats

A 7.1%
B 65.6%
C 10.5%
D 9.8%
E 6.9%

65.6% of users answered this question correctly

Search eMRCS

Search term Go

Question 54 of 74
gathered by dr. elbarky, for free, not intended for profit by anyone elsewhere.

 

A 25 year old male presents with altered bowel habit. He is known to have familial
polyposis coli. A colonoscopy shows widespread polyps, with high grade dysplasia
in a polyp removed from the rectum. What is the best course of action?

Undertake a pan proctocolectomy

Undertake an abdomino perineal resection of the rectum and sigmoid


colon

Undertake a sub total colectomy

Undertake a left hemicolectomy

Perform sequential colonoscopic polypectomies until all polyps are


resected

Since high grade dysplasia has been found in 1 polyp, the correct course of action
is to remove the entire colon, rectum and anus. An ileo-anal pouch could be offered
should the patient wish. None of the other procedures listed would be acceptable
or safe under any circumstances.

Please rate this question:

 Discuss and give feedback

Next question 

Colonic polyps

Colonic Polyps
May occur in isolation, or greater numbers as part of the polyposis syndromes. In
FAP greater than 100 polyps are typically present. The risk of malignancy in
association with adenomas is related to size, and is the order of 10% in a 1cm
adenoma. Isolated adenomas seldom give risk of symptoms (unless large and
distal). Distally sited villous lesions may produce mucous and if very large,
electrolyte disturbances may occur.
Follow up of colonic polyps

Group Action

Colorectal cancer Colonoscopy 1 year post resection


gathered by dr. elbarky, for free, not intended for profit by anyone elsewhere.

Large non pedunculated colorectal One off scope at 3 years


polyps (LNPCP), R0 resection

Large non pedunculated colorectal Site check at 2-6 months and then a
polyps (LNPCP) R1 or non en bloc further scope at 12 months
resection

High risk findings at baseline One off surveillance at 3 years


colonoscopy

No high risk findings at baseline No colonoscopic surveillance and invite


colonoscopy participation in NHSBCSP programme
when due

High risk findings


More than 2 premalignant polyps including 1 or more advanced
colorectal polyps
OR
More than 5 pre malignant polyps

Exceptions to guidelines
If patient more than 10 years younger than lower screening age and has polyps but
no high risk findings, consider colonoscopy at 5 or 10 years.

Segmental resection or complete colectomy should be considered when:

1. Incomplete excision of malignant polyp


2. Malignant sessile polyp
3. Malignant pedunculated polyp with submucosal invasion
4. Polyps with poorly differentiated carcinoma
5. Familial polyposis coli
-Screening from teenager up to 40 years by 2 yearly sigmoidoscopy/colonoscopy
-Panproctocolectomy and Ileostomy or Restorative Panproctocolectomy.

Rectal polypoidal lesions may be amenable to trans anal endoscopic microsurgery.

References
Rutter MD et al. British Society of Gastroenterology/Association of Coloproctology
of Great Britain and Ireland/Public Health England post- polypectomy and post-
colorectal cancer resection surveillance guidelines. Gut 2019;0:123.

Next question 
gathered by dr. elbarky, for free, not intended for profit by anyone elsewhere.

Display my notes on this topic

          

Save my notes

Question stats

A 66%
B 9%
C 11.5%
D 6.5%
E 7.1%

66% of users answered this question correctly

Search eMRCS

Search term Go

 Open MRCS Part A textbook (../review/textbook.php)

External links

+ Suggest a link

Dashboard

1

Question 55 of 74
gathered by dr. elbarky, for free, not intended for profit by anyone elsewhere.

 

A 75 year old lady is admitted with large bowel obstruction. She is previously well.
She is investigated with an abdominal CT scan and this shows an obstructing
carcinoma of the ascending colon. What is the best course of action?

Laparotomy, right hemicolectomy and ileo-colic anastomosis

Laparotomy and ileo-colic bypass

Laparotomy and loop ileostomy alone

Insertion of self expanding metallic stent

Laparotomy and sub total colectomy

Obstructing right sided cancers can be safely resected by right hemicolectomy. An


abdominal CT scan will provide sufficient information to allow operative planning.
Even if distant disease were present, immediate treatment is still warranted unless
the patient is in the terminal phase of illness. In many cases, a primary
anastomosis can be undertaken. Insertion of colonic stents for right sided
obstructing lesions is not generally undertaken.

Please rate this question:

 Discuss and give feedback

Next question 

Large bowel obstruction

Colonic obstruction remains a common surgical problem. It is most commonly due


to malignancy (60%) and diverticular disease (20%). Volvulus affecting the colon
accounts for 5% of cases. Acute colonic pseudo-obstruction remains a potential
differential diagnosis in all cases. Intussusception affecting the colon (most often
due to tumours in the adult population) remains a rare but recognised cause.
The typical patient will present with gradual onset of progressive abdominal
distension, colicky abdominal pain and either obstipation or absolute constipation.
On examination abdominal distension is present, the presence of caecal
tenderness (assuming no overt evidence of peritonitis) is a useful sign to elicit. A
digital rectal examination and rigid sigmoidoscopy should be performed.
A plain abdominal x-ray is the usual first line test and; the caecal diameter and
ileocaecal valve competency should be assessed on this film.
gathered by dr. elbarky, for free, not intended for profit by anyone elsewhere.

Imaging modalities
Debate long surrounds the use of CT versus gastrograffin enemas. The latter
investigation has always been the traditional method of determining whether a
structural lesion is indeed present. However, in the UK the use of this technique
has declined and in most units a CT scan will be offered as the first line
investigation by the majority of radiologists (and is advocated by the ACPGBI). In
most cases this will provide sufficient detail to allow operative planning, and since
malignancy accounts for most presentations may also stage the disease. In the
event that the radiologist cannot provide a clear statement of lesion site, the
surgeon should have no hesitation in requesting a contrast enema.

Surgical options
The decision as to when to operate or not is determined firstly by the patients
physiological status. Unstable patients require resuscitation prior to surgery and
admission to a critical care unit for invasive monitoring and potential inotropic
support may be needed. In patients who are otherwise stable the decision then
rests on the radiological and clinical findings. As a general rule the old adage that
the sun should not rise and set on unrelieved large bowel obstruction still holds
true. A caecal diameter of 12cm or more in the presence of complete obstruction
with a competent ileocaecal valve and caecal tenderness is a sign of impending
perforation and a relative indication for prompt surgery.

Right sided and transverse lesions


Right sided lesions producing large bowel obstruction should generally be treated
by right hemicolectomy or its extended variant if the lesion lies in the distal
transverse colon or splenic flexure. In these cases an ileocolic anastomosis may
be easily constructed and even in the emergency setting has a low risk of
anastomotic leak.

Left sided lesions


The options here lie between sub total colectomy and anastomosis, left
hemicolectomy with on table lavage and primary anastomosis, left hemicolectomy
and end colostomy formation and finally colonic stent insertion.
The usefulness of colonic stents was the subject of a Cochrane review in 2011.
The authors concluded that on the basis of the data that they reviewed, there was
no benefit from the use of colonic stents over conventional surgical resection with
a tendency to better outcomes seen in the surgical group (1). A more recently
conducted meta analysis met with the same conclusion (2). However, the recently
concluded CREST trial has suggested that self expanding metallic stents can
improve outcomes and avoids a stoma.

Rectosigmoid lesions
Lesions below the peritoneal reflection that are causing obstruction should
generally be treated with a loop colostomy. Primary resection of unstaged rectal
cancer would most likely carry a high CRM positivity rate and cannot be condoned.
Where the lesion occupies the distal sigmoid colon the usual practice would be to
perform a high anterior resection. The decision surrounding restoration of
gathered by dr. elbarky, for free, not intended for profit by anyone elsewhere.

intestinal continuity would lie with the operating surgeon.

References
1. Sagar J. Colorectal stents for the management of malignant colonic
obstructions. Cochrane Database of Systematic Reviews 2011, Issue 11. Art. No.:
CD007378. DOI: 10.1002/14651858.CD007378.pub2.
2. Cirrochi et al Safety and efficacy of endoscopic colonic stenting as a bridge to
surgery in the management of intestinal obstruction due to left colon and rectal
cancer: A systematic review and meta-analysis. Surg Oncol. 2013 Mar;22(1):14-21.

Next question 

Display my notes on this topic

          

Save my notes

Question stats

A 54.5%
B 6.6%
C 21.5%
D 8.8%
E 8.7%

54.5% of users answered this question correctly

Search eMRCS

Search term Go

Question 56 of 74
gathered by dr. elbarky, for free, not intended for profit by anyone elsewhere.

 

What is the commonest type of fistula in ano?

Trans-sphincteric

Supra levator

Complex supra levator

Intersphincteric

Suprasphincteric

Intersphincteric fistulas are the commonest type and the external opening may be
internal or external. These are the classical type of fistula and will have an internal
opening near the anal verge and obey Goodsalls rule. Primary fistulotomy in this
situation usually poses little risk to continence.

Please rate this question:

 Discuss and give feedback

Next question 

Anal fistula

Fistula in ano is the most common form of ano rectal sepsis. Fistulae will have
both an internal opening and external opening, these will be connected by tract(s).
Complexity arises because of the potential for multiple entry and exit sites,
together with multiple tracts. Fistulae are classified into four main groups
according to anatomical location and the degree of sphincter involvement. Simple
uncomplicated fistulae are low and do not involve more than 30% of the external
sphincter. Complex fistulae involve the sphincter, have multiple branches or are
non cryptoglandular in origin[1]

Assessment
Examination of the perineum for signs of trauma, external openings or the
stigmata of IBD is important. Digital rectal examination may reveal the cord linking
the internal and external openings. At the same time the integrity of the sphincter
mechanism can be assessed. Low, uncomplicated fistulas may not require any
further assessment, other groups will usually require more detailed investigation.
For the fistula, the use of endo-anal USS with instillation of hydrogen peroxide into
gathered by dr. elbarky, for free, not intended for profit by anyone elsewhere.

the fistula tract may be helpful. Ano-rectal MRI scanning is also a useful tool, it is
sensitive and specific for the identification of fistula anatomy, branching tracts and
identifying occult sphincter involvement[2].

Identification of the internal opening


Fistulas with an external opening less than 3cm from the anal verge will typically
obey Goodsalls rule (see below).

(https://d2zgo9qer4wjf4.cloudfront.net/images_eMRCS/swb142b.png)
Image sourced from Wikipedia (https://d2zgo9qer4wjf4.cloudfront.net
(http://en.wikipedia.org/wiki/Goodsall) /images_eMRCS/swb142b.png)
Therapies
Seton suture
A seton is a piece of material that is passed through the fistula between the
internal and external openings that allows the drainage of sepsis. This is important
as undrained septic foci may drain along the path of least resistance, which may
result in the development of accessory tracts and openings. Their main use is in
treating complex fistula. Two types of seton are recognised, simple and cutting.
Simple setons lie within the fistula tract and encourage both drainage and fibrosis.
A cutting seton is inserted and the skin incised. The suture is tightened and re-
tightened at regular intervals. This may convert a high fistula to a low fistula. Since
the tissue will scar surrounding the fistula it is hoped that this technique will
minimise incontinence[3]. Unfortunately, a large retrospective review of the
literature related to the use of cutting setons has found that they are associated
with a 12% long term incontinence rate [4]

Fistulotomy
Low fistulas, that are simple should be treated by fistulotomy once the acute
sepsis has been controlled. Fistulotomy (where safe) provides the highest healing
rates [5]. Because fistulotomy is regarded as having a high cure rate, there are
some who prefer to use this technique with more extensive sphincter involvement.
In these patients the fistulotomy is performed as for a low fistula. However, the
muscle that is encountered is then divided and reconstructed with an overlapping
sphincter repair. A price is paid in terms of incontinence with this technique and up
gathered by dr. elbarky, for free, not intended for profit by anyone elsewhere.

to 12.5% of patients who were continent pre-operatively will have issues relating to
continence post procedure[6]. The same group also randomised between
fistulotomy and sphincter reconstruction and ano-rectal advancement flaps for the
treatment of complex cryptoglandular fistulas and reported similar outcomes in
terms of recurrence (>90%) and disturbances to continence (20%)[7].
Other authors have found adverse outcomes following fistulotomy in patients who
have undergone previous surgery, are of female gender or who have high internal
openings [8], in these patients careful assessment of pre-operative sphincter
function should be considered mandatory prior to fistulotomy.

Anal fistula plugs and fibrin glue


The desire to avoid injury to the sphincter complex has led to surgeons using both
fibrin glue and plugs to try and improve fistula healing. Meticulous preparation of
the tract and prior use of a draining seton is likely to improve chances of success.
The use of anal fistula plugs in high transphincteric fistula of cryptoglandular
origin is to be discouraged because of the high incidence of non response in
patients treated with such devices [9]In most patients septic complications are the
reasons for failure [10]. Fibrin glue is a popular option for the treatment of fistula.
There is variability of reported healing rates In some cases initial success rates of
up to 50% healing at six months are reported (in patients with complex cryptogenic
fistula). Of these successes 25% suffer a long term recurrence of fistula [11].
There are, however, no obvious cases of damage to the sphincter complex and the
use of the devices does not appear to adversely impact on subsequent surgical
options.

Ano-rectal advancement flaps


This procedure is primarily directed at high fistulae, and is considered attractive as
a sphincter saving operation. The procedure is performed either with the patient in
the prone jack knife position or in lithotomy (depending upon the site of the
fistula). The dissection is commenced in the sub mucosal plane (which may be
infiltrated with dilute adrenaline solution to ease dissection). The dissection is
continued into healthy proximal tissue. This is brought down and sutured over the
defect.
Follow up of patients with cryptoglandular fistulas treated with advancement flaps
shows a success in up to 80% patients[12-14]. With most recurrences occurring in
the first 6 months following surgery[12]. Continence was affected in some
patients, with up to 10% describing major continence issues post operatively.

Ligation of the intersphincteric tract procedure


In this procedure an incision is made in the intersphincteric groove and the fistula
tract dissected out in this plane and divided. A greater than 90% cure rate within 4
weeks was initially reported[15]. Others have subsequently performed similar
studies on larger numbers of patients with similar success rates.
Fistulotomy at the time of abscess drainage?
A Cochrane review conducted in 2010 suggests that primary fistulotomy for low,
uncomplicated fistula in ano may be safe and associated with better outcomes in
relation to long term chronic sepsis[16]. However, there is a danger that such
gathered by dr. elbarky, for free, not intended for profit by anyone elsewhere.

surgery performed by non specialists may result in a higher complication rate and
therefore the traditional teaching is that primary treatment of acute sepsis is
incision and drainage only. All agree that high/ complex fistulae should never be
subject to primary fistulotomy in the acute setting.

References
1. Parks, A.G., P.H. Gordon, and J.D. Hardcastle, A classification of fistula-in-ano. Br
J Surg, 1976. 63(1): p. 1-12.
2. Lunniss, P.J., et al., Magnetic resonance imaging of fistula-in-ano. Dis Colon
Rectum, 1994. 37(7): p. 708-18.
3. Misra, M.C. and B.M. Kapur, A new non-operative approach to fistula in ano. Br J
Surg, 1988. 75(11): p. 1093-4.
4. Ritchie, R.D., J.M. Sackier, and J.P. Hodde, Incontinence rates after cutting seton
treatment for anal fistula. Colorectal Dis, 2009. 11(6): p. 564-71.
5. Tyler, K.M., C.B. Aarons, and S.M. Sentovich, Successful sphincter-sparing
surgery for all anal fistulas. Dis Colon Rectum, 2007. 50(10): p. 1535-9.
6. Perez, F., et al., Prospective clinical and manometric study of fistulotomy with
primary sphincter reconstruction in the management of recurrent complex fistula-
in-ano. Int J Colorectal Dis, 2006. 21(6): p. 522-6.
7. Perez, F., et al., Randomized clinical and manometric study of advancement flap
versus fistulotomy with sphincter reconstruction in the management of complex
fistula-in-ano. Am J Surg, 2006. 192(1): p. 34-40.
8. Garcia-Aguilar, J., et al., Anal fistula surgery. Factors associated with recurrence
and incontinence. Dis Colon Rectum, 1996. 39(7): p. 723-9.
9. Ortiz, H., et al., Randomized clinical trial of anal fistula plug versus endorectal
advancement flap for the treatment of high cryptoglandular fistula in ano. Br J
Surg, 2009. 96(6): p. 608-12.
10. El-Gazzaz, G., M. Zutshi, and T. Hull, A retrospective review of chronic anal
fistulae treated by anal fistulae plug. Colorectal Dis, 2010. 12(5): p. 442-7.
11. Haim, N., et al., Long-term results of fibrin glue treatment for cryptogenic
perianal fistulas: a multicenter study. Dis Colon Rectum, 2011. 54(10): p. 1279-83.
12. Ortiz, H., et al., Length of follow-up after fistulotomy and fistulectomy
associated with endorectal advancement flap repair for fistula in ano. Br J Surg,
2008. 95(4): p. 484-7.
13. Kodner, I.J., et al., Endorectal advancement flap repair of rectovaginal and other
complicated anorectal fistulas. Surgery, 1993. 114(4): p. 682-9; discussion 689-90.
14. Abbas, M.A., R. Lemus-Rangel, and A. Hamadani, Long-term outcome of
endorectal advancement flap for complex anorectal fistulae. Am Surg, 2008.
74(10): p. 921-4.
15. Rojanasakul, A., et al., Total anal sphincter saving technique for fistula-in-ano;
the ligation of intersphincteric fistula tract. J Med Assoc Thai, 2007. 90(3): p.
581-6.
16. Malik, A.I., R.L. Nelson, and S. Tou, Incision and drainage of perianal abscess
with or without treatment of anal fistula. Cochrane Database Syst Rev, 2010(7): p.

Question 58 of 74
gathered by dr. elbarky, for free, not intended for profit by anyone elsewhere.

 

A 19 year old man presents with diarrhoea and rectal bleeding that has been
present for the past two weeks. In addition, he has noticed that he has had faecal
incontinence at night. What is the most likely cause?

Viral gastroenteritis

Inflammatory bowel disease

Intersphincteric abscess

Irritable bowel syndrome and haemorrhoids

Irritable bowel syndrome and a fissure in ano

Nocturnal diarrhea and incontinence is a key feature in the history and is strongly
suggestive of a diagnosis of IBD. More benign IBS presentations seldom have
nocturnal events or a short history.

Please rate this question:

 Discuss and give feedback

Next question 

Rectal bleeding

Rectal bleeding is a common cause for patients to be referred to the surgical clinic.
In the clinical history it is useful to try and localise the anatomical source of the
blood. Bright red blood is usually of rectal anal canal origin, whilst dark red blood is
more suggestive of a proximally sited bleeding source. Blood which has entered
the GI tract from a gastro-duodenal source will typically resemble malaena due to
the effects of the digestive enzymes on the blood itself.

In the table below we give some typical bleeding scenarios together with physical
examination findings and causation.
Cause Type of Features in history Examination findings
bleeding

Fissure in Bright red Painful bleeding that Muco-epithelial defect


gathered by dr. elbarky, for free, not intended for profit by anyone elsewhere.

ano rectal occurs post defecation usually in the midline


bleeding in small volumes. posteriorly (anterior
Usually antecedent fissures more likely to
features of be due to underlying
constipation disease)

Haemorroids Bright red Post defecation Normal colon and


rectal bleeding noted both on rectum. Proctoscopy
bleeding toilet paper and drips may show internal
into pan. May be haemorrhoids. Internal
alteration of bowel haemorrhoids are
habit and history of usually impalpable.
straining. No blood
mixed with stool. No
local pain.

Crohns Bright red Bleeding that is Perineal inspection may


disease or mixed accompanied by other show fissures or
blood symptoms such as fistulae. Proctoscopy
altered bowel habit, may demonstrate
malaise, history of indurated mucosa and
fissures (especially possibly strictures. Skip
anterior) and lesions may be noted at
abscesses. colonoscopy.

Ulcerative Bright red Diarrhoea, weight loss, Proctitis is the most


colitis bleeding nocturnal marked finding. Peri
often incontinence, passage anal disease is usually
mixed of mucous PR. absent. Colonoscopy
with stool will show continuous
mucosal lesion.

Rectal Bright red Alteration of bowel Usually obvious


cancer blood habit. Tenesmus may mucosal abnormality.
mixed be present. Symptoms Lesion may be fixed or
volumes of metastatic disease. mobile depending upon
disease extent.
Surrounding mucosa
often normal, although
polyps may be present.

Image showing a fissure in ano. Typically these are located posteriorly and in the
midline. Fissures at other sites may be associated with underlying disease.
gathered by dr. elbarky, for free, not intended for profit by anyone elsewhere.

(https://d2zgo9qer4wjf4.cloudfront.net/images_eMRCS/swb054b.jpg)
Image sourced from Wikipedia
(https://d2zgo9qer4wjf4.cloudfront.net
(http://en.wikipedia.org/wiki/Anal
/images_eMRCS/swb054b.jpg)
fissure)

Colonoscopic image of internal haemorroids. Note these may often be impalpable.

(https://d2zgo9qer4wjf4.cloudfront.net/images_eMRCS/swb055b.jpg)
Image sourced from Wikipedia
(https://d2zgo9qer4wjf4.cloudfront.net
(http://en.wikipedia.org
/images_eMRCS/swb055b.jpg)
/wiki/Haemorrhoids)

Investigation
All patients presenting with rectal bleeding require digital rectal examination
and procto-sigmoidoscopy as a minimal baseline.
Remember that haemorrhoids are typically impalpable and to attribute
bleeding to these in the absence of accurate internal inspection is
gathered by dr. elbarky, for free, not intended for profit by anyone elsewhere.

unsatisfactory.
In young patients with no other concerning features in the history a carefully
performed sigmoidoscopy that demonstrates clear haemorrhoidal disease
may be sufficient. If clear views cannot be obtained then patients require
bowel preparation with an enema and a flexible sigmoidscopy performed.
In those presenting with features of altered bowel habit or suspicion of
inflammatory bowel disease a colonoscopy is the best test.
Patients with excessive pain who are suspected of having a fissure may
require an examination under general or local anaesthesia.
In young patients with external stigmata of fissure and a compatible history
it is acceptable to treat medically and defer internal examination until the
fissure is healed. If the fissure fails to heal then internal examination
becomes necessary along the lines suggested above to exclude internal
disease.

Special tests
In patients with a malignancy of the rectum the staging investigations
comprise an MRI of the rectum to identify circumferential resection margin
compromise and to identify mesorectal nodal disease. In addition to this CT
scanning of the chest abdomen and pelvis is necessary to stage for more
distant disease. Some centres will still stage the mesorectum with endo
rectal ultrasound but this is becoming far less common.

Patients with fissure in ano who are being considered for surgical
sphincterotomy and are females who have an obstetric history should
probably have ano rectal manometry testing performed together with endo
anal ultrasound. As this service is not universally available it is not
mandatory but in the absence of such information there are continence
issues that may arise following sphincterotomy.

Management

Disease Management

Fissure in ano GTN ointment 0.2% or diltiazem cream applied topically is


the usual first line treatment. Botulinum toxin for those who
fail to respond. Internal sphincterotomy for those who fail
with botox, can be considered earlier in males.
Haemorroids Lifestyle advice, for small internal haemorrhoids can
consider injection sclerotherapy or rubber band ligation. For
external haemorrhoids consider haemorrhoidectomy.
Modern options include HALO procedure and stapled
gathered by dr. elbarky, for free, not intended for profit by anyone elsewhere.

haemorrhoidectomy.

Inflammatory Medical management- although surgery may be needed for


bowel disease fistulating Crohns (setons).

Rectal cancer Anterior resection or abdomino-perineal excision of the


colon and rectum. Total mesorectal excision is now standard
of care. Most resections below the peritoneal reflection will
require defunctioning ileostomy. Most patients will require
preoperative radiotherapy.

Next question 

Display my notes on this topic

          

Save my notes

Question stats

A 10.5%
B 54.1%
C 13.8%
D 12.7%
E 8.9%

54.1% of users answered this question correctly

Search eMRCS

Search term Go

Question 57 of 74
gathered by dr. elbarky, for free, not intended for profit by anyone elsewhere.

 

A 63 year old man presents with episodic rectal bleeding the blood tends to be
dark in colour and may be mixed with stool. His bowel habit has been erratic since
an abdominal aortic aneurysm repair 6 weeks previously. What is the most likely
cause?

Ischaemic colitis

Diverticulitis

Angiodysplasia

Cancer

Ulcerative colitis

The inferior mesenteric artery may have been ligated and being an arteriopath
collateral flow through the marginal may be imperfect.

Please rate this question:

 Discuss and give feedback

Next question 

Lower Gastrointestinal bleeding

Colonic bleeding
This typically presents as bright red or dark red blood per rectum. Colonic bleeding
rarely presents as malaena type stool, this is because blood in the colon has a
powerful laxative effect and is rarely retained long enough for transformation to
occur and because the digestive enzymes present in the small bowel are not
present in the colon. Up to 15% of patients presenting with haemochezia will have
an upper gastrointestinal source of haemorrhage.

As a general rule right sided bleeds tend to present with darker coloured blood
than left sided bleeds. Haemorrhoidal bleeding typically presents as bright red
rectal bleeding that occurs post defecation either onto toilet paper or into the toilet
pan. It is very unusual for haemorrhoids alone to cause any degree of
haemodynamic compromise.

Causes
gathered by dr. elbarky, for free, not intended for profit by anyone elsewhere.

Cause Presenting features

Colitis Bleeding may be brisk in advanced cases, diarrhoea is


commonly present. Abdominal x-ray may show featureless
colon.

Diverticular Acute diverticulitis often is not complicated by major


disease bleeding and diverticular bleeds often occur sporadically.
75% all will cease spontaneously within 24-48 hours.
Bleeding is often dark and of large volume.

Cancer Colonic cancers often bleed and for many patients this may
be the first sign of the disease. Major bleeding from early
lesions is uncommon

Haemorrhoidal Typically bright red bleeding occurring post defecation.


bleeding Although patients may give graphic descriptions bleeding
of sufficient volume to cause haemodynamic compromise
is rare.

Angiodysplasia Apart from bleeding, which may be massive, these


arteriovenous lesions cause little in the way of symptoms.
The right side of the colon is more commonly affected.

Management
Prompt correction of any haemodynamic compromise is required. Unlike
upper gastrointestinal bleeding the first line management is usually
supportive. This is because in the acute setting endoscopy is rarely helpful.
When haemorrhoidal bleeding is suspected a proctosigmoidoscopy is
reasonable as attempts at full colonoscopy are usually time consuming and
often futile.
In the unstable patient the usual procedure would be an angiogram (either
CT or percutaneous), when these are performed during a period of
haemodynamic instability they may show a bleeding point and may be the
only way of identifying a patch of angiodysplasia.
In others who are more stable the standard procedure would be a
colonoscopy in the elective setting. In patients undergoing angiography
attempts can be made to address the lesion in question such as coiling.
Otherwise surgery will be necessary.
In patients with ulcerative colitis who have significant haemorrhage the
standard approach would be a sub total colectomy, particularly if medical
management has already been tried and is not effective.
Indications for surgery
Patients > 60 years
gathered by dr. elbarky, for free, not intended for profit by anyone elsewhere.

Continued bleeding despite endoscopic intervention


Recurrent bleeding
Known cardiovascular disease with poor response to hypotension

Surgery
Selective mesenteric embolisation if life threatening bleeding. This is most helpful
if conducted during a period of relative haemodynamic instability. If all
haemodynamic parameters are normal then the bleeding is most likely to have
stopped and any angiography normal in appearance. In many units a CT angiogram
will replace selective angiography but the same caveats will apply.

If the source of colonic bleeding is unclear; perform a laparotomy, on table colonic


lavage and following this attempt a resection. A blind sub total colectomy is most
unwise, for example bleeding from an small bowel arterio-venous malformation
will not be treated by this manoeuvre.

Summary of Acute Lower GI bleeding recommendations


Consider admission if:
* Over 60 years
* Haemodynamically unstable/profuse PR bleeding
* On aspirin or NSAID
* Significant co morbidity

Management
All patients should have a history and examination, PR and proctoscopy
Colonoscopic haemostasis aimed for in post polypectomy or diverticular
bleeding

References
http://www.sign.ac.uk/guidelines/fulltext/105/index.html

Next question 

Display my notes on this topic

          

Save my notes
Question stats

A 63.1%
gathered by dr. elbarky, for free, not intended for profit by anyone elsewhere.

B 9.9%
C 14.5%
D 6.3%
E 6.1%

63.1% of users answered this question correctly

Search eMRCS

Search term Go

 Open MRCS Part A textbook (../review/textbook.php)

External links

+ Suggest a link

Dashboard

10

Question 59 of 74
gathered by dr. elbarky, for free, not intended for profit by anyone elsewhere.

 

A 48 year old lady has previously undergone a sigmoid colectomy for carcinoma.
On follow up imaging she is found to have a 3cm foci of metastatic disease in
segment IV of the liver. What is the most appropriate course of action?

Palliative chemotherapy

External beam radiotherapy

Brachytherapy

Surgical resection alone

Chemotherapy followed by surgical resection

Patients with colorectal cancer and liver metastasis can still be treated. They
should be staged with a PET scan in addition to standard staging.

The treatment of colorectal liver metastasis is usually with chemotherapy followed


by surgical resection. Where surgery is performed for liver metastasis with curative
intent, the 5 year survival is 20%. Palliation would generally only be considered if
the patient were frail or widespread disease found on imaging. Radiotherapy is not
part of the treatment of liver metastasis.

Please rate this question:

 Discuss and give feedback

Next question 

Colorectal cancer treatment

Patients diagnosed as having colorectal cancer should be completely staged using


CT of the chest/ abdomen and pelvis. Their entire colon should have been
evaluated with colonoscopy or CT colonography. Patients whose tumours lie below
the peritoneal reflection should have their mesorectum evaluated with MRI.
Once their staging is complete patients should be discussed within a dedicated
colorectal MDT meeting and a treatment plan formulated.
gathered by dr. elbarky, for free, not intended for profit by anyone elsewhere.

Treatment of colonic cancer


Cancer of the colon is nearly always treated with surgery. Stents, surgical bypass
and diversion stomas may all be used as palliative adjuncts. Resectional surgery is
the only option for cure in patients with colon cancer. The procedure is tailored to
the patient and the tumour location. The lymphatic drainage of the colon follows
the arterial supply and therefore most resections are tailored around the resection
of particular lymphatic chains (e.g. ileo-colic pedicle for right sided tumours).
Some patients may have confounding factors that will govern the choice of
procedure, for example a tumour in a patient from a HNPCC family may be better
served with a panproctocolectomy rather than segmental resection. Following
resection the decision has to be made regarding restoration of continuity. For an
anastomosis to heal the key technical factors include; adequate blood supply,
mucosal apposition and no tissue tension. Surrounding sepsis, unstable patients
and inexperienced surgeons may compromise these key principles and in such
circumstances it may be safer to construct an end stoma rather than attempting
an anastomosis.
When a colonic cancer presents with an obstructing lesion; the options are to
either stent it or resect. In modern practice it is unusual to simply defunction a
colonic tumour with a proximal loop stoma. This differs from the situation in the
rectum (see below).
Following resection patients with risk factors for disease recurrence are usually
offered chemotherapy, a combination of 5FU and oxaliplatin is common.

Rectal cancer
The management of rectal cancer is slightly different to that of colonic cancer.
This reflects the rectum's anatomical location and the challenges posed as a
result. Tumours located in the rectum can be surgically resected with either an
anterior resection or an abdomino - perineal resection. The technical aspects
governing the choice between these two procedures can be complex to appreciate
and the main point to appreciate for the MRCS is that involvement of the sphincter
complex or very low tumours require APER. In the rectum a 2cm distal clearance
margin is required and this may also impact on the procedure chosen. In addition
to excision of the rectal tube an integral part of the procedure is a meticulous
dissection of the mesorectal fat and lymph nodes (total mesorectal excision/
TME). In rectal cancer surgery invovlement of the cirumferential resection margin
carries a high risk of disease recurrence. Because the rectum is an extraperitoneal
structure (until you remove it that is!) it is possible to irradiate it, something which
cannot be offered for colonic tumours. This has a major impact in rectal cancer
treatment and many patients will be offered neoadjuvent radiotherapy (both long
and short course) prior to resectional surgery. Patients with T1, 2 and 3 /N0
disease on imaging do not require irradiation and should proceed straight to
surgery. Patients with T4 disease will typically have long course chemo
radiotherapy. Patients presenting with large bowel obstruction from rectal cancer
should not undergo resectional surgery without staging as primary treatment (very
different from colonic cancer). This is because rectal surgery is more technically
demanding, the anastomotic leak rate is higher and the danger of a positive
resection margin in an unstaged patient is high. Therefore patients with
obstructing rectal cancer should have a defunctioning loop colostomy.
gathered by dr. elbarky, for free, not intended for profit by anyone elsewhere.

Summary of procedures
The operations for cancer are segmental resections based on blood supply and
lymphatic drainage. These commonly performed procedures are core knowledge
for the MRCS and should be understood.

Site of Risk of
cancer Type of resection Anastomosis leak

Right colon Right hemicolectomy Ileo-colic Low


<5%

Transverse Extended right hemicolectomy Ileo-colic Low


<5%

Splenic Extended right hemicolectomy Ileo-colic Low


flexure <5%

Splenic Left hemicolectomy Colo-colon 2-5%


flexure

Left colon Left hemicolectomy Colo-colon 2-5%

Sigmoid High anterior resection Colo-rectal 5%


colon

Upper Anterior resection (TME) Colo-rectal 5%


rectum

Low rectum Anterior resection (Low TME) Colo-rectal 10%


(+/- Defunctioning
stoma)

Anal verge Abdomino-perineal excision of None n/a


colon and rectum

In the emergency setting, where the bowel has perforated, the risk of an
anastomotic breakdown is much greater, particularly when the anastomosis is
colon-colon. In this situation, an end colostomy is often safer and can be reversed
later. When resection of the sigmoid colon is performed and an end colostomy is
fashioned the operation is referred to as a Hartmans procedure. Whilst left sided
resections are more risky, ileo-colic anastomoses are relatively safe even in the
emergency setting and do not need to be defunctioned.
References
A review of the diagnosis and management of colorectal cancer and a summary of
the UK National Institute of Clinical Excellence guidelines is provided in:
gathered by dr. elbarky, for free, not intended for profit by anyone elsewhere.

Poston G, et al . Diagnosis and management of colorectal cancer: summary of


NICE guidance. BMJ 2011: 343: d 6751.

Next question 

Display my notes on this topic

          

Save my notes

Question stats

A 11.6%
B 8.4%
C 6.3%
D 13.4%
E 60.4%

60.4% of users answered this question correctly

Search eMRCS

Search term Go

 Open MRCS Part A textbook (../review/textbook.php)

External links

+ Suggest a link

Question 60 of 74
gathered by dr. elbarky, for free, not intended for profit by anyone elsewhere.

 

A 19 year old male presents with bright red rectal bleeding that occurs post
defecation onto the paper and into the pan. Apart from constipation his bowel
habit is normal. Digital rectal examination is normal. What is the most likely cause?

Haemorrhoidal disease

Fissure in ano

Solitary rectal ulcer

Rectal cancer

Crohns disease

Uncomplicated grade 1 or 2 haemorrhoids are usually impalpable

This is likely to be haemorrhoidal disease. A sigmoidoscopy should always be


performed to exclude more sinister pathology.

Please rate this question:

 Discuss and give feedback

Next question 

Ano rectal disease

Location: 3, 7, 11 o'clock position


Haemorrhoids Internal or external
Treatment: Conservative, Rubber band ligation,
Haemorrhoidectomy
Fissure in ano Location: midline 6 (posterior midline 90%) and 12 o'clock
position. Distal to the dentate line
Chronic fissure > 6/52: triad: Ulcer, sentinel pile, enlarged
anal papillae
gathered by dr. elbarky, for free, not intended for profit by anyone elsewhere.

Proctitis Causes: Crohn's, ulcerative colitis, Clostridium difficile

Ano rectal E.coli, staph aureus


abscess Positions: Perianal, Ischiorectal, Pelvirectal, Intersphincteric

Anal fistula Usually due to previous ano-rectal abscess


Intersphincteric, transsphincteric, suprasphincteric, and
extrasphincteric. Goodsalls rule determines location

Rectal prolapse Associated with childbirth and rectal intussceception. May


be internal or external

Pruritus ani Systemic and local causes

Anal neoplasm Squamous cell carcinoma commonest unlike


adenocarcinoma in rectum

Solitary rectal Associated with chronic straining and constipation.


ulcer Histology shows mucosal thickening, lamina propria
replaced with collagen and smooth muscle (fibromuscular
obliteration)

Rectal prolapse
Common especially in multiparous women.
May be internal or external.
Internal rectal prolapse can present insidiously.
External prolapse can ulcerate and in long term impair continence.
Diagnostic work up includes colonoscopy, defecating proctogram, ano rectal
manometry studies and if doubt exists an examination under anaesthesia.

Treatments for prolapse


In the acute setting reduce it (covering it with sugar may reduce swelling.
Delormes procedure which excises mucosa and plicates the rectum (high
recurrence rates) may be used for external prolapse.
Altmeirs procedure which resects the colon via the perineal route has lower
recurrence rates but carries the risk of anastamotic leak.
Rectopexy is an abdominal procedure in which the rectum is elevated and
usually supported at the level of the sacral promontory. Post operative
constipation may be reduced by limiting the dissection to the anterior plane
(laparoscopic ventral mesh rectopexy).
Pruritus ani
Extremely common.
Check not secondary to altered bowel habits (e.g. Diarrhoea)
Associated with underlying diseases such as haemorrhoids.
gathered by dr. elbarky, for free, not intended for profit by anyone elsewhere.

Examine to look for causes such as worms.


Proctosigmoidoscopy to identify associated haemorrhoids and exclude
cancer.
Treatment is largely supportive and patients should avoid using perfumed
products around the area.

Fissure in ano
Typically painful PR bleeding (bright red).
Nearly always in the posterior midline.
Usually solitary.

Treatment
Stool softeners.
Topical diltiazem (or GTN).
If topical treatments fail then botulinum toxin should be injected.
If botulinum toxin fails then males should probably undergo lateral internal
sphincterotomy.
Females who do not respond to botulinum toxin should undergo ano rectal
manometry studies and endo anal USS prior to being offered surgery such
as sphincterotomy.

Next question 

Display my notes on this topic

          

Save my notes

Question stats

A 61.9%
B 16%
C 9.7%
D 5.8%
E 6.6%

Question 61 of 74
gathered by dr. elbarky, for free, not intended for profit by anyone elsewhere.

 

A 75 year old man is admitted with large bowel obstruction and on investigation is
found to have a significant sigmoid diverticular stricture as the underlying cause.
What is the most appropriate treatment?

Colonoscopy and pneumatic dilatation of the stricture

Laparotomy and Hartmanns procedure

Colonoscopy and insertion of self expanding metallic stent

Loop ileostomy

Laparotomy, sigmoid colectomy and colorectal anastomosis

Diverticular strictures have a high complication rate with stent insertion. Where
patients present with large bowel obstruction, the best option is to resect the
affected area. Given the fact that there is underlying colonic obstruction, a primary
anastomosis would be unwise. Diverticular strictures should not be dilated.

Please rate this question:

 Discuss and give feedback

Next question 

Diverticular disease

Diverticular disease is a common surgical problem. It consists of herniation of


colonic mucosa through the muscular wall of the colon. The usual site is between
the taenia coli where vessels pierce the muscle to supply the mucosa. For this
reason, the rectum, which lacks taenia, is often spared.

Symptoms
Altered bowel habit
Bleeding
Abdominal pain
Complications
Diverticulitis
Haemorrhage
gathered by dr. elbarky, for free, not intended for profit by anyone elsewhere.

Development of fistula
Perforation and faecal peritonitis
Perforation and development of abscess
Development of diverticular phlegmon

Diagnosis
Patients presenting in clinic will typically undergo either a colonoscopy, CT
cologram or barium enema as part of their diagnostic work up. All tests can
identify diverticular disease. It can be far more difficult to confidently exclude
cancer, particularly in diverticular strictures.

Acutely unwell surgical patients should be investigated in a systematic way. Plain


abdominal films and an erect chest x-ray will identify perforation. An abdominal CT
scan (not a CT cologram) with oral and intravenous contrast will help to identify
whether acute inflammation is present but also the presence of local
complications such as abscess formation.

Severity Classification- Hinchey

I Para-colonic abscess

II Pelvic abscess

III Purulent peritonitis

IV Faecal peritonitis

Treatment
Increase dietary fibre intake.
Mild attacks of diverticulitis may be managed conservatively with
antibiotics.
Peri colonic abscesses should be drained either surgically or radiologically.
Recurrent episodes of acute diverticulitis requiring hospitalisation are a
relative indication for a segmental resection.
Hinchey IV perforations (generalised faecal peritonitis) will require a
resection and usually a stoma. This group have a very high risk of post
operative complications and usually require HDU admission.

Next question 
Display my notes on this topic

          
gathered by dr. elbarky, for free, not intended for profit by anyone elsewhere.

Save my notes

Question stats

A 10.6%
B 45.8%
C 15.3%
D 10.8%
E 17.5%

45.8% of users answered this question correctly

Search eMRCS

Search term Go

 Open MRCS Part A textbook (../review/textbook.php)

External links

+ Suggest a link

Dashboard

5

Question 62 of 74
gathered by dr. elbarky, for free, not intended for profit by anyone elsewhere.

 

A 19 year old lady has a long standing history of diarrhoea and weight loss. She is
investigated with an upper gastro intestinal endoscopy which is normal. A small
bowel contrast study shows a terminal ileal stricture. A colonoscopy was
performed which was normal but the endoscopist was unable to intubate the
terminal ileum. One week after the colonoscopy she is admitted with small bowel
obstruction. Steroids are administered but despite this she fails to improve. What
is the most appropriate treatment?

Administration of steroids at increased dose

Right hemicolectomy

Small bowel resection

Sub total colectomy

Pan proctocolectomy

Crohns disease commonly affects the terminal ileum and in this case the
ileocaecal valve, this means some form of colonic resection will be needed in
addition to the small bowel resection.

It is likely that this lady has terminal ileal disease. Although first presentation of
Crohns disease is usually managed with IV steroids, these have been trialed here
and failed. A resection will remove the stricturing disease. If proximal small bowel
disease has not been excluded pre-operatively then this must be evaluated during
surgery to exclude other small bowel strictures.

Please rate this question:

 Discuss and give feedback

Next question 

IBD
Ulcerative colitis Vs Crohns

Crohn's disease Ulcerative colitis


gathered by dr. elbarky, for free, not intended for profit by anyone elsewhere.

Distribution Mouth to anus Rectum and colon

Macroscopic Cobblestone appearance, Contact bleeding


changes apthoid ulceration

Depth of Transmural inflammation Superficial inflammation


disease

Distribution Patchy Continuous


pattern

Histological Granulomas (non caseating Crypt abscesses,


features epithelioid cell aggregates with Inflammatory cells in the
Langerhans' giant cells) lamina propria

Surgical treatment

Ulcerative colitis
In UC the main place for surgery is when medical treatment has failed, in the
emergency setting this will be a sub total colectomy, end ileostomy and a mucous
fistula. Electively it will be a pan proctocolectomy, an ileoanal pouch may be a
selected option for some. Remember that longstanding UC increases colorectal
cancer risk.

(https://d2zgo9qer4wjf4.cloudfront.net/images_eMRCS/swb042b.jpg)
Image sourced from Wikipedia
(https://d2zgo9qer4wjf4.cloudfront.net
(http://en.wikipedia.org/wiki/Ulcerative
/images_eMRCS/swb042b.jpg)
colitis)
gathered by dr. elbarky, for free, not intended for profit by anyone elsewhere.

Crohn's disease
Unlike UC Crohn's patients need to avoid surgeons, minimal resections are the rule.
They should not have ileoanal pouches as they will do poorly with them.
Management of Crohn's ano rectal sepsis is with a minimal approach, simply drain
sepsis and use setons to facilitate drainage. Definitive fistula surgery should be
avoided.

(https://d2zgo9qer4wjf4.cloudfront.net/images_eMRCS/swb043b.jpg)
Image sourced from Wikipedia (https://d2zgo9qer4wjf4.cloudfront.net
(http://en.wikipedia.org/wiki/Crohn) /images_eMRCS/swb043b.jpg)

Next question 

Display my notes on this topic

          

Save my notes
Question stats

A 9.5%
gathered by dr. elbarky, for free, not intended for profit by anyone elsewhere.

B 37.1%
C 33%
D 11.2%
E 9.1%

37.1% of users answered this question correctly

Search eMRCS

Search term Go

 Open MRCS Part A textbook (../review/textbook.php)

External links

+ Suggest a link

Dashboard

10

Question 63 of 74
gathered by dr. elbarky, for free, not intended for profit by anyone elsewhere.

 

Which of the following statements in relation to fistula in ano is untrue?

High fistulae are safest treated with a seton insertion

Low fistulae may be laid open

They are typically probed with Lockhart Mummery probes

When discovered during incision and drainage of peri anal abscess; should
always be probed to locate the internal opening

When complicating Crohns disease, may respond to infliximab

Probing fistulae during acute sepsis is associated with a high complication rate
and should not be undertaken routinely.

Please rate this question:

 Discuss and give feedback

Next question 

Fistulas

A fistula is defined as an abnormal connection between two epithelial


surfaces.
There are many types ranging from Branchial fistulae in the neck to entero-
cutaneous fistulae abdominally.
In general surgical practice the abdominal cavity generates the majority and
most of these arise from diverticular disease and Crohn's.
As a general rule all fistulae will resolve spontaneously as long as there is no
distal obstruction. This is particularly true of intestinal fistulae.

The four types of fistulae are:

Enterocutaneous
These link the intestine to the skin. They may be high (>500ml) or low output
(<250ml) depending upon source. Duodenal /jejunal fistulae will tend to produce
high volume, electrolyte rich secretions which can lead to severe excoriation of the
skin. Colo-cutaneous fistulae will tend to leak faeculent material. Both fistulae may
gathered by dr. elbarky, for free, not intended for profit by anyone elsewhere.

result from the spontaneous rupture of an abscess cavity onto the skin (such as
following perianal abscess drainage) or may occur as a result of iatrogenic input.
In some cases it may even be surgically desirable e.g. mucous fistula following
sub total colectomy for colitis.

Suspect if there is excess fluid in the drain.

Enteroenteric or Enterocolic
This is a fistula that involves the large or small intestine. They may originate in a
similar manner to enterocutaneous fistulae. A particular problem with this fistula
type is that bacterial overgrowth may precipitate malabsorption syndromes. This
may be particularly serious in inflammatory bowel disease.

Enterovaginal
Aetiology as above.

Enterovesical
This type of fistula goes to the bladder. These fistulas may result in frequent
urinary tract infections, or the passage of gas from the urethra during urination.

Management
Some rules relating to fistula management:
They will heal provided there is no underlying inflammatory bowel disease
and no distal obstruction, so conservative measures may be the best option
Where there is skin involvement, protect the overlying skin, often using a
well fitted stoma bag- skin damage is difficult to treat
A high output fistula may be rendered more easily managed by the use of
octreotide, this will tend to reduce the volume of pancreatic secretions.
Nutritional complications are common especially with high fistula (e.g. high
jejunal or duodenal) these may necessitate the use of TPN to provide
nutritional support together with the concomitant use of octreotide to
reduce volume and protect skin.
When managing perianal fistulae surgeons should avoid probing the fistula
where acute inflammation is present, this almost always worsens
outcomes.
When perianal fistulae occur secondary to Crohn's disease the best
management option is often to drain acute sepsis and maintain that
drainage through the judicious use of setons whilst medical management is
implemented.
Always attempt to delineate the fistula anatomy, for abscesses and fistulae
that have an intra abdominal source the use of barium and CT studies
should show a track. For perianal fistulae surgeons should recall Goodsall's
rule in relation to internal and external openings.
Next question 
gathered by dr. elbarky, for free, not intended for profit by anyone elsewhere.

Display my notes on this topic

          

Save my notes

Question stats

A 12.7%
B 10.3%
C 15.4%
D 47%
E 14.7%

47% of users answered this question correctly

Search eMRCS

Search term Go

 Open MRCS Part A textbook (../review/textbook.php)

External links

+ Suggest a link

Dashboard

3

Question 64 of 74
gathered by dr. elbarky, for free, not intended for profit by anyone elsewhere.

 

A 24 year old man is identified as having a 5cm carcinoid tumour of the appendix.
Imaging and diagnostic work up does not demonstrate any distant disease. What
is the best course of action?

Appendicectomy

Right hemicolectomy

Resection of the caecal pole

External beam radiotherapy

Observation with imaging

Large carcinoid tumours should be formally resected. In many cases, they will be
identified as an incidental finding. In such cases, it can be difficult to distinguish
between carcinoid tumours and other appendiceal neoplasms.

Please rate this question:

 Discuss and give feedback

Next question 

Carcinoid syndrome

Carcinoid tumours secrete serotonin


Originate in neuroendocrine cells mainly in the intestine (midgut-distal
ileum/appendix)
Can occur in the rectum, bronchi
Hormonal symptoms mainly occur when disease spreads outside the bowel

Clinical features
Onset: insidious over many years
Flushing face
Palpitations
Pulmonary valve stenosis and tricuspid regurgitation causing dyspnoea
Asthma
Severe diarrhoea (secretory, persists despite fasting)
gathered by dr. elbarky, for free, not intended for profit by anyone elsewhere.

Investigation
5-HIAA in a 24-hour urine collection
Somatostatin receptor scintigraphy
CT scan
Blood testing for chromogranin A

Treatment
Octreotide
Surgical removal

Next question 

Display my notes on this topic

          

Save my notes

Question stats

A 24.1%
B 50.8%
C 11.1%
D 7.1%
E 7%

50.8% of users answered this question correctly

Search eMRCS

Search term Go

Question 65 of 74
gathered by dr. elbarky, for free, not intended for profit by anyone elsewhere.

 

A 70 year old female is admitted with a history of passing brown coloured urine
and abdominal distension. Clinically she has features of large bowel obstruction
with central abdominal tenderness. She is maximally tender in the left iliac fossa.
There is no evidence of haemodynamic instability. What is the most appropriate
investigation?

Cystogram

Abdominal X-ray of the kidney, ureters and bladder

Computerised tomogram of the abdomen and pelvis

Flexible sigmoidoscopy

Barium enema

This lady is most likely to have a colovesical fistula complicating diverticular


disease of the sigmoid colon. In addition she may also have developed a
diverticular stricture resulting in large bowel obstruction. A locally advanced
tumour of the sigmoid colon may produce a similar clinical picture. The best
investigation of this acute surgical patient is an abdominal CT scan, this will
demonstrate the site of the disease and also supply regional information such as
organ involvement and other local complications such as a pericolic abscess. A
barium enema would require formal bowel preparation and this is contra indicated
where large bowel obstruction is suspected. A flexible sigmoidoscopy is unlikely to
be helpful and the air insufflated at the time of endoscopy may make the colonic
distension worse. A cystogram would provide only very limited information.

Please rate this question:

 Discuss and give feedback

Next question 

Diverticular disease

Diverticular disease is a common surgical problem. It consists of herniation of


colonic mucosa through the muscular wall of the colon. The usual site is between
the taenia coli where vessels pierce the muscle to supply the mucosa. For this
reason, the rectum, which lacks taenia, is often spared.
gathered by dr. elbarky, for free, not intended for profit by anyone elsewhere.

Symptoms
Altered bowel habit
Bleeding
Abdominal pain

Complications
Diverticulitis
Haemorrhage
Development of fistula
Perforation and faecal peritonitis
Perforation and development of abscess
Development of diverticular phlegmon

Diagnosis
Patients presenting in clinic will typically undergo either a colonoscopy, CT
cologram or barium enema as part of their diagnostic work up. All tests can
identify diverticular disease. It can be far more difficult to confidently exclude
cancer, particularly in diverticular strictures.

Acutely unwell surgical patients should be investigated in a systematic way. Plain


abdominal films and an erect chest x-ray will identify perforation. An abdominal CT
scan (not a CT cologram) with oral and intravenous contrast will help to identify
whether acute inflammation is present but also the presence of local
complications such as abscess formation.

Severity Classification- Hinchey

I Para-colonic abscess

II Pelvic abscess

III Purulent peritonitis

IV Faecal peritonitis

Treatment
Increase dietary fibre intake.
Mild attacks of diverticulitis may be managed conservatively with
antibiotics.
Peri colonic abscesses should be drained either surgically or radiologically.
Recurrent episodes of acute diverticulitis requiring hospitalisation are a
relative indication for a segmental resection.
Hinchey IV perforations (generalised faecal peritonitis) will require a
resection and usually a stoma. This group have a very high risk of post
operative complications and usually require HDU admission.
gathered by dr. elbarky, for free, not intended for profit by anyone elsewhere.

Next question 

Display my notes on this topic

          

Save my notes

Question stats

A 9.5%
B 8.3%
C 62.5%
D 9.9%
E 9.7%

62.5% of users answered this question correctly

Search eMRCS

Search term Go

 Open MRCS Part A textbook (../review/textbook.php)

External links

+ Suggest a link

Dashboard

Question 66 of 74
gathered by dr. elbarky, for free, not intended for profit by anyone elsewhere.

 

A 22 year old man presents with a 6 day history of passage of bloody diarrhoea
with passage of mucous and slime. He is passing an average of 8 to 9 bowel
movements per day. On digital rectal examination there is no discrete abnormality
to feel, but there is some blood stained mucous on the glove. What is the most
likely diagnosis?

Solitary rectal ulcer syndrome

Ulcerative colitis

Irritable bowel syndrome

Rectal cancer

Diverticulitis

The passage of bloody diarrhoea together with mucus and a short history makes
this a likely first presentation of inflammatory bowel disease. A rectal malignancy
in a 22 year old would be a very unlikely event. The history is too short to be
consistent with solitary rectal ulcer.

Please rate this question:

 Discuss and give feedback

Next question 

Rectal bleeding

Rectal bleeding is a common cause for patients to be referred to the surgical clinic.
In the clinical history it is useful to try and localise the anatomical source of the
blood. Bright red blood is usually of rectal anal canal origin, whilst dark red blood is
more suggestive of a proximally sited bleeding source. Blood which has entered
the GI tract from a gastro-duodenal source will typically resemble malaena due to
the effects of the digestive enzymes on the blood itself.

In the table below we give some typical bleeding scenarios together with physical
examination findings and causation.

Cause Type of Features in history Examination findings


bleeding
gathered by dr. elbarky, for free, not intended for profit by anyone elsewhere.

Fissure in Bright red Painful bleeding that Muco-epithelial defect


ano rectal occurs post defecation usually in the midline
bleeding in small volumes. posteriorly (anterior
Usually antecedent fissures more likely to
features of be due to underlying
constipation disease)

Haemorroids Bright red Post defecation Normal colon and


rectal bleeding noted both on rectum. Proctoscopy
bleeding toilet paper and drips may show internal
into pan. May be haemorrhoids. Internal
alteration of bowel haemorrhoids are
habit and history of usually impalpable.
straining. No blood
mixed with stool. No
local pain.

Crohns Bright red Bleeding that is Perineal inspection may


disease or mixed accompanied by other show fissures or
blood symptoms such as fistulae. Proctoscopy
altered bowel habit, may demonstrate
malaise, history of indurated mucosa and
fissures (especially possibly strictures. Skip
anterior) and lesions may be noted at
abscesses. colonoscopy.

Ulcerative Bright red Diarrhoea, weight loss, Proctitis is the most


colitis bleeding nocturnal marked finding. Peri
often incontinence, passage anal disease is usually
mixed of mucous PR. absent. Colonoscopy
with stool will show continuous
mucosal lesion.

Rectal Bright red Alteration of bowel Usually obvious


cancer blood habit. Tenesmus may mucosal abnormality.
mixed be present. Symptoms Lesion may be fixed or
volumes of metastatic disease. mobile depending upon
disease extent.
Surrounding mucosa
often normal, although
polyps may be present.
Image showing a fissure in ano. Typically these are located posteriorly and in the
midline. Fissures at other sites may be associated with underlying disease.
gathered by dr. elbarky, for free, not intended for profit by anyone elsewhere.

(https://d2zgo9qer4wjf4.cloudfront.net/images_eMRCS/swb054b.jpg)
Image sourced from Wikipedia
(https://d2zgo9qer4wjf4.cloudfront.net
(http://en.wikipedia.org/wiki/Anal
/images_eMRCS/swb054b.jpg)
fissure)

Colonoscopic image of internal haemorroids. Note these may often be impalpable.

(https://d2zgo9qer4wjf4.cloudfront.net/images_eMRCS/swb055b.jpg)
Image sourced from Wikipedia
(https://d2zgo9qer4wjf4.cloudfront.net
(http://en.wikipedia.org
/images_eMRCS/swb055b.jpg)
/wiki/Haemorrhoids)
Investigation
All patients presenting with rectal bleeding require digital rectal examination
and procto-sigmoidoscopy as a minimal baseline.
gathered by dr. elbarky, for free, not intended for profit by anyone elsewhere.

Remember that haemorrhoids are typically impalpable and to attribute


bleeding to these in the absence of accurate internal inspection is
unsatisfactory.
In young patients with no other concerning features in the history a carefully
performed sigmoidoscopy that demonstrates clear haemorrhoidal disease
may be sufficient. If clear views cannot be obtained then patients require
bowel preparation with an enema and a flexible sigmoidscopy performed.
In those presenting with features of altered bowel habit or suspicion of
inflammatory bowel disease a colonoscopy is the best test.
Patients with excessive pain who are suspected of having a fissure may
require an examination under general or local anaesthesia.
In young patients with external stigmata of fissure and a compatible history
it is acceptable to treat medically and defer internal examination until the
fissure is healed. If the fissure fails to heal then internal examination
becomes necessary along the lines suggested above to exclude internal
disease.

Special tests
In patients with a malignancy of the rectum the staging investigations
comprise an MRI of the rectum to identify circumferential resection margin
compromise and to identify mesorectal nodal disease. In addition to this CT
scanning of the chest abdomen and pelvis is necessary to stage for more
distant disease. Some centres will still stage the mesorectum with endo
rectal ultrasound but this is becoming far less common.

Patients with fissure in ano who are being considered for surgical
sphincterotomy and are females who have an obstetric history should
probably have ano rectal manometry testing performed together with endo
anal ultrasound. As this service is not universally available it is not
mandatory but in the absence of such information there are continence
issues that may arise following sphincterotomy.

Management

Disease Management

Fissure in ano GTN ointment 0.2% or diltiazem cream applied topically is


the usual first line treatment. Botulinum toxin for those who
fail to respond. Internal sphincterotomy for those who fail
with botox, can be considered earlier in males.
Haemorroids Lifestyle advice, for small internal haemorrhoids can
consider injection sclerotherapy or rubber band ligation. For
external haemorrhoids consider haemorrhoidectomy.
Modern options include HALO procedure and stapled
gathered by dr. elbarky, for free, not intended for profit by anyone elsewhere.

haemorrhoidectomy.

Inflammatory Medical management- although surgery may be needed for


bowel disease fistulating Crohns (setons).

Rectal cancer Anterior resection or abdomino-perineal excision of the


colon and rectum. Total mesorectal excision is now standard
of care. Most resections below the peritoneal reflection will
require defunctioning ileostomy. Most patients will require
preoperative radiotherapy.

Next question 

Display my notes on this topic

          

Save my notes

Question stats

A 11.6%
B 61.9%
C 11.6%
D 5.8%
E 9.2%

61.9% of users answered this question correctly

Search eMRCS

Search term Go

Question 67 of 74
gathered by dr. elbarky, for free, not intended for profit by anyone elsewhere.

 

A 21 year old female presents with a 24 hour history of increasingly severe ano-
rectal pain. On examination, she is febrile and the skin surrounding the anus looks
normal. She did not tolerate an attempted digital rectal examination. What is the
most likely diagnosis?

Fissure in ano

Haemorrhoidal disease

Proctalgia fugax

Solitary rectal ulcer

Intersphincteric abscess

The presence of fever and severe pain makes an abscess more likely than a
fissure. Although fissures may be painful they do not, in themselves, cause fever.
The usual management for this condition is examination of the ano-rectum under
general anaesthesia and drainage of the sepsis.

Please rate this question:

 Discuss and give feedback

Next question 

Ano rectal disease

Location: 3, 7, 11 o'clock position


Haemorrhoids Internal or external
Treatment: Conservative, Rubber band ligation,
Haemorrhoidectomy
Fissure in ano Location: midline 6 (posterior midline 90%) and 12 o'clock
position. Distal to the dentate line
Chronic fissure > 6/52: triad: Ulcer, sentinel pile, enlarged
anal papillae
gathered by dr. elbarky, for free, not intended for profit by anyone elsewhere.

Proctitis Causes: Crohn's, ulcerative colitis, Clostridium difficile

Ano rectal E.coli, staph aureus


abscess Positions: Perianal, Ischiorectal, Pelvirectal, Intersphincteric

Anal fistula Usually due to previous ano-rectal abscess


Intersphincteric, transsphincteric, suprasphincteric, and
extrasphincteric. Goodsalls rule determines location

Rectal prolapse Associated with childbirth and rectal intussceception. May


be internal or external

Pruritus ani Systemic and local causes

Anal neoplasm Squamous cell carcinoma commonest unlike


adenocarcinoma in rectum

Solitary rectal Associated with chronic straining and constipation.


ulcer Histology shows mucosal thickening, lamina propria
replaced with collagen and smooth muscle (fibromuscular
obliteration)

Rectal prolapse
Common especially in multiparous women.
May be internal or external.
Internal rectal prolapse can present insidiously.
External prolapse can ulcerate and in long term impair continence.
Diagnostic work up includes colonoscopy, defecating proctogram, ano rectal
manometry studies and if doubt exists an examination under anaesthesia.

Treatments for prolapse


In the acute setting reduce it (covering it with sugar may reduce swelling.
Delormes procedure which excises mucosa and plicates the rectum (high
recurrence rates) may be used for external prolapse.
Altmeirs procedure which resects the colon via the perineal route has lower
recurrence rates but carries the risk of anastamotic leak.
Rectopexy is an abdominal procedure in which the rectum is elevated and
usually supported at the level of the sacral promontory. Post operative
constipation may be reduced by limiting the dissection to the anterior plane
(laparoscopic ventral mesh rectopexy).
Pruritus ani
Extremely common.
Check not secondary to altered bowel habits (e.g. Diarrhoea)
Associated with underlying diseases such as haemorrhoids.
gathered by dr. elbarky, for free, not intended for profit by anyone elsewhere.

Examine to look for causes such as worms.


Proctosigmoidoscopy to identify associated haemorrhoids and exclude
cancer.
Treatment is largely supportive and patients should avoid using perfumed
products around the area.

Fissure in ano
Typically painful PR bleeding (bright red).
Nearly always in the posterior midline.
Usually solitary.

Treatment
Stool softeners.
Topical diltiazem (or GTN).
If topical treatments fail then botulinum toxin should be injected.
If botulinum toxin fails then males should probably undergo lateral internal
sphincterotomy.
Females who do not respond to botulinum toxin should undergo ano rectal
manometry studies and endo anal USS prior to being offered surgery such
as sphincterotomy.

Next question 

Display my notes on this topic

          

Save my notes

Question stats

A 18.5%
B 6.7%
C 13.4%
D 7.5%
E 54%

Question 68 of 74
gathered by dr. elbarky, for free, not intended for profit by anyone elsewhere.

 

A 62 year old man has previously undergone a left hemicolectomy for carcinoma
of the descending colon. On follow up imaging he is found to have two deposits of
metastatic disease located in the right lobe of his liver. What is the best treatment
strategy?

Chemotherapy alone

Chemotherapy followed by surgical resection

Radiofrequency ablation

Chemoradiotherapy

Palliation

Liver metastasis from colorectal cancer are still potentially curable. Without
resection, survival at 5 years is around 5%. With resection, this figure rises to
around 20%. The best outcomes are seen where chemotherapy is given, followed
by resection. Radiofrequency ablation is an option for those patients who lack the
physiological reserve for surgery. However, there is a higher longer term recurrence
rates with all the non resectional strategies. There is no role for radiotherapy.

Please rate this question:

 Discuss and give feedback

Next question 

Colorectal cancer

Annually, about 150,000 new cases are diagnosed and 50,000 deaths from
the disease
About 75% will have sporadic disease and 25% will have a family history
Colorectal tumours comprise a spectrum of disease ranging from
adenomas through to polyp cancers and frank malignancy.
Polyps may be categorised into: neoplastic polyps, adenomatous polyps and
non neoplastic polyps.
The majority of adenomas are polypoidal lesions, although flat lesions do
occur and may prove to be dysplastic.
Non-neoplastic polyps include hyperplastic, juvenile, hamartomatous,
inflammatory, and lymphoid polyps, which have not generally been thought
gathered by dr. elbarky, for free, not intended for profit by anyone elsewhere.

of as precursors of cancer.
Three characteristics of adenomas that correlate with malignant potential
have been characterised. These include increased size, villous architecture
and dysplasia. For this reason most polyps identified at colonoscopy should
be removed.
The transformation from polyp to cancer is described by the adenoma -
carcinoma sequence and its principles should be appreciated. Essentially
genetic changes accompany the transition from adenoma to carcinoma; key
changes include APC, c-myc, K RAS mutations and p53 deletions.

Next question 

Display my notes on this topic

          

Save my notes

Question stats

A 14.5%
B 46.1%
C 10.7%
D 13.3%
E 15.4%

46.1% of users answered this question correctly

Search eMRCS

Search term Go

Question 69 of 74
gathered by dr. elbarky, for free, not intended for profit by anyone elsewhere.

 

A 21 year old man presents with a 5 week history of painful bright red bleeding that
typically occurs post defecation and is noted on the toilet paper. External
inspection of the anal canal shows a small skin tag a the six o'clock position. The
patient declines internal palpation. What is the most likely underlying diagnosis?

Fissure in ano

Fistula in ano

Haemorrhoidal disease

Solitary rectal ulcer

Internal rectal prolapse

Painful bright red rectal bleeding is usually due to a fissure

The presence of pain and the sentinel tag suggests a posterior fissure in ano.

Please rate this question:

 Discuss and give feedback

Next question 

Anal fissure

Anal fissures are a common cause of painful, bright red, rectal bleeding.
Most fissures are idiopathic and present as a painful mucocutaneous defect in the
posterior midline (90% cases). Fissures are more likely to be anteriorly located in
females, particularly if they are multiparous. Multiple fissures and those which are
located at other sites are more likely to be due to an underlying cause.
Diseases associated with fissure in ano include:
Crohns disease
Tuberculosis
Internal rectal prolapse

Diagnosis
In most cases the defect can be visualised as a posterior midline epithelial defect.
gathered by dr. elbarky, for free, not intended for profit by anyone elsewhere.

Where symptoms are highly suggestive of the condition and examination findings
are unclear an examination under anaesthesia may be helpful. Atypical disease
presentation should be investigated with colonoscopy and EUA with biopsies of
the area.

Treatment
Stool softeners are important as the hard stools may tear the epithelium and result
in recurrent symptoms. The most effective first line agents are topically applied
GTN (0.2%) or Diltiazem (2%) paste. Side effects of diltiazem are better tolerated.
Resistant cases may benefit from injection of botulinum toxin or lateral internal
sphincterotomy (beware in females). Advancement flaps may be used to treat
resistant cases.
Sphincterotomy produces the best healing rates. It is associated with incontinence
to flatus in up to 10% of patients in the long term.

Next question 

Display my notes on this topic

          

Save my notes

Question stats

A 58.1%
B 8.7%
C 19.3%
D 7.8%
E 6%

58.1% of users answered this question correctly

Search eMRCS

Question 70 of 74
gathered by dr. elbarky, for free, not intended for profit by anyone elsewhere.

 

A 78 year old lady from a nursing home is admitted with a 24 hour history of
absolute constipation and abdominal pain. On examination, she has a distended
abdomen with a soft mass in her left iliac fossa. An x-ray is performed which
shows a large dilated loop of bowel in the left iliac fossa which contains a fluid
level. What is the most likely diagnosis?

Caecal volvulus

Sigmoid volvulus

Incarcerated femoral hernia

Diverticular stricture

Malignant colonic stricture

Sigmoid volvulus may present with an asymmetrical mass in an elderly patient. It


may contain a fluid level, visible on plain films. It's very rare for femoral hernia to
cause large bowel obstruction.

Please rate this question:

 Discuss and give feedback

Next question 

Colonic obstruction

Cause Features Treatment


Cause Features Treatment

Cancer Usually insidious onset Establish diagnosis


History of progressive (e.g. contrast enema/
gathered by dr. elbarky, for free, not intended for profit by anyone elsewhere.

constipation endoscopy)
Systemic features (e.g. Laparotomy and
anaemia) resection, stenting,
Abdominal distension defunctioning
Absence of bowel gas distal colostomy or bypass
to site of obstruction

Diverticular Usually history of previous Once diagnosis


stricture acute diverticulitis established, usually
Long history of altered bowel surgical resection
habit Colonic stenting should
Evidence of diverticulosis on not be performed for
imaging or endoscopy benign disease

Volvulus Twisting of bowel around its Initial treatment is to


mesentery untwist the loop, a
Sigmoid colon affected in flexible sigmoidoscopy
76% cases may be needed
Patients usually present with Those with clinical
abdominal pain, bloating and evidence of ischaemia
constipation should undergo surgery
Examination usually shows Patient with recurrent
asymmetrical distension volvulus should
Plain X-rays usually show undergo resection
massively dilated sigmoid
colon, loss of haustra and U
shape are typical, the loop
may contain fluid levels

Acute colonic Symptoms and signs of large Colonoscopic


pseudo- bowel obstruction with no decompression
obstruction lesion Correct metabolic
Usually associated with disorders
metabolic disorders IV neostigmine
Usually a cut off in the left Surgery
colon (82% cases)
Although abdomen tense
and distended, it is usually
not painful
All patients should undergo
contrast enema (may be
therapeutic)
Next question 
gathered by dr. elbarky, for free, not intended for profit by anyone elsewhere.

Display my notes on this topic

          

Save my notes

Question stats

A 9.6%
B 59%
C 9.9%
D 11.4%
E 10.1%

59% of users answered this question correctly

Search eMRCS

Search term Go

 Open MRCS Part A textbook (../review/textbook.php)

External links

+ Suggest a link

Dashboard

3

Question 71 of 74
gathered by dr. elbarky, for free, not intended for profit by anyone elsewhere.

 

A 45 year old female is diagnosed as having a carcinoma of the caecum. She


undergoes a CT scan which shows a tumour invading the muscularis propria with
some regional lymphadenopathy. What is the most appropriate initial treatment?

Right hemicolectomy

External beam radiotherapy

Chemotherapy

Combined long course chemo radiotherapy

Referral for palliative care

Right sided colonic cancers should proceed straight to surgery. Radiotherapy to


this area is poorly tolerated and almost never offered as first line treatment. The
decision as to whether or not chemotherapy is given is dependent upon the final
histology.

Please rate this question:

 Discuss and give feedback

Next question 

Colorectal cancer treatment

Patients diagnosed as having colorectal cancer should be completely staged using


CT of the chest/ abdomen and pelvis. Their entire colon should have been
evaluated with colonoscopy or CT colonography. Patients whose tumours lie below
the peritoneal reflection should have their mesorectum evaluated with MRI.

Once their staging is complete patients should be discussed within a dedicated


colorectal MDT meeting and a treatment plan formulated.

Treatment of colonic cancer


Cancer of the colon is nearly always treated with surgery. Stents, surgical bypass
and diversion stomas may all be used as palliative adjuncts. Resectional surgery is
the only option for cure in patients with colon cancer. The procedure is tailored to
the patient and the tumour location. The lymphatic drainage of the colon follows
the arterial supply and therefore most resections are tailored around the resection
gathered by dr. elbarky, for free, not intended for profit by anyone elsewhere.

of particular lymphatic chains (e.g. ileo-colic pedicle for right sided tumours).
Some patients may have confounding factors that will govern the choice of
procedure, for example a tumour in a patient from a HNPCC family may be better
served with a panproctocolectomy rather than segmental resection. Following
resection the decision has to be made regarding restoration of continuity. For an
anastomosis to heal the key technical factors include; adequate blood supply,
mucosal apposition and no tissue tension. Surrounding sepsis, unstable patients
and inexperienced surgeons may compromise these key principles and in such
circumstances it may be safer to construct an end stoma rather than attempting
an anastomosis.
When a colonic cancer presents with an obstructing lesion; the options are to
either stent it or resect. In modern practice it is unusual to simply defunction a
colonic tumour with a proximal loop stoma. This differs from the situation in the
rectum (see below).
Following resection patients with risk factors for disease recurrence are usually
offered chemotherapy, a combination of 5FU and oxaliplatin is common.

Rectal cancer
The management of rectal cancer is slightly different to that of colonic cancer.
This reflects the rectum's anatomical location and the challenges posed as a
result. Tumours located in the rectum can be surgically resected with either an
anterior resection or an abdomino - perineal resection. The technical aspects
governing the choice between these two procedures can be complex to appreciate
and the main point to appreciate for the MRCS is that involvement of the sphincter
complex or very low tumours require APER. In the rectum a 2cm distal clearance
margin is required and this may also impact on the procedure chosen. In addition
to excision of the rectal tube an integral part of the procedure is a meticulous
dissection of the mesorectal fat and lymph nodes (total mesorectal excision/
TME). In rectal cancer surgery invovlement of the cirumferential resection margin
carries a high risk of disease recurrence. Because the rectum is an extraperitoneal
structure (until you remove it that is!) it is possible to irradiate it, something which
cannot be offered for colonic tumours. This has a major impact in rectal cancer
treatment and many patients will be offered neoadjuvent radiotherapy (both long
and short course) prior to resectional surgery. Patients with T1, 2 and 3 /N0
disease on imaging do not require irradiation and should proceed straight to
surgery. Patients with T4 disease will typically have long course chemo
radiotherapy. Patients presenting with large bowel obstruction from rectal cancer
should not undergo resectional surgery without staging as primary treatment (very
different from colonic cancer). This is because rectal surgery is more technically
demanding, the anastomotic leak rate is higher and the danger of a positive
resection margin in an unstaged patient is high. Therefore patients with
obstructing rectal cancer should have a defunctioning loop colostomy.

Summary of procedures
The operations for cancer are segmental resections based on blood supply and
lymphatic drainage. These commonly performed procedures are core knowledge
for the MRCS and should be understood.
gathered by dr. elbarky, for free, not intended for profit by anyone elsewhere.

Site of Risk of
cancer Type of resection Anastomosis leak

Right colon Right hemicolectomy Ileo-colic Low


<5%

Transverse Extended right hemicolectomy Ileo-colic Low


<5%

Splenic Extended right hemicolectomy Ileo-colic Low


flexure <5%

Splenic Left hemicolectomy Colo-colon 2-5%


flexure

Left colon Left hemicolectomy Colo-colon 2-5%

Sigmoid High anterior resection Colo-rectal 5%


colon

Upper Anterior resection (TME) Colo-rectal 5%


rectum

Low rectum Anterior resection (Low TME) Colo-rectal 10%


(+/- Defunctioning
stoma)

Anal verge Abdomino-perineal excision of None n/a


colon and rectum

In the emergency setting, where the bowel has perforated, the risk of an
anastomotic breakdown is much greater, particularly when the anastomosis is
colon-colon. In this situation, an end colostomy is often safer and can be reversed
later. When resection of the sigmoid colon is performed and an end colostomy is
fashioned the operation is referred to as a Hartmans procedure. Whilst left sided
resections are more risky, ileo-colic anastomoses are relatively safe even in the
emergency setting and do not need to be defunctioned.

References
A review of the diagnosis and management of colorectal cancer and a summary of
the UK National Institute of Clinical Excellence guidelines is provided in:
Poston G, et al . Diagnosis and management of colorectal cancer: summary of
NICE guidance. BMJ 2011: 343: d 6751.
Next question 
gathered by dr. elbarky, for free, not intended for profit by anyone elsewhere.

Display my notes on this topic

          

Save my notes

Question stats

A 51.8%
B 9.1%
C 16%
D 15.7%
E 7.4%

51.8% of users answered this question correctly

Search eMRCS

Search term Go

 Open MRCS Part A textbook (../review/textbook.php)

External links

+ Suggest a link

Dashboard

3

Question 72 of 74
gathered by dr. elbarky, for free, not intended for profit by anyone elsewhere.

 

A 25 year old man complains of passing painless bright red blood rectally. It has
been occurring over the past week and tends to occur post defecation. He also
suffers from pruritus ani. The underlying cause is likely to be amenable by
treatment from which of the following modalities?

Topical GTN

Topical diltiazem

Rubber band ligation

Injection sclerotherapy

Lateral internal sphincterotomy

The history of one of the haemorrhoidal bleeding. The recent HUBLE trial showed
equivalence of banding vs HALO for haemorrhoids. Rubber band ligation has a
30% failure rate but is generally easy and well tolerated. Painful PR bleeding is
more suggestive of a fissure which is treated with nitrates or surgery.

Please rate this question:

 Discuss and give feedback

Next question 

Lower Gastrointestinal bleeding

Colonic bleeding
This typically presents as bright red or dark red blood per rectum. Colonic bleeding
rarely presents as malaena type stool, this is because blood in the colon has a
powerful laxative effect and is rarely retained long enough for transformation to
occur and because the digestive enzymes present in the small bowel are not
present in the colon. Up to 15% of patients presenting with haemochezia will have
an upper gastrointestinal source of haemorrhage.

As a general rule right sided bleeds tend to present with darker coloured blood
than left sided bleeds. Haemorrhoidal bleeding typically presents as bright red
rectal bleeding that occurs post defecation either onto toilet paper or into the toilet
pan. It is very unusual for haemorrhoids alone to cause any degree of
haemodynamic compromise.
gathered by dr. elbarky, for free, not intended for profit by anyone elsewhere.

Causes

Cause Presenting features

Colitis Bleeding may be brisk in advanced cases, diarrhoea is


commonly present. Abdominal x-ray may show featureless
colon.

Diverticular Acute diverticulitis often is not complicated by major


disease bleeding and diverticular bleeds often occur sporadically.
75% all will cease spontaneously within 24-48 hours.
Bleeding is often dark and of large volume.

Cancer Colonic cancers often bleed and for many patients this may
be the first sign of the disease. Major bleeding from early
lesions is uncommon

Haemorrhoidal Typically bright red bleeding occurring post defecation.


bleeding Although patients may give graphic descriptions bleeding
of sufficient volume to cause haemodynamic compromise
is rare.

Angiodysplasia Apart from bleeding, which may be massive, these


arteriovenous lesions cause little in the way of symptoms.
The right side of the colon is more commonly affected.

Management
Prompt correction of any haemodynamic compromise is required. Unlike
upper gastrointestinal bleeding the first line management is usually
supportive. This is because in the acute setting endoscopy is rarely helpful.
When haemorrhoidal bleeding is suspected a proctosigmoidoscopy is
reasonable as attempts at full colonoscopy are usually time consuming and
often futile.
In the unstable patient the usual procedure would be an angiogram (either
CT or percutaneous), when these are performed during a period of
haemodynamic instability they may show a bleeding point and may be the
only way of identifying a patch of angiodysplasia.
In others who are more stable the standard procedure would be a
colonoscopy in the elective setting. In patients undergoing angiography
attempts can be made to address the lesion in question such as coiling.
Otherwise surgery will be necessary.
In patients with ulcerative colitis who have significant haemorrhage the
standard approach would be a sub total colectomy, particularly if medical
management has already been tried and is not effective.

Indications for surgery


gathered by dr. elbarky, for free, not intended for profit by anyone elsewhere.

Patients > 60 years


Continued bleeding despite endoscopic intervention
Recurrent bleeding
Known cardiovascular disease with poor response to hypotension

Surgery
Selective mesenteric embolisation if life threatening bleeding. This is most helpful
if conducted during a period of relative haemodynamic instability. If all
haemodynamic parameters are normal then the bleeding is most likely to have
stopped and any angiography normal in appearance. In many units a CT angiogram
will replace selective angiography but the same caveats will apply.

If the source of colonic bleeding is unclear; perform a laparotomy, on table colonic


lavage and following this attempt a resection. A blind sub total colectomy is most
unwise, for example bleeding from an small bowel arterio-venous malformation
will not be treated by this manoeuvre.

Summary of Acute Lower GI bleeding recommendations


Consider admission if:
* Over 60 years
* Haemodynamically unstable/profuse PR bleeding
* On aspirin or NSAID
* Significant co morbidity

Management
All patients should have a history and examination, PR and proctoscopy
Colonoscopic haemostasis aimed for in post polypectomy or diverticular
bleeding

References
http://www.sign.ac.uk/guidelines/fulltext/105/index.html

Next question 

Display my notes on this topic

          
Save my notes

Question stats
gathered by dr. elbarky, for free, not intended for profit by anyone elsewhere.

A 20%
B 14.4%
C 43.6%
D 14.6%
E 7.4%

43.6% of users answered this question correctly

Search eMRCS

Search term Go

 Open MRCS Part A textbook (../review/textbook.php)

External links

+ Suggest a link

Dashboard

9

Question 73 of 74
gathered by dr. elbarky, for free, not intended for profit by anyone elsewhere.

 

A 56 year old man presents with episodes of pruritus ani and bright red rectal
bleeding. On examination there is a mass in the ano rectal region and biopsies
confirm squamous cell cancer. What is the most likely cause?

Anal cancer

Rectal cancer

Soft tissue sarcoma

Retro-rectal cyst

Pilonidal sinus disease

These are features of anal cancer. Anal cancers arise from the cutaneous
epithelium and are therefore typically squamous cell. They are usually sensitive to
chemoradiotherapy.

Please rate this question:

 Discuss and give feedback

Next question 

Rectal bleeding

Rectal bleeding is a common cause for patients to be referred to the surgical clinic.
In the clinical history it is useful to try and localise the anatomical source of the
blood. Bright red blood is usually of rectal anal canal origin, whilst dark red blood is
more suggestive of a proximally sited bleeding source. Blood which has entered
the GI tract from a gastro-duodenal source will typically resemble malaena due to
the effects of the digestive enzymes on the blood itself.

In the table below we give some typical bleeding scenarios together with physical
examination findings and causation.
Cause Type of Features in history Examination findings
bleeding

Fissure in Bright red Painful bleeding that Muco-epithelial defect


gathered by dr. elbarky, for free, not intended for profit by anyone elsewhere.

ano rectal occurs post defecation usually in the midline


bleeding in small volumes. posteriorly (anterior
Usually antecedent fissures more likely to
features of be due to underlying
constipation disease)

Haemorroids Bright red Post defecation Normal colon and


rectal bleeding noted both on rectum. Proctoscopy
bleeding toilet paper and drips may show internal
into pan. May be haemorrhoids. Internal
alteration of bowel haemorrhoids are
habit and history of usually impalpable.
straining. No blood
mixed with stool. No
local pain.

Crohns Bright red Bleeding that is Perineal inspection may


disease or mixed accompanied by other show fissures or
blood symptoms such as fistulae. Proctoscopy
altered bowel habit, may demonstrate
malaise, history of indurated mucosa and
fissures (especially possibly strictures. Skip
anterior) and lesions may be noted at
abscesses. colonoscopy.

Ulcerative Bright red Diarrhoea, weight loss, Proctitis is the most


colitis bleeding nocturnal marked finding. Peri
often incontinence, passage anal disease is usually
mixed of mucous PR. absent. Colonoscopy
with stool will show continuous
mucosal lesion.

Rectal Bright red Alteration of bowel Usually obvious


cancer blood habit. Tenesmus may mucosal abnormality.
mixed be present. Symptoms Lesion may be fixed or
volumes of metastatic disease. mobile depending upon
disease extent.
Surrounding mucosa
often normal, although
polyps may be present.

Image showing a fissure in ano. Typically these are located posteriorly and in the
midline. Fissures at other sites may be associated with underlying disease.
gathered by dr. elbarky, for free, not intended for profit by anyone elsewhere.

(https://d2zgo9qer4wjf4.cloudfront.net/images_eMRCS/swb054b.jpg)
Image sourced from Wikipedia
(https://d2zgo9qer4wjf4.cloudfront.net
(http://en.wikipedia.org/wiki/Anal
/images_eMRCS/swb054b.jpg)
fissure)

Colonoscopic image of internal haemorroids. Note these may often be impalpable.

(https://d2zgo9qer4wjf4.cloudfront.net/images_eMRCS/swb055b.jpg)
Image sourced from Wikipedia
(https://d2zgo9qer4wjf4.cloudfront.net
(http://en.wikipedia.org
/images_eMRCS/swb055b.jpg)
/wiki/Haemorrhoids)

Investigation
All patients presenting with rectal bleeding require digital rectal examination
and procto-sigmoidoscopy as a minimal baseline.
Remember that haemorrhoids are typically impalpable and to attribute
bleeding to these in the absence of accurate internal inspection is
gathered by dr. elbarky, for free, not intended for profit by anyone elsewhere.

unsatisfactory.
In young patients with no other concerning features in the history a carefully
performed sigmoidoscopy that demonstrates clear haemorrhoidal disease
may be sufficient. If clear views cannot be obtained then patients require
bowel preparation with an enema and a flexible sigmoidscopy performed.
In those presenting with features of altered bowel habit or suspicion of
inflammatory bowel disease a colonoscopy is the best test.
Patients with excessive pain who are suspected of having a fissure may
require an examination under general or local anaesthesia.
In young patients with external stigmata of fissure and a compatible history
it is acceptable to treat medically and defer internal examination until the
fissure is healed. If the fissure fails to heal then internal examination
becomes necessary along the lines suggested above to exclude internal
disease.

Special tests
In patients with a malignancy of the rectum the staging investigations
comprise an MRI of the rectum to identify circumferential resection margin
compromise and to identify mesorectal nodal disease. In addition to this CT
scanning of the chest abdomen and pelvis is necessary to stage for more
distant disease. Some centres will still stage the mesorectum with endo
rectal ultrasound but this is becoming far less common.

Patients with fissure in ano who are being considered for surgical
sphincterotomy and are females who have an obstetric history should
probably have ano rectal manometry testing performed together with endo
anal ultrasound. As this service is not universally available it is not
mandatory but in the absence of such information there are continence
issues that may arise following sphincterotomy.

Management

Disease Management

Fissure in ano GTN ointment 0.2% or diltiazem cream applied topically is


the usual first line treatment. Botulinum toxin for those who
fail to respond. Internal sphincterotomy for those who fail
with botox, can be considered earlier in males.
Haemorroids Lifestyle advice, for small internal haemorrhoids can
consider injection sclerotherapy or rubber band ligation. For
external haemorrhoids consider haemorrhoidectomy.
Modern options include HALO procedure and stapled
gathered by dr. elbarky, for free, not intended for profit by anyone elsewhere.

haemorrhoidectomy.

Inflammatory Medical management- although surgery may be needed for


bowel disease fistulating Crohns (setons).

Rectal cancer Anterior resection or abdomino-perineal excision of the


colon and rectum. Total mesorectal excision is now standard
of care. Most resections below the peritoneal reflection will
require defunctioning ileostomy. Most patients will require
preoperative radiotherapy.

Next question 

Display my notes on this topic

          

Save my notes

Question stats

A 60.3%
B 17.9%
C 7.5%
D 6.1%
E 8.2%

60.3% of users answered this question correctly

Search eMRCS

Search term Go

Question 74 of 74
gathered by dr. elbarky, for free, not intended for profit by anyone elsewhere.

A 22 year old man has a long history of ulcerative colitis. His symptoms are well
controlled with steroids. However, attempts at steroid weaning and use of steroid
sparing drugs have repeatedly failed. He wishes to avoid a permanent stoma.
Which of the following is the best operative option?

Pan proctolectomy and end ileostomy

Abdomino perineal excision of the colon and rectum and end colostomy

Abdomino perineal excision of the colon and rectum and construction of


an ileo anal pouch

Pan proctocolectomy and construction of an ileo anal pouch

Sub total colectomy and construction of an ileo anal pouch

Don't confuse AP resection and proctectomy. The former is a cancer related


procedure.

In patients with UC where medical management is not successful, surgical


resection may offer a chance of cure. Those patients wishing to avoid a permanent
stoma may be considered for an ileoanal pouch. However, this procedure is only
offered in the elective setting.

Please rate this question:

 Discuss and give feedback

Surgery for inflammatory bowel disease

Patients with inflammatory bowel disease (UC and Crohns) frequently present in
surgical practice. Ulcerative colitis may be cured by surgical resection
(Proctocolectomy), this is not the case in Crohns disease which may recur and
affect other areas of the gastrointestinal tract.
Ulcerative colitis
Elective indications for surgery include disease that is requiring maximal
therapy, or prolonged courses of steroids.
Longstanding UC is associated with a risk of malignant transformation.
gathered by dr. elbarky, for free, not intended for profit by anyone elsewhere.

Dysplastic transformation of the colonic epithelium with associated mass


lesions is an absolute indication for a proctocolectomy.
Emergency presentations of poorly controlled colitis that fails to respond to
medical therapy should usually be managed with a sub total colectomy.
Excision of the rectum is a procedure with a higher morbidity and is not
generally performed in the emergency setting. An end ileostomy is usually
created and the rectum either stapled off and left in situ, or, if the bowel is
very oedematous, may be brought to the surface as a mucous fistula.
Patients with IBD have a high incidence of DVT and appropriate
thromboprophylaxis is mandatory.
Restorative options in UC include an ileoanal pouch. This procedure can only
be performed whilst the rectum is in situ and cannot usually be undertaken
as a delayed procedure following proctectomy.
Ileoanal pouch complications include, anastomotic dehiscence, pouchitis
and poor physiological function with seepage and soiling.

Crohns disease
Surgical resection of Crohns disease does not equate with cure, but may
produce substantial symptomatic improvement.
Indications for surgery include complications such as fistulae, abscess
formation and strictures.
Extensive small bowel resections may result in short bowel syndrome and
localised stricturoplasty may allow preservation of intestinal length.
Staging of Crohns will usually involve colonoscopy and a small bowel study
(e.g. MRI enteroclysis).
Complex perianal fistulae are best managed with long term draining seton
sutures, complex attempts at fistula closure e.g. advancement flaps, may be
complicated by non healing and fistula recurrence.
Severe perianal and / or rectal Crohns may require proctectomy. Ileoanal
pouch reconstruction in Crohns carries a high risk of fistula formation and
pouch failure and is not recommended.
Terminal ileal Crohns remains the commonest disease site and these
patients may be treated with limited ileocaecal resections.
Terminal ileal Crohns may affect enterohepatic bile salt recycling and
increase the risk of gallstones.

Display my notes on this topic

          
Save my notes
gathered by dr. elbarky, for free, not intended for profit by anyone elsewhere.

Question stats

A 13.4%
B 7.7%
C 14.2%
D 46.5%
E 18.3%

46.5% of users answered this question correctly

Search eMRCS

Search term Go

 Open MRCS Part A textbook (../review/textbook.php)

External links

+ Suggest a link

Dashboard

You might also like